SlideShare una empresa de Scribd logo
1 de 161
Descargar para leer sin conexión
MINISTERIO DE EDUCACION
CONSEJO NACIONAL DE UNIVERSIDADES
UNAN-MANAGUA
UNI
UNAN-LEON
Solucionario de Guía de Estudio de Matemática
Agosto, 2014
UNIDAD DE ARITMÉTICA
1. La expresión 311
+ 311
+ 311
equivale a:
Solución :
Al sumar los tres términos se obtiene
3 311
= 312
2. Al número de tres dígitos 2a3 se le suma el número 326 y da el número de tres dígitos 5b9. Si sabemos que el
número 5b9 es divisible entre 9, entonces a + b es:
Solución :
Al sumar ambos números se obtiene
2a3 + 326 = 5b9
como el número 5b9 es divisible entre 9; esto signi…ca que la suma de los valores absolutos de sus cifras es
múltiplo de 9, entonces 5 + b + 9 = 18; de aqui b = 4; entonces a + 2 = b; lo cual signi…ca que a = 2 y por tanto
a + b = 6:
3. A una determinada cantidad le sumo el 10% de sí misma y a la cantidad así obtenida le resto su 10%. ¿Qué
porcentaje de la cantidad original me queda?
Solución :
Sea x = Cantidad Inicial , entonces
x + 0:1x = 1:1x
es la cantidad aumentada en un 10%, pero a ésta le restamo su 10% y obtenemos
1:1x 0:1 (1:1x) = 0:99x
lo cual representa un 99% de la cantidad inicial.
4. Al simpli…car [(9 4) + ( 10 + 3)] ((6) ( 5)) [( 12 + 8) (6 9) (95 90)] el resultado es:
Solución :
Al efectuar las operaciones indicadas se tiene
[(9 4) + ( 10 + 3)] ((6) ( 5)) [( 12 + 8) (6 9) (95 90)] = [5 + ( 7)] ( 30) [( 4) ( 3) (5)]
= ( 2) ( 30) 60
= 60 60
= 1
2
5. ¿Cuántos divisores diferentes tiene el número 2000?
Solución :
La descomposición del 2000 en factores primos es 2000 = 24
53
; sumando 1 a cada exponente y multiplicando
dichas expresiones, la cantidad de divisores será (4 + 1) (3 + 1) = 20
6. Al simpli…car 4 (3)
2
6 3
p
4 + 2 [5 (7) 15 3] 4 12 9. El resultado es:
Solución :
Al efectuar las operaciones indicadas y respetando el orden de prioridad de los operadores aritméticos, se tiene
4 (3)
2
6 3
p
4 + 2 [5 (7) 15 3] 4 12 9 = 36 6 6 + 2 [35 15 3] 4 12 9
= 6 6 + 2 [35 5] 4 12 9
= 60 4 12 9
= 240 12 9
= 20 9
= 11
7. Simpli…que
1
2
5
3
3
4
3
4
3
5
6
17 1
Solución:
1
2
5
3
3
4
3
4
3
5
6
17 1 =
1
2
5
4
3
10
9
17 1
=
3
4
17
9
17 1
=
27
68
17 1
=
27
4
1
= 7
3
4
8. ¿Cuántos números válidos (números que no tienen al cero como primer dígito) de cinco cifras se pueden escribir
usando solo los dígitos 0; 1; 2; 3 y 4?
Solución :
El número 0 no puede ser el primer dígito, entonces, los otros lugares pueden ser ocupados por cualquieras de
los 5 dígitos restantes, es decir,
4 5 5 5 5 = 4 54
3
9. Pedro tiene 69 años y su edad excede a la de Juan en un 15%. ¿Qué edad tiene Juan?
Solución :
Una de la formas de resolver este problema es
69 ! 115%
x ! 100%
, de aqui x =
69 100%
115%
= 60
10. En una ciudad,
2
3
de los hombres están casados con los
3
5
de las mujeres. Si nunca se casan con forasteros,
¿Cuál es la proporción de solteros en dicha ciudad?
Solución :
Sea x = Cantidad de Hombres
y = Cantidad de Mujeres
2
3
x =
3
5
y ; de aqui, y =
10
9
x
x +
10
9
x 2
2
3
x
x +
10
9
x
=
7
19
11. El resultado de 125
2
3 + 16
1
2 + 343
1
3
1
2
es:
Solución:
125
2
3 + 16
1
2 + 343
1
3
1
2
= 53
2
3 + 24
1
2 + 73
1
3
1
2
= 52
+ 22
+ 7
1
2
= [36]
1
2
= 6
12. Obtenga el resultado de (0:027)
1
3 + 2560:75
3 1
+ (4:5)
0
Solución:
(0:027)
1
3 + 2560:75
3 1
+ (4:5)
0
= 3
10
3
1
3
+ 28
3
4 1
3 + 1
= 3
10
1
+ 26 1
3 + 1
= 10
3 + 64 1
3 + 1
= 68
13. ¿Cuál es el valor de a en (3a)
5
= 248832?
Solución:
(3a)
5
= 248832
35
a5
= 210
35
a5
=
210
35
35
a = 22
a = 4
4
14. Un equipo de jugadores ganó 15 juegos y perdió 5. ¿Cuál es la razón geométrica de los juegos ganados a los
jugados?
Solución :
Total de juegos = 20, Total de juegos ganados =15, dicha proporción es
15
20
=
3
4
15. Si x es un número par y y es un número impar. ¿Cuál.de las siguientes a…rmaciones siempre es falsa?
Solución:
La falsa es y+y
2 = 2y
2 = y es par, porque aqui se produce una contradicción, y no puede ser par e impar a la
vez.
16. El mínimo común múltiplo de dos números es 105 y su máximo común divisor es 5. ¿Cuál de los siguientes
números puede representar la suma de estos dos números?
Solución :
Como su m.c.d es 5, signi…ca que 5 es el único divisor común. Por tanto, se trata de dos números múltiplos de
5. Como su m.c.m. es 105, entonces
105
5
= 21. Descomponemos el 21 en el producto de dos divisores, esto es
3 y 7. Por tanto, uno de los números es 15 = 3 5, el otro es 35 = 5 7 , por tanto, su suma es 15 + 35 = 50
17. La maestra distribuyó la misma cantidad de dulces entre cada uno de 5 niños y se quedó tres para ella misma.
No se acuerda cuántos dulces tenía, pero se acuerda que era un múltiplo de 6 entre 65 y 100. ¿Cuántos dulces
tenía?
Solución:
Como se quedó con 3 dulces, el número inicial de dulces termina en 3 o en 8, pero como es un múltiplo de 6,
es par, por lo que termina en 8. La única posibilidad es 78.
18. El resultado de
2
6
6
6
4
5 4
0
B
B
B
@
1
2
2
1
1
2
1
1
C
C
C
A
3
7
7
7
5
es
Solución :
Al desarrollar la fracción se tiene
5 4
0
B
B
B
@
1
2
2
1
1
2
1
1
C
C
C
A
= 5 4
3
2
= 1
5
19. El resultado de
2
3
4
5
6
7
es:
Solución :
Al realizar operaciones básicas aritmética se tiene
2
3
4
5
7
6
=
2
3
14
15
=
4
15
20. Juan gasta el 20% de sus ingresos en el pago de impuestos y 20% del resto en el pago de la mensualidad de su
casa. ¿Qué porcentaje de su ingreso gasta en el pago de su casa?
Solución :
El valor gastado en el pago de impuesto es
0:2x
luego lo que le queda es
x 0:2x = 0:8x
por tanto, el pago de la mensualidad de la casa es
0:2 (0:8x) = 0:16x
lo cual corresponde a un 16%
21. ¿Cuánto gano o pierdo si vendo por los
3
5
de los
7
2
del costo de un juguete que me ha costado C$40:00?
Solución :
Aplicando operaciones básicas aritméticas
3
5
7
2
40 = 84:0
luego se ha ganado 84 40 = 44:00 córdobas.
22. Cuatro personas juntaron sus ahorros para abrir un negocio aportando el 15%, 20%, 25% y 40%, respectiva-
mente, del monto total. Si la menor de las aportaciones fue de C$9; 000, la mayor de las aportaciones fue
de:
Solución :
La menor de la aportaciones equivale
0:15x = 9000
luego el monto total es
x = 60; 000
La mayor de las aportaciones equivale
0:4 (60000) = 24; 000
6
23. De acuerdo al Reglamento de Admisión de una universidad, el puntaje total alcanzado por un estudiante está
formado por el 70% de la nota obtenida en el Examen de Admisión y el 30% de su promedio de los dos últimos
años de bachillerato. Si un estudiante alcanza un puntaje total de 81 y su promedio de los dos últimos años de
bachillerato es 95, ¿qué puntaje obtuvo en el examen de admisión?
Solución :
Sea x la nota obtenida en el examen de admisión, entonces
0:7x + 0:3 (95) = 81
x = 75
24. Un grupo de amigas va de paseo y disponen de C$240:00 para la compra de sus pasajes. Si compran pasajes
de C$30:00, les sobra dinero; pero si compran pasajes de C$40:00, les falta dinero. ¿Cuántas amigas van de
paseo?
Solución :
Sea n la cantidad de amigas, entonces
30n < 240
40n > 240
y la solución de dicho sistema de ecuación se encuentra en el intervalo (6; 8), de aqui que la solución entera es
n = 7:
25. En el parqueo de una cierta universidad, entre carros y motos hay 20 vehículos. Sabiendo que el número total
de ruedas es 70. ¿Cuántos carros hay?
Solución :
Sean x la cantidad de carros y (20 x) la cantidad de motos respectivamente, entonces
4x + 2 (20 x) = 70
x = 15
por tanto, hay 15 carros y 5 motos.
26. Un estudiante de una cierta universidad proveniente del interior del país gasta la cuarta parte de su “mesada”
en el alquiler de una habitación, la mitad en comida, la quinta parte en materiales educativos y el resto, C$
100.00, en recreación. ¿Cuánto es la “mesada”de este estudiante?
Solución :
Sea x la cantidad de la mesada recibida, entonces
x
x
4
x
2
x
5
= 100
x = 2000
7
27. El hielo disminuye su volumen en un 9% cuando se derrite. Si se derriten 1000cc de hielo, ¿Cuál es el volumen
del líquido que se forma?
Solución :
Hay que obtener el 9% de 1000, es decir
0:09 1000 = 90cc
por tanto, el volumen que se forma es de
1000cc 90cc = 910cc
28. ¿Cuál de las siguientes expresiones es impar para cualquier entero n?
Solución :
La expresión 2n2
es un número par y 2003 es un número impar, por tanto, su suma siempre será impar
29. El resultado de
2
6
6
6
4
5 4
0
B
B
B
@
1
2
2
1
1
2
1
1
C
C
C
A
3
7
7
7
5
es
Solución :
Al desarrollar la fracción se tiene
5 4
0
B
B
B
@
1
2
2
1
1
2
1
1
C
C
C
A
= 5 4
3
2
= 1
30. Calcular el producto L H sabiendo que L = a + b + c , H = d + c = f + g siendo a; b; c; d; f; g números
naturales y que b f = 91 ; a d = 18 ; c d = 16 ; b g = 39
Solución :
Como sabemos que b f = 91 ; a d = 18 ; c d = 16 ; b g = 39; podemos aplicar la teoria de máximo
común divisor y obtenemos : b = gcd (39; 91) = 13 , d = gcd (16; 18) = 2; de aqui f = 7; c = 8; a = 9; g = 3 y
entonces L = a + b + c , H = d + c = f + g; y sustituyendo L = 9 + 13 + 8 = 30; H = 2 + 8 = 7 + 3 = 10;
por tanto el producto es 300:
31. Al desarrollar la expresión
qpp
625a8
2
el resultado es:
Solución: qpp
625a8
2
=
h
54
a8
1
8
i2
=
h
5
1
2 a
i2
= 5a2
8
32. El resultado de
q
a 3
p
a
p
a es:
Solución: q
a 3
p
a
p
a =
q
a
3
pp
a a2
=
q
3
p
a3
p
a3
=
q
3
pp
a9
=
12
p
a9
=
4
p
a3
33. Una epidemia mató los
5
8
de las reses de un ganadero y luego él vendió los
2
3
de las que le quedaban. Si aún
tiene 216 reses, ¿Cuántas tenía al principio, cuántas murieron y cuántas vendió?
Solución :
Formamos una ecuación lineal x
5
8
x
2
3
x
5
8
x = 216, cuya solución es x = 1728; este valor son las
reses que tiene al inicio, las que mata la epidemia son
5
8
(1728) = 1080; las que le quedan son 1728 1080 = 648
y las vende son
2
3
(648) = 432
34. Una gallina pone dos huevos en tres días. ¿Cuántos días se necesitan para que cuatro gallinas pongan dos
docenas de huevos?
Solución :
Este es un problema de proporcionalidad compuesta,
Gallinas Huevos Dias
1 2 3
4 24 x
De aqui
4
1
2
24
=
3
x
; x = 9
35. El 41
2
3
% es equivalente a:
Solución:
Usando una regla de tres simple:
100% ! 1
125
3
% ! x
Tenemos que equivale a
5
12
9
36. Halla el número cuyo 3:6 porciento vale
3 + 4:2 0:1
1 0:3 2
1
3
0:3125
Solución :
Llamamos N al número buscado y A a la expresión dada. Entonces: A =
(3:6 N)
100
; de aqui N = A
100
3:6
;
haciendo las operaciones respectivas, se obtiene que
3 + 4:2 0:1
1 0:3 2
1
3
0:3125
100
36
= 4000
37. Al realizar la operación 4:62 10 2
2:2 10 4
se obtiene el número
Solución :
Al realizar la división indicada
4:62
2:2
102
= 210
38. Un albañil y su ayudante pueden hacer una obra en 24 días. Después de 4 días de trabajo, el ayudante se retira
y el albañil termina lo que falta en 30 días. El número de días que podría hacer la obra el ayudante trabajando
solo es:
Solución:
Al plantear una regla de tres compuesta
Hombres Dias Proyectado Dias Reales
2 24 20
1 x 30
De aqui
x =
2 24 30
1 20
= 72
39. Al simpli…car la expresión
21
+ 20
+ 2 1
2 2 + 2 3 + 2 4
se obtiene:
Solución:
Al reescribir la expresión dada
2 + 1 +
1
2
1
4
+
1
8
+
1
16
y al efectuar operaciones básicas de suma y cociente, se tiene que el valor dado es 8
10
40. Se va a tender una línea eléctrica de 35:75km de longitud con postes separados entre sí por una distancia de
125m. Si el primer poste se coloca al inicio de la línea, y el último al …nal ¿cuántos postes serán necesarios en
total?
Solución:
Al hacer la conversión de 35:75km a metros se tiene
35:75 1000 = 35750
lo cual a dividir entre 125; se tendría la cantidad de poste utilizado, es decir
35750
125
= 286
pero como el primer poste se coloca al inicio de la línea, se tiene que el total de poste es de 287:
41. La operación está de…nida por a b = 2ab 3b en la que a y b son números enteros. ¿Cuál es el resultado
de [4 ( 1)] ( 3)?
Solución:
Realizando las operaciones por partes:
[4 ( 1)] = 2 (4) ( 1) 3 ( 1) = 8 + 3
[( 5) ( 3)] = 2 ( 5) ( 3) 3 ( 3) = 30 + 9 = 39
42. ¿Cuál es la diferencia entre el 50% de 50 y el 20% de 20?
Solución:
Calculemos los porcentajes dados
0:5 (50) = 25
0:2 (20) = 4
por tanto, la diferencia dada es 21
43. En la sustracción a b = c, la suma del minuendo, el sustraendo y la diferencia es 32. ¿Cuál es el valor del
minuendo?
Solución:
Sabemos que
a + b + c = 32
pero a b = c; entonces
a + b + a b = 32
a = 16
11
44. El resultado de la operación
2
2
5
4
5
+
3
1
3
4
3
4
1
4
1
2
+
5
1
5
24
7
20
11
2
es:
Solución:
2
2
5
4
5
+
3
1
3
4
3
4
1
4
1
2
+
5
1
5
24
7
20
11
2
=
2 + 2
15
2 + 1
5
77
40
=
4
77
10
77
40
= 1
45. El valor numérico de la expresión
42
(3 2)
2
( 6 + 1)
2 es:
Solución:
Al desarrollar la expresión dada
42
(3 2)
2
( 6 + 1)
2 =
16 1
25
=
3
5
46. Si A comió
1
4
de un queque, B comió
1
3
de lo que quedó después que A comió; C comió
1
2
de lo que quedó
después que A y B comieron ¿Qué parte del queque quedó?
Solución :
Sea x el total del queque, entonces al restar las partes que se comieron, se tiene
x
1
4
x
1
3
x
1
4
x
1
2
x
1
4
x
1
3
x
1
4
x
1
4
x
47. Con los
2
7
del dinero que tenía, Mara compró gaseosas para festejar su cumpleaños. Con los
3
5
del dinero que
le sobró compró hamburguesas. Al …nal Mara se quedó con C$100:00. ¿Cuánto gastó Mara en hamburguesas?
Solución :
Al aplicar los datos
x
2
7
x
3
5
x
2
7
x = 100
x = 350
12
lo gastado en hamburguesa es
3
5
x
2
7
x =
3
5
350
2
7
350 = 150
48. En una fábrica 60% de los artículos son producidos por una máquina A y el resto por otra máquina B. Si 3%
de los artículos producidos por la máquina A y 8% de los producidos por la máquina B resultaron defectuosos
¿cuál es el porcentaje de artículos defectuosos producidos en toda la fábrica.
Solución :
Sea x el total de artículos producidos por la máquina A, entonces según los datos
0:6x + 0:4 (x 0:6x) = 100
49. La última vez que llené el tanque de gasolina, mi automóvil había recorrido 47; 286km. Ahora que acabo de
llenarlo, la bomba marcó 22 litros y el cuentakilómetros marcaba 47; 506 km recorridos. Si el litro de gasolina
cuesta C$20. ¿Cuánto me cuesta en promedio recorrer un kilómetro?
Solución :
Haciendo la diferencia
47506 47286 = 220
el promedio en kilometraje es
220
20
= 11
50. Un frasco contiene 12 onzas de una solución cuya composición es una parte de ácido por cada 2 partes de agua.
Se agrega a otro frasco que contiene 8 onzas de una solución que contiene 1 parte de ácido por cada 3 partes
de agua. ¿Cuál es la razón entre el ácido y el agua de la solución obtenida?
Solución :
La relación en el frasco de 12 onzas es
4
8
y en el frasco de 8 onzas es
2
6
; entonces la relación total entre el ácido
y el agua es
6
14
=
3
7
51. Por un préstamo de 20; 000 pesos se paga al cabo de un año 22; 400 pesos. ¿Cuál es la tasa de interés cobrada?
Solución :
La fórmula dada es F = P (1 + i)
t
entonces
22400 = 20000 (1 + i)
i = 0:12
lo cual representa 12%
13
52. Si un número N se divide entre 4, se obtiene 9 de cociente y 1 de residuo. Si N se divide entre M, se obtiene
5 de cociente y 2 de residuo. ¿Cuál es el valor de M?
Solución :
De acuerdo a los datos del problema
N = cd + R = 36 + 1 = 37
N = 5M + 2
al sustituir los datos
37 2
5
= M
M = 7
53. Un contratista compró 4000 piedras y las vendió por 8,800 córdobas. ¿Cuánto pagó el por cada piedra si ganó,
en relación a lo que pagó, un porcentaje igual a 5 veces el número de córdobas que a él le costó cada piedra?
Solución :
El costo real de cada piedra es
8800
4000
= x + 5x
x
100
2:2 = x +
x2
20
x = 2
54. El valor de la expresión
1
2
2
+ ( 2)
2
( 2)
3 es:
Solución :
Al reescribir la expresión dada
1
2
2
+ ( 2)
2
( 2)
3 =
22
+ 4
8
= 1
55. Calcular a cuánto asciende el interés simple producido por un capital de 25 000 córdobas invertido durante 4
años a una tasa del 6 % anual.
Solución :
La fórmula a utilizar es
I = C i t = 25000 0:06 4 = 6; 000
14
56. En el año 1982 la edad de la tierra era de 1:3 1017
segundos y la de la pirámide de Keops, 1:5 1011
segundos.
La diferencia de edad entre la tierra y la pirámide en notación cientí…ca es:
Solución :
Sea d la diferencia de edad, entonces
d = 1:3 1017
1:5 1011
d = 1011
1:3 106
1:5
d = 1011
(1299998:5)
d = 1:2999985 1017
seg
57. La luz recorre aproximadamente 3 105
km por segundo. ¿Cuántos metros recorrerá en 365 días? El resultado
en notación cientí…ca es:
Solución:
En un día hay 24 (60) (60) = 86; 400seg, en 365 días hay 365 (86; 400) = 31; 536; 000seg. Como la luz recorre
3 105
103
m, en esos segundos la luz recorrerá:
31; 536; 000 3 108
m = 94; 608 1011
m
= 9:4608 1015
m
58. La velocidad de la luz es aproximadamente de 3 105
km=seg: La estrella más cercana a la tierra está a 4300
años luz de distancia. La distancia en km y escrita en notación cientí…ca es:
Solución:
En un día hay 24 (60) (60) = 86; 400seg, en 365 días hay 365 (86; 400) = 31; 536; 000seg. Como la luz recorre
3 105
103
m, en esos segundos la luz recorrerá 31; 536; 000 3 108
m = 94; 608 1011
m, entonces:
31; 536; 000 3 108
m = 94; 608 1011
m
= 9:4608 1012
m=seg
La estrella más cercana está a 4300 AL, entonces
4300 9:4608 1012
= 40681:44 1012
= 4:068144 104
1012
= 4:068144 1016
km
59. ¿Qué altura tendría una pila de 1; 000; 000 de hojas de cuaderno si se necesitan 10 hojas para tener 1mm?
Solución:
Utilizando una regla de tres simple:
15
1; 000; 000 ! x
10 ! 1mm
Entonces, la altura x de la pila es 1; 000; 000 10 = 100; 000mm = 105
mm.
60. ¿Cuántos rieles de 15m se necesitan para enlazar a una fábrica con la estación que dista 765m?
Solución:
Se necesitan 765m 15m = 51 rieles.
61. ¿Cuántos al…leres de 3:5cm de largo pueden fabricarse con un alambre de latón de 152:07m, sabiendo que hay
una pérdida de 2mm de alambre por al…ler?
Solución:
En total hay 152:07m = 152:07 100 = 15; 207cm de alambre y se pierde 2mm = 2 10 = 0:2cm de alambre
por cada al…ler. Entonces, si x representa la cantidad de al…leres que pueden fabricarse:
15207 = (3:5 + 0:2) x
15207
3:7
= x
4110 = x
Se pueden fabricar 4,110 al…leres.
62. Para ir a clase, Pedro tiene que andar por término medio 1; 520 pasos de 62 cm. ¿Cuántos km habrá recorrido
durante un año escolar de 210 días si va al colegio y vuelve a su casa?
Solución:
De su casa a la escuela (y viceversa) recorre 1; 520 0:62 = 942:40m, al día recorre
2 942:40m = 1884:8m = 1884:8 1000 = 1:8848km:
Durante el año habrá recorrido 210 1:8848 = 395:8km.
63. Se ha necesitado 54; 000 losetas para pavimentar los 2; 430 m2
que miden las aceras de una calle. ¿Cuál es en
mm2
la super…cie de una loseta?
Solución:
La super…cie de cada loseta es de 2; 430 m2
54; 000 = 0:045m2
. Como 1m2
= 1; 000; 000mm2
. Entonces:
0:045 1; 000; 000mm2
= 45; 000mm2
16
64. Si el m2
de un terreno vale 2 dolar, ¿Cuántos dólares vale comprar un campo de 7 Ha?
Solución:
Como 7Ha = 7 10; 000m2
= 70; 000m2
, entonces comprar el campo cuesta 70; 000 $2 = $140; 000.
65. La isla mayor de la Tierra es Groenlandia y mide 2; 180; 000 km2
y una de las más pequeñas es Cabrera, con
2000 Ha. ¿Cuántas veces cabe Cabrera en Groenlandia?
Solución:
Como 2; 180; 000 km2
= 2; 180; 000 100 = 218; 000; 000Ha. Entonces la isla Cabrera cabe 218; 000; 000Ha
2000 Ha = 109; 000 veces en Groelandia.
66. Una tinaja que contiene 0; 4 m3
de aceite ha costado 800 euros ¿a cuántos euros resulta el litro?
Solución:
Como 0; 4m3
= 0; 4 1000 = 400l, el precio del aceite por litro es 800 400 = 2 euros.
67. Un caramelo tiene un volumen de 1; 3 cm3
. ¿Cuántos caramelos caben en una caja de 0; 4498 dm3
?
Solución:
Como 0; 4498 dm3
= 0; 4498 1000cm3
= 449; 8cm3
. En la caja caben 449; 8 1; 3 = 346 caramelos.
68. Los trozos cúbicos de jabón de 5 cm de arista se envían en cajas cúbicas de 60 cm de arista. ¿Cuántos trozos
puede contener la caja?
Solución:
El volumen de los trozos de jabón es (5cm)
3
= 125cm3
y el volumen de cada caja es (60cm)
3
= 216; 000cm3
.
Entonces cada caja puede contener 216; 000 125 = 1728 trozos de jabón.
69. ¿Cuántas botellas de 750 cm3
se necesitan para envasar 300 litros de refresco.
Solución:
Como 750 cm3
= 750 1000 = 0:75 litros, entonces se necesitan 300 0:75 = 400 botellas para envasar 300
litros de refresco.
70. La capacidad de un depósito de gasolina es 1500 litros. ¿Cuál es su volumen en cm3
?
Solución:
El volumen del depósito es de 1500 1000 = 150; 000cm3
.
17
71. Un camión transporta 50 cajas con botellas llenas de agua. Cada caja contiene 20 botellas de litro y medio.
Una caja vacía pesa 1500 g, y una botella vacía, 50 g. ¿Cuál es el peso total de la carga?
Solución:
Cajas vacías: 50 1500g = 75; 000g
Botellas vacías: 50 20 50g = 50; 000g
Cajas llenas : 50 20 1:5l = 1500l = 1500 1000g = 1; 500; 000g
El peso total de la carga es de
75; 000g + 50; 000g + 1; 500; 000g = 1; 625; 000g = 1; 625; 000 1000 = 1; 625kg:
72. Si para construir un muro necesito 2 toneladas de cemento, ¿cuántos sacos de 25 kilos de cemento tendré que
comprar?
Solución:
Como 2 toneladas = 2 1000 = 2; 000kg, se tienen que comprar 2; 000 25 = 80 sacos.
73. Un barco transporta 2800 toneladas de mercancía. ¿Cuántos vagones harán falta para transportar esa mercancía
si cada vagón carga 1400 kg?
Solución:
Como 2800 toneladas = 2800 1000 = 2; 800; 000kg, hacen falta 2; 800; 000 1; 400 = 2000 vagones.
74. La temperatura del cuerpo humano es 37 C. ¿A cuántos grados Fahrenheit equivalen?
Solución:
Para convertir grados celsius a Fahrenheit se utiliza la siguiente fórmula:
o
F = o
C
9
5
+ 32
En este caso:
o
F = 37
9
5
+ 32 = 66:6 + 32 = 98:6o
F
75. Para asar un pollo se necesita que el horno de la cocina alcance una temperatura de 374 F. ¿A qué temperatura
debo …jar el graduador para asar el pollo, si la graduación está en grados centígrados ( C)?
Solución:
Para convertir grados Celsius a Fahrenheit se utiliza la siguiente fórmula:
o
C = (o
F 32)
5
9
En este caso:
o
C = (374 32)
5
9
= 342
5
9
= 190o
C
18
UNIDAD DE ÁLGEBRA
1. Dado el polinomio lineal f(x) = x
1
2
; la suma f(x) + f(x +
1
4
) + f(x +
2
4
) + f(x +
3
4
) es igual a:
Solución:
Al evaluar el polinomio f (x) = x
1
2
; se obtiene que
f (x) = x
1
2
f x +
1
4
= x +
1
4
1
2
= x
1
4
f x +
2
4
= x +
2
4
1
2
= x
f x +
3
4
= x +
3
4
1
2
= x +
1
4
de donde
f (x) + f x +
1
4
+ f x +
2
4
+ f x +
3
4
= x
1
2
+ x
1
4
+ x + x +
1
4
= 4x
1
2
2. Si x + y = 1 y xy = 1 , ¿cuál será el valor de x3
+ y3
?
Solución:
Elevando al cubo la expresión (x + y) = 1, y aplicando las condiciones dadas en el ejercicio, se obtiene
(x + y)
3
= x3
+ 3x2
y + 3xy2
+ y3
= 1
x3
+ 3xy (x + y) + y3
= 1
x3
+ 3 (1) (1) + y3
= 1
x3
+ y3
= 2
3. Si a = 1; b = 3; c = 5, entonces
a + b ja bj
jaj + jbj + jcj
es igual a:
Solución:
Haciendo las debidas sustituciones resulta
1 + 3 j 1 3j
j 1j + j3j + j5j
=
2 4
9
=
2
9
:
4. El valor numérico de la expresión
a2
a + b2
a3
b3
a2
b
(a2 + b2) (2a 3b2)
para a = 1 y b = 2 es:
Solución :
Al sustituir los valores respectivos se obtiene
(1)
2
1 + ( 2)
2
(1)
3
( 2)
3
(1)
2
( 2)
(1)
2
+ ( 2)
2
2 (1) 3 ( 2)
2
=
27
10
19
5. Las raíces de la ecuación ax2
+ bx + c = 0 serán recíprocas si:
Solución :
Las raíces de la ecuación seran recíprocas si al multiplicarla el resultado es 1, de la fórmula general se puede
ver que
b +
p
b2 4ac
2a
!
b
p
b2 4ac
2a
!
= 1
b2
b2
4ac
4a2
= 1
de aquí, c = a
6. El resultado de (bn
5ym
) (5ym
+ bn
) es:
Solución :
El producto indicado es un producto notable y su resultado es
(bn
)
2
(5ym
)
2
= b2n
25y2m
7. La descomposición en factores de la expresión 3x2
2x 8 es:
Solución :
Al factorizar dicha expresión se tiene
3x2
2x 8 = (3x + 4) (x 2)
8. La descomposición en factores de la expresión x3
64y3
es
Solución :
Al factorizar se tiene
x3
64y3
= (x 4y) 4xy + x2
+ 16y2
9. La simpli…cación de
a2
4b2
ab + 2b2
3a2
5ab 2b2
3a2 + ab
es
Solución :
Al factorizar los diferentes términos de las fracciones, se tiene
a2
4b2
ab + 2b2
3a2
5ab 2b2
3a2 + ab
=
(a + 2b) (a 2b)
b (a + 2b)
(a 2b) (3a + b)
a (3a + b)
=
(a + 2b) (a 2b)
b (a + 2b)
a (3a + b)
(a 2b) (3a + b)
=
a
b
20
10. Al simpli…car la expresión
1
a
1
p
a
1
p
a
+
1
a
se obtiene
Solución :
Al determinar el mínimo común de ambos denominadores
p
a a
a
p
a
a +
p
a
a
p
a
=
p
a a
p
a + a
al racionalizar el denominador, obtenemos
p
a a
p
a + a
p
a a
p
a a
=
(
p
a a)
2
a a2
=
a1=2
1 a1=2 2
a (1 a)
=
(1
p
a)
2
1 a
11. El resultado de la siguiente operación
1
x 1
+
12x2
4x
4x2 11x 3
3x2
+ 8x 3
x2 9
es
Solución :
Al desarrollar las operaciones indicadas y factorizando, se tiene
1
x 1
+
4x (3x 1)
(4x + 1) (x 3)
(x + 3) (3x 1)
(x 3) (x + 3)
1
x 1
+
4x (3x 1)
(4x + 1) (x 3)
(x 3) (x + 3)
(x + 3) (3x 1)
1
x 1
+
4x
4x + 1
4x2
+ 1
4x2 3x 1
4x2
+ 1
(4x + 1) (x 1)
12. Al desarrollar
x
y
y
x
2
se obtiene
Solución :
Desarrollando el cuadrado
x
y
y
x
2
=
x2
y2
xy
2
=
x4
2x2
y2
+ y4
x2y2
21
13. Al racionalizar el denominador de la fracción
x 2
3 +
p
2x + 5
se obtiene
Solución :
Al multiplicar por su conjugado
x 2
3 +
p
2x + 5
3
p
2x + 5
3
p
2x + 5
=
p
2x + 5 3
2
14. El conjunto solución de la ecuación
3x
x 5
= 1 +
15
x 5
es
Solución :
Al multiplicar por el mínimo común denominador
(x 5)
3x
x 5
= (x 5) 1 +
15
x 5
3x = x 5 + 15
2x = 10
x = 5
15. El valor de k que proporciona sólo una solución real de la ecuación x2
+ kx + k = 2 3x es:
Solución :
Una ecuación de segundo orden tiene una solución si el discriminante b2
4ac = 0; entonces, al reescribir dicha
ecuación en la forma x2
+ (k + 3) x + (k + 2) = 0 y al analizar su discriminante, se tiene
(k + 3)
2
4 (1) (k + 2) = 0
y al resolver dicha ecuación, se tiene que k = 1
16. Al resolver el sistema de ecuaciones
8
><
>:
2
3x + y
+
4
3x y
= 3
2
3x + y
4
3x y
= 1
, se obtiene que el valor de la variable y es:
Solución :
Al sumar ambas ecuaciones,
4
3x + y
= 4
3x + y = 1
y = 1 3x
sustituyendo este valor en la primera ecuación, obtenemos
2
3x + 1 3x
+
4
3x 1 + 3x
= 3
4
6x 1
= 1
x =
6
6
y al sustituir en y = 1 3x; obtenemos y =
3
2 22
17. Al efectuar
x2
4
(x 2)
2 +
(x + 2)
2
x2 4
se obtiene :
Solución :
La expresión dada se puede reescribir por
(x + 2) (x 2)
(x 2)
2 +
(x + 2)
2
(x + 2) (x 2)
x + 2
x 2
+
x + 2
x 2
2 (x + 2)
x 2
18. Al resolver la ecuación
x + 1
x 1
+
2x 1
x + 1
= 4 se obtiene que la diferencia entre la mayor y la menor de las
raíces es :
Solución :
La expresión dada se puede reescribir por
(x + 1)
2
+ (2x 1) (x 1)
x2 1
= 4
3x2
x + 2 = 4x2
4
x2
+ x 6 = 0
al resolver dicha ecuación, se tiene que las soluciones reales son x1 = 3 ; x2 = 2; por tanto, la diferencia
entre las raíces es 5
19. Al resolver el sistema de ecuaciones
( p
2x +
p
3y
2
= 5 + 2
p
6xy
2x 3y = 1
, se obtiene que el valor de la variable
y es:
Solución :
Podemos ver que 2x = 1 + 3y al sustituir en la ecuación dada, se tiene que
p
1 + 3y +
p
3y
2
= 5 + 2
p
(1 + 3y) 3y
p
1 + 3y +
p
3y
2
= 5 + 2
p
3y (1 + 3y)
desarrollando el cuadrado
1 + 3y + 2
p
3y (1 + 3y) + 3y = 5 + 2
p
3y (1 + 3y)
4 + 6y = 0
y =
2
3
23
20. El conjunto solución de la desigualdad x3
+ x2
2x > 0 es :
Solución :
Al factorizar dicha expresión se tiene
x (x + 2) (x 1) > 0
entonces los números críticos son
x = 0; x = 2; x = 1
entonces
Expresión x x + 2 x 1 Signo
x < 2
2 < x < 0 + +
0 < x < 1 + +
x > 1 + + + +
por tanto, el conjunto solución está de…nido por los intervalos donde está el signo positivo, es decir
( 2; 0) [ (1; +1)
21. El valor de k de manera que la ecuación 2x2
+ kx + 4 = 0 tenga una raíz igual a 3 es:
Solución :
Sabemos que la raices de la ecuación cuadrática tiene la forma
b
p
b2 4ac
2a
entonces, como una de las raices es 3; obtenemos
k
p
k2 4 (2) (4)
2 (2)
= 3
k =
22
3
22. El conjunto solución de la desigualdad jx +
2
3
j 2 es
Solución :
Aplicando propiedad de valor absoluto
2 x +
2
3
2
2
2
3
x 2
2
3
8
3
x
4
3
24
23. El conjunto solución de la desigualdad 1
7 x
2
3 es
Solución :
Aplicando propiedad de valor absoluto
1
7 x
2
3
2 7 x 6
2 7 x 6 7
multiplicando por 1 y cambiando el sentido de la desigualdad
5 x 1
1 x 5
en forma de intervalo se tiene [ 1; 5]
24. El conjunto solución de la desigualdad j5 2xj < 7, está dado por el intervalo
Solución :
Aplicando propiedad de valor absoluto
7 < 5 2x < 7
7 5 < 2x < 7 5
12 < 2x < 2
multiplicando por
1
2
y cambiando el sentido de la desigualdad
6 > x > 1
1 < x < 6
la cual se puede escribir en notación de intervalo por ( 1; 6)
25. El conjunto solución de la desigualdad
(x + 10) (x 2)
x2 7x 8
0 es
Solución :
Factorizando la expresión del denominador, obtenemos
x2
7x 8 = (x + 1) (x 8)
entonces los puntos criticos son
x = 10 ; x = 1 ; x = 2 ; x = 8
25
entonces
Expresión x + 10 x 2 x + 1 x 8 Signo
x 10 +
10 x < 1 +
1 < x 2 + + +
2 x < 8 + + +
x > 8 + + + + +
por tanto, el conjunto solución está de…nido por los intervalos donde está el signo negativo, es decir
[ 10; 1) [ [2; 8)
26. El conjunto solución de la ecuación
p
2x + 3
p
x 2 = 2 es
Solución :
Elevando al cuadrado ambos miembros de la ecuación
p
2x + 3
p
x 2
2
= 4
2
p
(2x + 3) (x 2) = 3 3x
nuevamente elevando al cuadrado
2
p
(2x + 3) (x 2)
2
= (3 3x)
2
8x2
4x 24 = 9x2
18x + 9
x2
14x + 33 = 0
al resolver dicha ecuación, se tiene que el conjunto solución es f3; 11g
27. Si j2x 1j > 3, el valor de x que no pertenece al conjunto solución es
Solución :
Por de…nición se obtiene
2x 1 > 3 o 2x 1 < 3
entonces
2x > 4 o 2x < 2
x > 2 o x < 1
por tanto, el conjunto solución es ( 1; 1) [ (2; 1) ; es decir x =2 [ 1; 2] ; lo cual es el valor x = 1
26
28. Si x +
1
x
2
= 3 entonces x3
+
1
x3
es igual a:
Solución :
Multiplicando por x +
1
x
a la expresión x +
1
x
2
= 3 se obtiene que :
x +
1
x
x +
1
x
2
= 3 x +
1
x
x +
1
x
3
= x3
+ 3x2 1
x
+ 3x
1
x2
+
1
x3
= 3x +
3
x
x3
+
1
x3
= 0
29. El conjunto solución de 3x + jxj = 8 es
Solución :
Aplicando la de…nición de valor absoluto, para x 0
3x + x = 8
x = 2
para x < 0
3x x = 8
x = 4
podemos notar que para x = 2 se tiene una solución extraña y por tanto, la única solución es x = 4
30. Al factorizar la expresión 12x3
+ 36x2
27x uno de los factores es:
Solución :
Factorizando la expresión dada
12x3
+ 36x2
27x = 3x (2x 3)
2
31. El resultado simpli…cado de
3y
2
4
p
8x3y7
1
3x
4
p
8x2y3, es:
Solución :
Al efectuar las operaciones indicadas, se tiene
y
2x
4
p
82x5y10 =
y
2x
p
8x 4
p
xy2 4
p
y2 = y3 4
p
4xy2
27
32. Si x; y; z, son números positivos que satisfacen x +
1
y
= 4 ; y +
1
z
= 1 ; z +
1
x
=
7
3
entonces el valor de xyz es:
Solución :
Reescribiendo las expresiones dadas
xy + 1 = 4y
xyz + z = 4yz
xyz = 4yz z
xz + 1 =
7
3
x
xyz + y =
7
3
xy
Igualando las expresiones y sabiendo que x = 4
1
y
; z =
7
3
1
x
entonces
4yz z = z 4
1
y
4
1
y
=
1
y
(4y 1) (z 1)
=
1
y
(4y 1)
0
B
B
@
7
3
1
4
1
y
1
1
C
C
A
=
1
3y
(13y 4)
pero
1
3y
(13y 4) = xyz
1
3y
(13y 4) = 4
1
y
y
0
B
B
@
7
3
1
4
1
y
1
C
C
A
y =
2
5
entonces
x = 4
1
y
=
3
2
z =
7
3
1
x
=
5
3
de aqui
xyz = 1
28
33. Si n > 1, entonces 3
q
n 3
p
n 3
p
n es igual a:
Solución :
Aplicando la de…nición de radicales
3
r
n
3
q
n 3
p
n =
3
q
n
3
p
nn1=3 =
3
q
n
3
p
n4=3 =
3
p
nn4=9 =
3
p
n13=9 = n13=27
34. La expresión
n2p
a:a33
:a53
::::a(2n 1)3
es igual a:
Solución :
Al utilizar la igualdad 13
+ 33
+ ::: + (2n 1)
3
= n2
2n2
1 ; y observando que la cantidad subradical es
un producto de potencia de la misma base y que dicha suma coincide con la dada en la sugerencia, podemos
sustituir y obtener
n2p
an2(2n2 1) = a2n2
1
35. Si (x + y)
2
= 2 x2
+ y2
el valor de E =
3x3
y3
x2y
+
3x + y
5x
+
6y
2x + y
será:
Solución:
Al efectuar la suma tenemos que
3x3
y3
x2y
+
3x + y
5x
+
6y
2x + y
=
30x4
+ 21x3
y + 35x2
y2
9xy3
5y4
5 (2xy y2) y (2x + y)
=
30x4
+ 21x3
y + 35x2
y2
9xy3
5y4
20x2y2 5y4
Por otro lado, como (x + y)
2
= 2 x2
+ y2
, resulta que x2
= 2xy y2
, así al sustituir obtenemos
3x3
y3
x2y
+
3x + y
5x
+
6y
2x + y
=
30x4
+ 21x3
y + 35x2
y2
9xy3
5y4
20x2y2 5y4
=
30 2xy y2 2
+ 21 2xy y2
xy + 35 2xy y2
y2
9xy3
5y4
20 (2xy y2) y2 5y4
=
162x2
y2
80xy3
10y4
20 (2xy y2) y2 5y4
=
162 2xy y2
y2
80xy3
10y4
20 (2xy y2) y2 5y4
=
4y3
(43y 61x)
5y3 (5y 8x)
=
172y 244x
25y 40x
29
36. Si el polinomio P (x) = x4
+ ax3
bx2
+ cx 1 es divisible por (x 1) (x + 1) (x 1) ; el valor de (a + b + c)
2
es:
Solución
El polinomio (x 1) (x + 1) (x 1) es igual a x3
x2
x+1. Al dividir P (x) entre x3
x2
x+1, el residuo es
(a b + 2) x2
+(a + c) x+( a 2) y el cociente x+(a + 1). Pero P (x) es divisible entre (x 1) (x + 1) (x 1)
así que el residuo debe ser cero, es decir, (a b + 2) x2
+(a + c) x+( a 2), y para que esto ocurra a b+2 = 0,
a + c = 0, a 2 = 0. De esto resulta que a = 2; c = 2 y b = 0. Por lo tanto,
(a + b + c)
2
= ( 2 + 0 + 2)
2
= 02
= 0
37. Sabiendo que x +
1
x
= 3;al determinar el valor de E = x3
+ x2
+
1
x3
+
1
x2
obtenemos:
Solución
Se tiene que
x +
1
x
2
= x2
+
1
x2
+ 2
por lo cual al sustituir x +
1
x
= 3, resulta que x2
+
1
x2
= 32
2 = 7: Similarmente, como
x +
1
x
3
= x3
+
1
x3
+ 3 x +
1
x
= x3
+
1
x3
+ 9
se obtiene que x3
+
1
x3
= 33
9 = 18. Por tanto, E = 7 + 18 = 25
38. Si el cociente notable
x30
ym
xn y2
tiene 10 términos, entonces el valor de (m + n) es:
Solución
En un cociente notable, para hallar el número de términos que va a tener la solución de la división, por ejemplo
de:
xp
yq
xr ys
se calcula como la división de los exponentes de la misma variable:
n =
p
r
=
q
s
Así pues en este caso
10 =
30
n
=
m
2
de lo cual se deduce que n = 3 y m = 20; luego m + n = 23
30
39. Si 264
= aa
y
p
3
54
= (3b)
b
; al determinar el valor de 3a + b se obtiene:
Solución
Tenemos que aa
= 264
= 24 16
= (16)
16
, así que a = 16. Por otro lado, (3b)
b
=
p
3
54
= (3)
27
= 33 9
=
(3 9)
9
; lo cual indica que b = 9. Luego, 3a + b = 48 + 9 = 57
40. Si (2a + b)
c
=
1
5
; entonces el valor de b2
+ 4ab + 4a2 c
es:
Solución
Se tiene que
b2
+ 4ab + 4a2 c
=
h
(2a + b)
2
ic
= [(2a + b)
c
]
2
Como el inverso multiplicativo de (2a + b)
c
es 1
5 , signi…ca que (2a + b)
c
= 5, resultando que b2
+ 4ab + 4a2 c
=
[(2a + b)
c
]
2
= 52
= 25
41. Sabiendo que a + b + c = 0; ab + ac + bc = 7 y abc = 6 entonces el valor de
1
a2
+
1
b2
+
1
c2
es:
Solución
Tenemos que
1
a
+
1
b
+
1
c
2
=
2
ab
+
2
ac
+
2
bc
+
1
a2
+
1
b2
+
1
c2
bc + ac + ab
abc
2
=
2 (c + b + a)
abc
+
1
a2
+
1
b2
+
1
c2
1
a2
+
1
b2
+
1
c2
=
bc + ac + ab
abc
2
2 (c + b + a)
abc
Ahora haciendo las debidas sustituciones, resulta
1
a2
+
1
b2
+
1
c2
=
7
6
2
2 (0)
6
=
49
36
42. Al simpli…car la expresión A =
x2
(x y) (x z)
y2
(y z) (y x)
+
z2
(z x) (z y)
el resultado es:
Solución
Efectuando las operaciones indicadas se obtiene
x2
(x y) (x z)
y2
(y z) (y x)
+
z2
(z x) (z y)
=
x2
(y z) y2
(z x) + z2
(x y)
(x y) (x z) (y z)
=
(x z) xy xz + yz + y2
(x y) (x z) (y z)
=
xy xz + yz + y2
(x y) (y z)
31
43. El conjunto solución de la ecuación
x2
6x + 10
x2 + 8x + 17
=
x 3
x + 4
2
, es:
Solución
Desarrollando el cuadrado en el miembro derecho de la igualdad, tenemos
x2
6x + 10
x2 + 8x + 17
=
x2
6x + 9
x2 + 8x + 16
de lo cual se obtiene
x2
6x + 10 x2
+ 8x + 16 = x2
6x + 9 x2
+ 8x + 17
x4
+ 2x3
22x2
16x + 160 = x4
+ 2x3
22x2
30x + 153
16x + 160 = 30x + 153
16x + 30x = 153 160
14x = 7
x =
7
14
x =
1
2
44. Un barril contiene 120 litros de alcohol y 180 litros de agua;un segundo barril contiene 90 litros de alcohol y
30 litros de agua¿ Cuántos litros debe tomarse de cada uno de los barriles para formar una mezcla homogénea
que contenga 70 litros de agua y 70 litros de alcohol.
Solución
El primer barril contiene una mezcla 300 litros, en la cual el
(120) (100)
300
= 40 porciento es alcohol y el
(180) (100)
300
= 60 porciento es agua. En el segundo barril hay una mezcla de 90 litros de alcohol y 30 litros de
agua, es decir el
(90) (100)
120
= 75 porciento es alcohol y el
(30) (100)
120
= 25 porciento es agua. Esto signi…ca que
cualquier cantidad que se tome del primer barril contiene un 60% de agua y un 40% de alcohol, así mismo al
tomar cualquier cantidad del segundo barril contiene un 25% de agua y un 75% de alcohol.
Sea x la cantidad de litros que se tomará del primer barril y y la cantidad que será tomada del segundo barril.
Como la mezcla debe contener 70 litros de agua y 70 litros de alcohol, entonces planteamos el siguiente sistema
de ecuaciónes (
0:4x + 0:75y = 70 Ec (1)
0:6x + 0:25y = 70 Ec (2)
Luego, si multiplicamos la primera ecuación por 0:25 y la segunda por 0:75, obtenemos
(
0:1x 0:1875y = 17:5 Ec (3)
0:45x + 0:1875y = 52:5 Ec (4)
Ahora sumando miembro a miembro las ecuaciones 3 y 4 resulta
0:35x = 35:
32
Despejando x se llega x = 100. Sustituyendo en la ecuación 1 o en la ecuación 2 se obtiene que el valor de y es
40. Por lo tanto, para formar una mezcla homogénea que contenga 70 litros de agua y 70 litros de alcohol hay
que tomar 100 litros del primer barril y 40 del segundo.
45. La hierba crece en todo el prado de la hacienda "el Meymo" con igual rapidez y espesura. Se sabe que 70 vacas
se la comerían en 24 días y 30 en 60 días ¿ Cuántas vacas se comerían toda la hierba en 96 días?
Solución
Sea p el prado y v la rapidez de crecimiento por día del pasto. Sabemos que
# vacas Cantidad de pasto consumida # de días
70 p + 24v 24
30 p + 60v 60
Luego tendríamos que
30
70
p + 24v
p + 60v
=
24
60
30
70
60
24
=
p + 60v
p + 24v
15
14
=
p + 60v
p + 24v
15 (p + 24v) = 14 (p + 60v)
15p + 360v = 14p + 840v
15p 14p = 840v 360v
p = 480v:
Ahora
# vacas Cantidad de pasto consumida # de días
70 p + 24v = 480v + 24v = 504v 24
x p + 96v = 480v + 96v = 576v 96
por lo cual
x
70
504v
576v
=
24
96
x
70
7
8
=
24
96
x =
24
96
8
7
70
x = 20
46. En un gallinero había cierto número de gallinas, se duplicó el número y se vendio 27 quedando menos de 54.
después se triplicó el número de gallinas que habia al principio y se vendió 78, quedando más de 39,¿Cuántas
gallinas habia al principio?
Solución
Sea x el número de gallinas que había al principio. La expresión: "se duplicó el número y se vendió 27 quedando
menos de 54" se representa en el lenguaje algebraico de la siguiente manera 2x 27 < 54, y el conjunto solución
33
de dicha desigualdad es ( 1; 40:5) : La otra expresión "Después se triplicó el número de gallinas que habia al
principio y se vendió 78, quedando más de 39" la representamos mediante la desigualdad 3x 78 > 39, cuyo
conjunto solución es (39; 1). Luego, como la intersección de los conjuntos soluciones de las dos desigualdades
es 40, resulta que inicialmente habían 40 gallinas.
47. Un grupo de abejas cuyo número era igual a la raíz cuadrada de la mitad de todo su enjambre se posó sobre
un jazmin, habiendo dejado muy atrás a
8
9
de su enjambre, sólo una abeja del mismo enjambre revoloteaba en
torno a una ‡or de sacuanjoche, atraida por el zumbido de una de sus amigas que cayó imprudentemente en la
trampa de dulce fragancia.¿cuántas abejas formaban el enjambre?
Solución
Sea x el número de abejas del enjambre. La información del problema nos proporciona la siguiente ecuación:
r
x
2
=
x
9
2
Elevando al cuadrado ambos miembros de la igualdad, tenemos
x
2
=
x2
81
4x
9
+ 4: Luego,
x2
81
17
18
x + 4 = 0:
Utilizando la fórmula general para resolver esta ecuación, se obtienen las soluciones 72 y
9
2
; pero como
9
2
no
es entero, nos quedamos 72, es decir que en el enjambre habían 72 abejas.
48. Si x4
y4
= z3
y x2
+ y2
= 8, entonces
z3
8
es igual a:
Solución :
Sustituyendo x4
y4
= z3
y x2
+ y2
= 8 en
z3
8
obtenemos
z3
8
=
x4
y4
x2 + y2
;
factorizando la diferencia de cuadrados
z3
8
=
x2
+ y2
x2
y2
x2 + y2
y simpli…cando
z3
8
= x2
y2
factorizando una vez más obtenemos
z3
8
= (x + y) (x y)
49. Al simpli…car
x 2=3
y 4=3
z 4
x 1=3y2=3z 7=3
3
resulta
Solución :
Al aplicar propiedades de exponentes
x 2=3
y 4=3
z 4
x 1=3y2=3z 7=3
3
=
x2
y4
z12
xy 2z7
= xy6
z5
34
50. Si 2x3
+ x2
+ px + 2p2
es divisible entre x + 1, siendo p un número real, entonces el valor de p es:
Solución :
Como el polinomio dado es divisible por x + 1; entonces P ( 1) = 0
P (x) = 2 1 p + 2p2
2p2
p 3 = 0
(p + 1) p
3
2
= 0
luego el polinomio es divisible por (p + 1)
51. El conjunto solución de la desigualdad
3
2x + 3
<
1
x 2
es
Solución :
Al reescribir la desigualdad dada
3
2x + 3
1
x 2
< 0
x 9
2x2 x 6
< 0
x 9
(2x + 3) (x 2)
< 0
de aqui, podemos observar que los puntos criticos son x =
3
2
; x = 2 ; x = 9; entonces
Expresión x 9 2x + 3 x 2 Signo
x <
3
2
3
2
< x < 2 + +
2 < x < 9 + +
x > 9 + + + +
por tanto, el conjunto solución está de…nido por los intervalos donde está el signo negativo, es decir
1;
3
2
[ (2; 9)
52. Dos enteros a > 1 y b > 1 satisfacen ab
+ ba
= 57. Determinar la suma a + b
Solución :
Uno de los enteros, digamos a, debe ser par, mientras que el otro, b, debe ser impar. Como 43
= 64 > 57,
tenemos que a = 2; entonces b = 5. Entonces dicha suma debe ser 7
35
53. Si x + y = 1 y xy = 1 , ¿cuál será el valor de x3
+ y3
?
Solución :
Elevando al cubo la expresión (x + y) = 1, y aplicando las condiciones dadas en el ejercicio, se obtiene
(x + y)
3
= x3
+ 3x2
y + 3xy2
+ y3
= 1
x3
+ 3xy (x + y) + y3
= 1
x3
+ 3 (1) (1) + y3
= 1
x3
+ y3
= 2
54. El polinomio p(x) = x3
x2
+ x 1 se anula en 1, luego p(x) es divisible por:
Solución :
Decir que P (x) se anula en 1 signi…ca que
P (1) = 13
12
+ 1 1 = 0
entonces (x 1) es un divisor de este polinomio.
55. La suma de dos números es 666 y si se divide el mayor entre el menor el cociente es 5 y el residuo 78. Dichos
números son:
Solución :
Sean x el número mayor, y el número menor, entonces
x + y = 666
x = 5y + 78
cuya solución es fx = 568; y = 98g
56. Si suponemos que el cociente intelectual de Einstein era 170 y si éste se calcula al dividir la edad mental por
la edad cronológica multiplicado por 100, la edad mental de Einstein cuando publicó en 1905 su teoría sobre el
efecto fotoeléctrico era:
Solución :
El coe…ciente intelectual (CI), se de…ne por
CI =
EM
EC
100
donde EM es la Edad Mental y EC es la Edad Cronologica, de aqui
170 =
EM
26
100
cuya solución es 44:2
36
57. Mi hijo es ahora tres veces más joven que yo, pero hace cinco años era cuatro veces más joven. ¿Cuántos años
tiene el hijo?
Solución :
Sea P la edad actual del padre y H la edad actual del hijo, entonces
3H = P
4 (H 5) = P 5
cuya solución es H = 15; P = 45
58. Un grupo de amigos fue a tomar unos refrescos y unas empanadas, y lo pusieron todo en una cuenta que
ascendió a 36 córdobas. Todos iban a pagar por igual, pero tres de ellos se habían ido, por lo que a cada uno
le tocó pagar 1 córdobas más. ¿Cuántas personas conformaban el grupo original?
Solución :
Llamemos x al número de amigo al principio e y al costo si hubieran estado todos, entonces
xy = 36
(x 3) (y + 1) = 36
cuya solución es x = 12; y = 3
59. Un hombre entró en la cárcel para cumplir una condena. Para que su castigo fuera más duro no le dijeron
cuanto tiempo tendría que estar allí dentro. Pero el carcelero era un tipo muy decente y el preso le había caído
bien.
Preso: ¡Vamos!. ¿puedes darme una pequeña pista sobre el tiempo que tendré que estar en este lugar?
Carcelero: ¿Cuántos años tienes?
Preso: Veinticinco.
Carcelero: Yo tengo cincuenta y cuatro. Dime, ¿qué día naciste?
Preso: Hoy es mi cumpleaños.
Carcelero: Increíble. ¡También es el mío!. Bueno, por si te sirve de ayuda te diré (no es que deba, pero lo
haré) que el día que yo sea exactamente el doble de viejo que tú, ese día saldrás. ¿Cuánto tiempo dura la
condena del preso?
Solución :
Sea x tiempo de condena del preso, entonces
2 25 + x = 54
x = 4
37
60. El producto de tres enteros positivos consecutivos es 3360 y su suma es 45. ¿Cuál es el mayor de esos tres
números?
Solución :
Este problema se puede resolver utilizando la segunda condición
x + (x + 1) + (x + 2) = 45
x = 14
como los números son consecutivos, entonces el mayor es x + 2 = 16
61. Un autobús comienza su trayecto con un cierto número de pasajeros. En la primera parada descienden 1=3 de
los pasajeros y suben 8. En la segunda parada descienden 1=2 de los pasajeros que quedan y suben 2 nuevos.
En este momento, el autobús lleva la mitad del número de pasajeros de los que llevaba al principio del trayecto.
¿Cuántos pasajeros había al principio?
Solución :
Llamamos x al número de pasajeros que había al comienzo del viaje. En la primera parada desciende
1
3
de los
pasajeros. Luego se quedan
2
3
x. Suben 8. Por tanto, después de la primera parada en el autobús hay
2
3
x + 8
pasajeros. En la segunda pasada descienden
1
2
de los pasajeros, luego se queda
2
3
x + 8
2
. Suben otros 2. Por
tanto, después de la segunda parada en el autobús hay
2
3
x + 8
2
+2: En ese momento el número de pasajeros
es la mitad de los que había al principio, es decir,
x
2
: Igualamos y obtenemos
2
3
x + 8
2
+ 2 =
x
2
cuya solución es x = 36
62. Hallar tres números sabiendo que el segundo es mayor que el primero en la misma cantidad que el tercero es
mayor que el segundo, que el producto de los dos menores es 85 y que el producto de los dos mayores es 115.
Solución :
Sean x; y; z los tres números buscados, además x < y < z; entonces
8
>><
>>:
y x = z y
xy = 85
yz = 115
cuya solución es z =
23
2
; y = 10; x =
17
2 38
63. Daniel y Arturo, dos viejos amigos, vuelven a encontrarse en la calle al cabo de algunos años. Después de
saludarse,
Daniel : ¿Cuántos hijos tienes?
Arturo : Tres hijos.
Daniel : ¿Qué edades tienen?
Arturo : Tú mismo lo vas a averiguar. El producto de sus edades es 36. Daniel, después de pensar durante
algún tiempo, le dice a Arturo que necesita más datos.
Arturo : En efecto, la suma de sus edades es igual al número de la casa que tenemos enfrente. Daniel mira el
número de la casa que le indica Arturo y quedándose pensativo durante un par de minutos. - ¡No es posible! -
responde, con lo que me has dicho no puedo conocer las edades de tus hijos. Me falta un dato más.
Arturo : Perdona Daniel, olvidé decirte que mi hija la mayor toca el piano.
Daniel: En ese caso, ya sé sus edades. ¿Qué edades tienen los hijos de Arturo?
Solución :
El problema se reduce a encontrar tres números naturales cuyos producto sea 36, sean x; y; z dichos números,
entonces
xyz = 36
entonces se forman las siguientes posibilidades, cada una con sus respectivas sumas
1 1 36 = 38
2 2 9 = 13
1 4 9 = 14
1 6 6 = 13
1 2 18 = 21
Observemos que el dato del número de la casa es la clave, ya que el número 13 se repite dos veces ( el cual
constituye el número de la casa ), por tanto, de las posibilidades 2 2 9 = 13 y 1 6 6 = 13; la correcta
es la primera, ya que para el segundo caso, no existe una única hija mayor.
64. Un ciclista calcula que si avanza a 10 km=hora llegará a su destino a la 1p:m., y si avanza a 15 km=hora llegará
a su destino a las 11a:m. ¿a qué velocidad, en km=hora, tiene que avanzar para llegar a las 12m.?
Solución :
Sabemos que d = vt aplicando las condiciones del problema
d = 10t
d = 15 (t 2)
igualando
10t = 15t 30
t = 6
39
ahora
10t = v (t 1)
60 = 5v
v = 12
65. Un camino puede recorrerse en “t”horas con una cierta velocidad en km=hr. El mismo camino se puede hacer
en una hora menos aumentando en un kilómetro por hora la velocidad. Hallar la longitud del camino en km.
Solución :
Sabemos que d = vt además la velocidad se aumenta en 1; disminuyendo el tiempo en 1; es decir
d = (v + 1) (t 1)
d = vt v + t 1
entonces
vt = vt v + t 1
v = t 1
por tanto
d = (t 1) t
d = t2
t
66. De un depósito de 100 litros de capacidad, lleno de alcohol puro, se saca una cierta cantidad de alcohol y se
le reemplaza por agua. Se saca después la misma cantidad de mezcla y se reemplaza por agua, quedando ésta
última mezcla con un 49% de alcohol. Determinar la cantidad de líquido que se ha sacado cada vez.
Solución :
La cantidad que se saca y se reemplaza es la misma, entonces la proporción de lo que se extrae es
x
100
; entonces
en la primera extracción, se tiene
Seleccion Alcohol Agua
Inicio 100 0
1 100 x x
2 (100 x)
x
100
(100 x) x
x
100
por tanto
(100 x)
x
100
(100 x) = 49
y vemos que al resolver para x se obtiene el valor de x = 30: Note que este problema tiene una raíz rara, extraña
o falsa y ocurre para x = 170
40
67. La suma de tres números es 21. El cociente de dos de ellos es 2:5 y la suma de estos dividida entre el tercero
da como cociente 2. ¿Cuál es el menor de los tres números?
Solución :
Sean x; y; z los tres números, planteando el sistema de ecuaciones
8
>>><
>>>:
x + y + z = 21
x
y
= 2:5
x + y
z
= 2
8
>><
>>:
x + y + z = 21
x 2:5y = 0
x + y 2z = 0
al resolver dicho sistema, se tiene que
x = 10:0; y = 4:0; z = 7:0
entonces el número menor corresponde a y = 4
68. Un padre actualmente tiene el triple de la edad de su hijo; si hace 6 años la edad del padre era el quíntuple de
la edad de su hijo. Señale la suma de cifras de edad del padre.
Solución :
Sean x; y las edades respectivas del padre e hijo respectivamente, entonces
(
x = 3y
x 6 = 5 (y 6)
al resolver dicho sistema de ecuación, se tiene x = 36; y = 12; entonces la suma de las cifras de la edad del
padre es 9
69. Dos tuberías abiertas simultáneamente llenan un depósito en 1 hora 12 minutos. Si una de ellas tarda 1 hora
más que la otra, en llenar el mismo depósito ¿en qué tiempo lo llenará la tubería de mayor caudal?
Solución :
Sean x : tiempo de la tubería de menor caudal
y : tiempo de la tubería de mayor caudal
planteando el sistema de ecuaciones lineales en formato de minutos
8
<
:
1
x
+
1
y
=
1
72
x = y + 60
resolviendo para y; se tiene que y = 120 min, lo cual equivale a y = 2 horas
41
70. Un albañil y su ayudante pueden hacer una obra en 24 días. Después de 4 días de trabajo, el ayudante se retira
y el albañil termina lo que falta del trabajo en 30 días. ¿En cuántos días podría hacer el trabajo el ayudante
trabajando solo?
Solución :
Al plantear una regla de tres compuesta
Hombres Dias Proyectado Dias Reales
2 24 20
1 x 30
se obtiene
x =
2 24 30
1 20
= 72
71. En Navidad, en cierta empresa todos los empleados se ofrecen regalos. En esta ocasión las mujeres se han dado
mutuamente un regalo, pero los hombres lo han repartido: la mitad han dado un regalo a sus compañeros y la
otra mitad lo han ofrecido a cada una de sus compañeras. Sabemos que el doble del número de mujeres excede
en 6 al número de hombres. Si en total se han dado 318 regalos, ¿cuántos empleados tiene la empresa?
Solución :
Sean x número de hombres, y número de mujeres, sabemos que x = 2y 6, y (y 1) cantidad de regalos de
las mujeres, porque cada mujer da un regalo a otra mujer, también
x
2
(x 1) +
x
2
y porque la mitad de los
hombres da un regalo a otro hombre y la otra mitad a las mujeres, de aqui, obtenemos que
x = 2y 6
x
2
(x 1) +
x
2
y + y (y 1) = 318
cuya solución es y = 11; x = 16
72. Determinar un entero positivo con los datos siguientes: si se añade un 5 a la derecha el número resultante es
divisible exactamente por un número que sobrepasa en 3 el buscado, siendo el cociente igual al divisor menos
16.
Solución :
Llamemos N al número buscado. El número divisible será (10N + 5), el divisor será (N + 3) y el cociente será
(N + 3) 16 = (N 13). Entonces
(10N + 5) = (N + 3)(N 13)
N2
20N 44 = 0
cuya solución es N = 2; N = 22; por tanto, la solución positiva es el resultado.
42
73. Hallar un número de dos cifras sabiendo que el número de unidades excede en dos el número de decenas y que
el producto del número deseado por la suma de sus dígitos es 144.
Solución :
El número deseado N, cumplirá N = 10d + u; u = d + 2; entonces
(10d + u)(d + u) = 144
(10d + d + 2) (d + d + 2) = 144
22d2
+ 26d 140 = 0
cuya solución es d = 2; de donde u = 4 y N = 24
74. Si n es un entero positivo, la igualdad m4
km2
n + n2 n
= m2
n
2n
se cumple si k toma el valor:
Solución :
Si k = 2, entonces La expresión
m4
km2
n + n2 n
= m4
2m2
n + n2 n
=
h
m2
n
2
in
también
m2
n
2n
=
h
m2
n
2
in
75. Un factor de 5t 12 + 2t2
es t + 4 y el otro es:
Solución :
Al factorizar el polinomio dado
5t 12 + 2t2
= (t + 4) (2t 3)
el otro factor es 2t 3
76. Si el producto de los monomios x2n
yn
y xm
y es igual a x 2
y3
, entonces los valores de m y n son respectivamente:
Solución :
Como x2n
yn
(xm
y) = x 2
y3
entonces
2n + m = 2
n + 1 = 3
de donde se obtiene de forma inmediata que n = 2; m = 6
43
77. Supongamos que x1 y x2 son las raíces de la ecuación
ax2
+ bx + c = 0; (a 6= 0)
la expresión
1
x2
1
+
1
x2
2
expresada en función de las raíces, es igual a:
Solución :
La solución de una ecuación cuadrática tiene la forma
b
p
b2 4ac
2a
entonces se pretende calcular
1
x2
1
+
1
x2
2
=
x2
2 + x2
1
x2
1x2
2
=
b
p
b2 4ac
2a
!2
+
b +
p
b2 4ac
2a
!2
"
b +
p
b2 4ac
2a
!
b
p
b2 4ac
2a
!#2
=
b2
2ac
c2
78. La raíz quinta de la raíz cuarta de la raíz cuadrada de la raíz cuadrada de (a2
+ b2
) es igual a:
Solución :
Simbolizando las raices respectivas y multiplicando cada uno de sus indices, obtenemos que
5
s
4
rqp
(a2 + b2) = 80
p
(a2 + b2) = a2
+ b2 1=80
79. El sistema (
kx + y = 1
x + ky = 2
tiene solución única si:
Solución :
Un sistema de ecuación tiene solución unica si y solo si el determinante del sistema es distinto de cero, para
este ejercicio tenemos
k 1
1 k
= k2
1 6= 0
de aqui que, k 6= 1; k 6= 1
44
80. La suma de las cuatro raíces de las ecuaciones ax2
+ bx + c = 0 y ax2
bx + c = 0; con a 6= 0 y b2
4ac > 0
es igual a:
Solución :
Las raices de las ecuaciones ax2
+ bx + c = 0 y ax2
bx + c = 0 son respectivamente
b
2a
y
b
2a
las que al
sumarse se obtiene
b
2a
+
b
2a
= 0
45
UNIDAD DE GEOMETRÍA EUCLIDIANA
1. En la …gura, el ]COB = 120o
y el ]COD mide la mitad del ángulo BOA. Entonces, la medida del ]BOA es:
Solución:
Sea mBOA = x, luego mCOD = x
2 . Se tiene
x
2
+ 120o
+ x = 180o
3x
2
= 60o
x = 40o
2. Si dos planos diferentes se intersecan, su intersección es:
Solución:
Por uno de los axiomas de la Geometría Euclidiana si dos planos diferentes se intersecan, su intersección es una
única recta.
3. . En la …gura, !m1 ? !m4, !m2 ? !m3¿cuál de las siguientes expresiones es siempre verdadera?
Solución:
Con la información dada las parejas de rectas perpendiculares están “libres”, luego pueden ser giradas y
46
seguirían satisfaciendo los datos dados. Por tanto no puede a…rmarse ni A, ni B, ni C, ni D.
1m
↔
2m
↔
3m
↔
4m
↔
4. R; S y T son tres puntos colineales como se muestran en la …gura. Si ST = 4x + 4 y RS es la mitad de ST,
entonces la longitud de RT es:
Solución:
Dado que los puntos son colineales, se tiene
RT = RS + ST
=
1
2
ST + ST
=
3
2
ST
=
3
2
(4x + 4)
= 6x + 6
5. A partir de la información indicada en la …gura, el valor de Y es:
Solución:
Sean A, B y C los puntos indicados en la …gura, y sean mBAC = , mACB = . Se tiene = 50o
, por ser
opuesto por el vértice con el ángulo que mide 50o
y = 180o
130o
= 50o
. El ángulo que mide yo
es un ángulo
exterior con respecto al 4ABC, luego su medida equivale a la suma de los ángulos internos no adyacentes, es
decir y = + = 50o
+ 50o
= 100o
47
6. En la …gura, si AB k CD, el valor de X es:
Solución:
Dado que las rectas son paralelas, xo
= mFCE, por ser ángulos correspondientes. A su vez este ángulo por
ser externo al 4ECD, es la suma de las medidas de los ángulos CED y EDC.Se tiene mCED = 90o
y
mEDC = 180o
140o
= 40o
, luego x = 90o
+ 40o
= 130o
:
7. A partir de la información brindada en la …gura, el valor de Z resulta:
Solución:
Las marcas en el ángulo A, indican que AD es bisectriz de dicho ángulo, luego x = 40o
y mA = 80o
. Al
considerar el 4ABC, se tiene z = 180o
80o
70o
= 30o
.
8. En la …gura, AD ? AC; EB k DC,entonces el valor de Y es:
Solución:
Dado que EBkDC, se tiene x = 180o
130o
= 50o
por ser ángulos internos al mismo lado, entre paralelas y como
AD?AC el 4ADC es triangulo rectángulo y por tanto “y”es el complemento de “x”, luego y = 90o
50o
= 40o
.
48
9. En la …gura el valor de X es
Solución:
Se tiene mABC = 180o
140o
= 40o
, x = 115o
40o
= 75o
ya que el ACD es externo al 4ABC.
10. En la …gura el valor de X es:
Solución:
Se tiene mEDB = 180o
150o
= 30o
, ABC = DBE por ser opuestos por el vértice y por el teorema de
semejanza AA, 4ABC 4DBE, luego mBAC = xo
= mEDB = 30o
.
11. A B C D; E y F son puntos medios de AB y CD respectivamente; Si AC = 10 y BD = 12, entonces
EF =?
Solución:
Sean AE = EB = x, CF = FD = y (E y F son puntos medios de AB y CD respectivamente).Se tiene:
BC = AC AB = 10 2x (1)
y también
BC = BD CD = 12 2y (2)
49
Igualando (1) y (2): 10 2x = 12 2y: Al simpli…car se obtiene:
y x = 1 (3)
Por otro lado se tiene
EF = EB + BC + CF = x + (10 2x) + y = 10 x + y = 10 + (y x)
Al introducir (3) resulta
EF = 10 + 1 = 11
12. En la …gura o
+ o
= 255o
, entonces ¿mA =?
Solución:
En el 4ABC, tenemos que
mA = 180o
mABC mACB = 180o
(mABC mACB) (1)
Se tiene que mABC = 180o
y mACB = 180o
, luego mABC + mACB = 360o
( + ); Como
+ = 255, resulta mABC + mACB = 360o
255o
= 105o
;
Sustituyendo en (1) obtenemos mA = 180o
105o
= 75o
13. ¿Para qué valor de x, los segmentos ABy CD son paralelos?
Solución:
Como el ángulo a la izquierda de C es congruente con el ángulo a la derecha, también mide 25o
. Luego
mACD = 180o
2 (25o
) = 130o
.
Como el 4APC, es recto en P, mPAC = 90o
25o
= 65o
.
50
Para que AB y CD sean paralelos, el ángulo CAB debe ser el suplemento del ángulo ACD ya que serían
ángulos internos al mismo lado entre paralelas o sea mCAB = 180o
130o
= 50o
.
Se tiene entonces
x + 50 + 65 = 180
x = 180 50 65
x = 65
14. Si AB k CD, ¿cuál es el valor de X?
Solución:
Trazamos EF, paralela a las rectasAB y CD, luego mAEF = 180o
120o
= 60o
y mFEC = 180o
xo
.
Además se tiene mAEF + mFEC = mAEC = 90o
, luego 60o
+ (180o
xo
) = 90o
. Al despejar x, resulta
xo
= 150o
.
15. Si la medida de un] es tres veces la medida de su suplemento, ¿cuál es la medida de dicho ]?
Solución:
Sean la medida del ángulo buscado y la medida de su suplemento, luego + = 180o
) = 180o
.
El ejercicio indica que = 3 , luego
= 3(180o
)
= 540o
3
4 = 540o
= 135o
16. . Dos veces la medida de un ] es 30 menos que cinco veces la medida de su complemento, ¿cuál es la medida
de dicho ángulo?
Solución:
Sean la medida del ángulo buscado y la medida de su complemento, luego
+ = 90o
= 90o
51
Al interpretar la información del ejercicio se tiene
2 = 5 30o
2 = 5 (90o
) 30o
2 = 450o
5 30o
7 = 420o
= 60o
17. En la …gura las rectas !m1 y !m2 son paralelas. Entonces el valor de x es:
Solución:
Sean A, B, C y D los puntos indicados en la …gura. Al trazar por B una paralela a !m1 y !m2 , se forman ángulos
alternos –internos entre paralelas, y por tanto congruentes con los ángulos indicados inicialmente, luego
x + 60 = 110
x = 110 60
x = 50
Otra Forma:
Al prolongar CB, sea D el punto donde corta a la recta !m1. Se tiene mADB = xo
, por ser alterno –interno
con el ángulo que se forma en C. El ángulo ABC es externo al 4ABD, luego 60 + x = 110 ) x = 50.
18. En la …gura las rectas !m1 y !m2 son paralelas. Entonces el valor de x es:
52
Solución:
Como las rectas son paralelas se tiene:
(3x + 10) + (x 6) = 84
4x + 4 = 84
4x = 80
x = 20
19. Si mP = 90o
; 1 = 2; 3 = 4, entonces mR es
Solución:
Sean y las medidas de los ángulos indicados en la …gura. Se tiene + = 90o
. Como SQR = 2 y
1 = 2, se tiene
2 mSQR = 180o
(1)
Similarmente se obtiene que
2 mQSR = 180o
(2)
Al sumar (1) y (2) resulta
2 mSQR + 2 mQSR = (180o
) + (180o
)
= 360o
( + )
Como + = 90o
,
2(mSQR + mQSR) = 360o
90o
2(mSQR + mQSR) = 270o
mSQR + mQSR = 135o
Luego mR = 180o
(mSQR + mQSR) = 180o
135o
= 45o
20. En una recta se toman los puntos A; B y C, de manera que B es punto medio de . Se toma otro punto O, tal
que B O C. Encuentre el valor numérico de:
AO OC
OB
Solución:
53
Se tiene AB = BC = x, por ser B punto medio de AC. Sea OB = y, luego OC = x y, AO = x + y. Al
sustituir estos valores en la expresión dada se tiene: AO OC = (x y) (x + y) = 2y, luego
AO OC
OB
=
2y
y
= 2
Nota: en ejercicios de este tipo no se admite asignar valores arbitrarios, ya que se estaría resolviendo para
valores especí…cos. El planteamiento es general. Cuando se a…rma que B O C, está indicando que O es un
punto cualquiera que se encuentra entre B y C, y el valor numérico encontrado es valido para cualquier punto
O que esté entre B y C.
21. Un poste cercano a un árbol mide 2m y su sombra en un momento dado mide 1:8m, entonces si la sombra del
árbol en ese momento mide 11m, la altura del árbol es:
Solución:
Dado que los rayos del sol prácticamente caen paralelos y que el poste y el tronco del árbol son perpendiculares
al piso, el árbol y su sombra y la línea que une sus extremos forman un triángulo semejante al formado por el
poste su sombra y la línea que une sus extremos, tenemos
h
11
=
2
1:8
h =
11:2
1:8
h = 12:22
22. Una varilla clavada en el piso y cercana a un árbol mide 3m y su sombra mide 1:5m, entonces si el árbol mide
36m, su sombra mide.
Solución:
El problema es similar al anterior, en este caso se tiene
x
36
=
1:5
3
x =
36 1:5
3
x = 18
23. El perímetro de un triángulo rectángulo isósceles con hipotenusa igual a 10 redondeado a dos decimales es
Solución:
En un triángulo rectángulo isósceles, la hipotenusa mide
p
2x, siendo x la longitud de sus catetos, luego
p
2x = 10
x =
10
p
2
x = 5
p
2
54
Su perímetro será
P = 10 + 2 5
p
2
= 10 + 10
p
2
= 24:14
24. En el triángulo rectángulo de la …gura, los valores de x y y, respectivamente son
Solución:
Por el teorema de la altura se tiene
4x = 82
= 64
x = 16
y la hipotenusa mide 4 + x = 20. Por el teorema de los catetos se tiene
y2
= 4 20 = 80
y =
p
80
y = 4
p
5
y 8:94
25. Un método para encontrar la altura de un edi…cio es colocar un espejo en el suelo y después situarse de manera
que la parte más alta del edi…cio pueda verse en el espejo ¿qué altura tiene un edi…cio si una persona cuyos
ojos están a 1:5m del piso observa la parte superior del edi…cio cuando el espejo está a 120 m del edi…cio y la
persona está a6m del espejo?
Solución:
Dado que las leyes de la óptica indican que en un espejo plano, el ángulo de incidencia es igual al ángulo de
re‡exión, se forman dos triángulos rectángulos semejantes, luego
h
120
=
1:5
6
h = 30 m
26. La altura respecto a la hipotenusa de un triángulo rectángulo mide 10m y los segmentos que determina sobre
la hipotenusa son entre sí como 7 es a 14. Entonces la longitud del cateto menor es
55
Solución:
Sean m y n los segmentos determinados por la altura sobre la hipotenusa, con m < n, luego
m
n
=
7
14
n = 2m
Por el teorema de la altura
m n = 102
m 2m = 100
m2
= 50
m = 5
p
2
n = 10
p
2
La hipotenusa mide c = m + n = 15
p
2
Por el teorema de los catetos
a2
= m (m + n)
a2
= 5
p
2 15
p
2
a2
= 150
a =
p
150 = 5
p
6
a 12:25
27. El perímetro de un rectángulo es 85m y su diagonal mide M. Por lo tanto los lados del rectángulo miden:
Solución:
Sean a y b los lados del rectángulo. Se tiene
P = 2 (a + b) = 85
a + b = 42:5 (1)
Además
a2
+ b2
= 32:52
= 1056:25 (2)
Despejando b de (1), e introduciendo en (2)
a2
+ (42:5 a)
2
= 1056:25
2a2
85a + 750 = 0
a = 12:5 _ a = 30
56
Al sustituir en (1) se obtiene b = 30 _ b = 12:5
28. El perímetro de un triángulo mide50 y sus lados son proporcionales a 4; 6 y 8. Entonces su lado mayor mide.
Solución:
Sean a, b y c las longitudes de los lados, con a < b < c, luego P = a + b + c = 50 y
a
4
=
b
6
=
c
8
Por las propiedades de las proporciones
a
4
=
b
6
=
c
8
=
a + b + c
4 + 6 + 8
=
50
18
c =
8 50
18
=
200
9
29. En un triángulo rectángulo, un lado mide 2
p
106, otro 5
p
15. Si el lado desconocido es el menor, ¿cuánto
mide?
Solución:
Como 2
p
106 > 5
p
15, la hipotenusa de este triángulo es 2
p
106, luego el cateto menor es
a =
r
2
p
106
2
5
p
15
2
=
p
424 375 =
p
49 = 7
30. El área del triángulo de la …gura, redondeada al entero más cercano, mide:
Solución:
Aplicamos la fórmula de Herón: A =
p
s (s a) (s b) (s c), donde s es el semiperímetro. Se tiene s =
6 + 7 + 9
2
= 11, luego
A =
p
11 (11 6) (11 7) (11 9) =
p
11 5 4 2 =
p
440 20:97
31. ¿Cuál es el área del triángulo de la …gura?
Solución:
57
Dado que es un triángulo rectángulo su área es la mitad del producto de sus catetos. El cateto desconocido
mide
b =
p
102 62 =
p
100 36 =
p
64 = 8
Por tanto A = 1
2 (6) (8) = 24
32. Si un rectángulo de 3mde ancho y 10mde largo tiene la misma área que un triángulo rectángulo isósceles,
entonces la longitud de cada cateto del triángulo es
Solución:
El área de un triángulo rectángulo isósceles está dada por A = 1
2 x2
, donde x es la longitud de sus catetos, luego
tenemos que el área del rectángulo es 30, por tanto
1
2
x2
= 30
x =
p
60
x = 2
p
15
33. El área de un trapecio isósceles de bases 22m y 10m y cuyos lados congruentes miden 10 es
Solución:
Por ser un trapecio isósceles, al proyectar la base menor sobre la base mayor, la base mayor queda dividida en
tres segmentos de 6, 10 y 6 metros. Aplicando el teorema de Pitágoras, se tiene
h =
p
102 62 =
p
64 = 8
Aplicando la fórmula para el área de un trapecio: A = (B+b) h
2 resulta
A =
(22 + 10) 8
2
= 128 m2
34. La siguiente …gura consta de siete cuadrados congruentes. El área total de esta …gura es 63cm2
. Entonces el
perímetro de la …gura es:
58
Solución:
Observamos que el perímetro está formado por 16 veces el lado de cada cuadrado. Como hay siete cuadrados
congruentes, cada uno tiene un área de
x2
=
63
7
= 9
x = 3
Por tanto el perímetro de la …gura es P = 16 3 = 48cm.
35. Si ACEG es un cuadrado y el área del cuadrilátero BDFH mide 162 ¿cuánto mide AC? (las marcas iguales
representan partes congruentes).
Solución:
La …gura indica que B, D, F y H son puntos medios de los lados del cuadrado ACEG, luego su área es el
doble del área del cuadrado BDFH, es decir [ACEG] = 2 162 = 324 luego AC =
p
324 = 18
36. Se tiene un trapecio ABCD donde es la base menor. BC = 10cm y CD = 20cm. Las medidas de los ángulos
A; B y C son 30 ; 150 y 120 respectivamente, entonces AD =?
Solución:
Sean B0 y C0 las proyecciones de B y C sobre la base mayor y sean AB0 = x, C0D = y. Por ser BC paralela a
AD,
mD = 180 mC = 180o
120o
= 60o
El 4CC0D es un triángulo 30 –60, luego h = CC0 = 10
p
3 y y = C0D = 10. También el 4AB0B resulta ser
un triángulo 30 –60, con su cateto menor BB0 = h = 10
p
3, luego AB0 = 10
p
3
p
3 = 30.
Tenemos entonces que la base mayor mide
AD = x + 10 + y = 30 + 10 + 10 = 50
59
37. Si las medianas en un triángulo rectángulo, trazadas a partir de los vértices de los ángulos agudos miden 5cm
y
p
40cm, entonces la medida de la hipotenusa del triángulo rectángulo es.
Solución:
Sean M y N los puntos medios de BC y AB respectivamente. Sean AM = 5 y CN =
p
40, BC = a, AB = c,
luego BM =
a
2
y NB =
c
2
.
Sea la hipotenusa AC = b: Aplicando el teorema de Pitágoras en los 4ABM y 4BCN
AM2
= AB2
+ BM2
= c2
+
a2
4
= 25 (1)
CN2
= NB2
+ BC2
= a2
+
c2
4
= 40 (2)
Al sumar (1) y (2) resulta
5c2
4
+
5a2
4
= 65
a2
+ c2
=
4
5
(65) = 52 = b2
b =
p
52 = 2
p
13
38. En la …gura, los cuadrados ABCD y EFGH son congruentes. AB = 10cm y G es el centro del cuadrado
ABCD. Entonces el área total cubierta por el polígono AHEFBCDA es.
Solución:
Dado que los cuadrados son congruentes sus áreas son iguales y como el lado AB mide 10, cada uno tiene un
área de 100cm2
, pero ellos comparten el 4ABG de manera que para el área total del polígono a la suma de
las áreas de los cuadrados debemos restarle el área de este triángulo para que sea considerada solo una vez.
Dado que G es el centro del cuadrado ABCD, el área del triángulo es la cuarta parte del área del cuadrado o
sea 25cm2
. Luego el área buscada es
A = [ABCD] + [EFGH] [ABG] = 100 + 100 25 = 175cm2
60
39. ABCD es un cuadrado, el 4ABE es isósceles, CF = FB. Entonces, la medida del ángulo EFB es igual a.
Solución:
Como el 4ABE es isósceles, AE = BE y por ser ABCD un cuadrado, E es el punto medio de DC y por
tanto EC = CF, ya que por ser CF = FB, F es punto medio de BC. Luego el 4ECF resulta ser triangulo
rectángulo isósceles y como consecuencia mCFE = 45o
. El ángulo buscado es el suplemento del CFE,
luego
mEFB = 180o
mCFE = 180o
45o
= 135o
40. En la …gura, ABCF es un paralelogramo. B; C y D son colineales. Si AB = 18; AD = 30 y FE = 12.
¿Cuánto mide AE?
Solución:
Se tiene que CF = AB = 18, ya que ABCF es un paralelogramo. CE = CF FE = 18 12 = 6.
Por otro lado BD y AF son paralelas, luego FAE = CDE, ya que son alternos internos entre paralelas y
FEA = CED, ya que son opuestos por el vértice. Como consecuencia se tiene 4FEA 4CED.
Sea AE = x, luego ED = AD AE = 30 x. Por la semejanza anterior,
CE
FE
=
ED
EA
6
12
=
30 x
x
6x = 360 12x
18x = 360
x = 20
41. En un trapecio isósceles, la diferencia de las bases es de 10m. La altura mide 12m. y el perímetro 76m.
Entonces su área es:
61
Solución:
Como B b = 10, al proyectar la base menor sobre la base mayor se forman tres segmentos de longitudes 5, b
y 5 como se muestra en la …gura. Luego como la altura es 12, en los extremos del trapecio se forman triángulos
rectángulos de catetos 5 y 12. Aplicando el teorema de Pitágoras hallamos que la hipotenusa mide 13 lo cual
corresponde a la longitud de los lados no paralelos del trapecio. Considerando que el perímetro mide 76 m. se
tiene:
2b + 2(13) + 2(5) = 76
b = 20
Al considerar que B b = 10, resulta B = 30. Aplicando la fórmula para el área de un trapecio, el área buscada
resulta
A =
(B + b) h
2
=
(20 + 30) 12
2
= 300 m2
42. En la …gura ABCD es un cuadrado de lado 1cm y CE = 2cm, entonces el área del triángulo ADF en cm2
es
igual a
Solución:
Dado que ABCD es un cuadrado AD y CE son paralelas, resultando que 4ADF 4ECF por el teorema de
semejanza AA, ya que DAF = CEF por ser alternos internos entre paralelas y DFA = CFE por ser
opuestos por el vértice.
De la semejanza resulta que
AD
CE
=
DF
CF
1
2
=
DF
CF
CF = 2DF (1)
Como CD = CF + FD = 1, resulta DF = 1
3 . Por tanto el área buscada resulta
[ADF] =
1
2
AD DF =
1
2
1
1
3
=
1
6
62
43. Sea ABC un triángulo isósceles con AB = BC = 10 y AC = 16. Sea BD la mediana trazada sobre el lado AC
y sea G el baricentro. Entonces el área del triángulo ADG es
Solución:
Por ser BD mediana, D es punto medio de AC, o sea AD = DC = 8. Ya que 4ABC es isósceles, BD además
de mediana también es altura, luego mADB = 90o
. Aplicando el teorema de Pitágoras hallamos que
BD =
p
102 82 = 6
Como G es el baricentro, BG = 2 GD y como BD = BG + GD = 6, resulta GD = 2. Por tanto el 4ADG
resulta ser un triángulo rectángulo con catetos de longitudes 8 y 2, por tanto su área es
[ADG] =
1
2
AD DG =
1
2
8 2 = 8
44. Sea ABC un triángulo isósceles con AB = AC = 17cm y P un punto cualquiera del lado BC, diferente de los
puntos extremos. Por P se trazan una paralela a AC que corta a AB en Q y una paralela a AB que corta a
AC en R. El perímetro del cuadrilátero AQPR es.
Solución:
Dado que QPkAC y RPkAB, AQPR es un paralelogramo y de ahí AQ = RP y AR = QP. Del paralelismo de
los segmentos señalados anteriormente también resulta que los 4QBP y 4RPC son semejantes con el 4ABC
y por tanto también son isósceles y de ahí QB = QP y RP = RC. Por tanto el perímetro del cuadrilátero
AQPR, resulta
P = AQ + QP + AR + RP
= (AQ + QB) + (AR + RC)
= AB + AC
= 17 + 17
= 34
63
45. De acuerdo a la información que se proporciona en la …gura, el segmento de mayor longitud es.
Solución:
Dado que la suma de los ángulos internos de un triángulo suman 180o
, en el 4ABD, resulta que el ángulo
ABD mide 180o
70o
60o
= 50o
y en el 4BDC, mBDC = 180o
55o
60o
= 65o
.
Una de las propiedades de los triángulos indica que el lado mayor se opone al ángulo mayor y viceversa. Al
comparar las medidas de los ángulos del 4ABD, resulta que el mayor mide 70o
y su lado opuesto es BD, luego
BD es mayor que AB y AD. Pero al considerar el 4BDC, su ángulo mayor es 65o
y el lado que se le opone
es BC y por tanto BC > BD. Luego el lado mayor de la …gura resulta BC.
46. En la …gura ABCD es un cuadrado de lado 1; 4CMN es equilátero. El área de 4CMN es igual a.
Solución:
El área de un triángulo equilátero está dada por
p
3
4 x2
, donde x es la longitud de su lado, luego debemos
encontrar primero cuanto mide cada lado del triángulo equilátero CMN.
Como ABCD es un cuadrado y CM = CN = x, se tiene que 4CDM = 4CBN y de ahí MD = NB y como
AD = AB, resulta AM = AN y por tanto el 4MAN es rectángulo isósceles, luego
MN = x
x =
p
2AN
AN =
x
p
2
Como AB = 1, resulta NB = 1 AN = 1 xp
2
=
p
2 xp
2
. El 4CBN es un triángulo rectángulo luego al aplicar
el teorema de Pitágoras resulta
CN2
= CB2
+ NB2
= 1 +
p
2 x
p
2
!2
= x2
64
Al desarrollar, simpli…car y resolver la ecuación resultante se obtiene x =
p
6
p
2. Por tanto el área buscada
es
[CMN] =
p
3
4
p
6
p
2
2
0:4641
47. La siguiente …gura muestra dos cuadrados de lado 1cm, donde AEFG se ha obtenido de ABCD al girar este
cuadrado 45 sobre el vértice A. Entonces el área sombreada es.
Solución:
Al girar 45o
, la recta diagonal AC se convierte en la recta AB la cual equivale a la recta diagonal AF, por
tanto A, B, F son colineales. Además se tiene mBFH = 45o
y por tanto FBH es un triángulo rectángulo
isósceles con FB = BH.
Luego [AGHB] = [AGF] [FBH]. Por ser AEFG un cuadrado de lado 1, su diagonal mide
p
2 y como
AB = 1, BF = BH =
p
2 1. Como [AGF] tiene como área la mitad de la área del cuadrado, que tiene lado
de longitud 1, resulta
[AGHB] =
1
2
1
2
p
2 1
2
=
1
2
1
2
2 2
p
2 + 1 =
p
2 1
48. Los ángulos agudos de un triángulo rectángulo, que también es isósceles, miden
Solución:
Por ser triángulo rectángulo isósceles tiene un ángulo de 90o
y los otros dos ángulos congruentes, y dado que
la suma de los ángulos internos de un triángulo suman 180o
, cada uno de ellos mide 45o
.
49. En la …gura ABCD es un cuadrilátero con AD kBC . La diagonal AC es perpendicular al lado CD .mBAC =
30 ; AC = 4
p
3 y AB = BC. Entonces el área de ABCD es igual a.
65
Solución:
Como AB = BC, el 4ABC es isósceles con mABC = 120o
y mBCA = 30o
y su base AC = 4
p
3. Al trazar
una perpendicular desde B a AC, sea E el pie de la perpendicular.
Por ser isósceles, BE también es mediana es decir E es punto medio de AC, luego se forman dos triángulos 30
–60 con las hipotenusas AB = BC y catetos mayor AE = EC = AC
2 = 2
p
3: Luego como el cateto mayor en
un triángulo 30 –60, es veces el cateto menor, en este caso se tiene BE = 2.
Como ADkBC el BAD es el suplemento del ABC, resulta mBAD = 180o
120o
= 60o
y como mBAC =
30o
, se tiene mCAD = 30o
y de ahí también el 4ADC es un triángulo 30 –60 con cateto mayor AC = 4
p
3.
Como en todo triangulo 30 –60, el cateto mayor es
p
3 el cateto menor, se tiene CD = 4. Finalmente tenemos
[ABCD] = [ABC] + [ACD]
=
1
2
AC BE +
1
2
AC CD
=
1
2
4
p
3 2 +
1
2
4
p
3 4
= 12
p
3
50. Se tiene un trapecio ABCD donde BC es la base menor. BC = 10cm y CD = 20cm. Las medidas de los
ángulos A; B y C son 30 ; 150 y 120 respectivamente, entonces el área del trapecio mide.
Solución:
Sean B0 y C0 las proyecciones de B y C sobre la base mayor y sean AB0 = x, C0D = y. Por ser BC paralela a
AD, mD = 180 mC = 180o
120o
= 60o
.
El 4CC0D es un triángulo 30 –60, luego h = CC0 = 10
p
3 y y = C0D = 10. También el 4AB0B resulta ser
un triángulo 30 –60, con su cateto menor BB0 = h = 10
p
3, luego AB0 = 10
p
3
p
3 = 30.
Tenemos entonces que la base mayor mide
AD = x + 10 + y = 30 + 10 + 10 = 50
66
Luego el área del trapecio resulta
[ABCD] =
(50 + 10) 10
p
3
2
= 300
p
3
51. En la …gura, mBAC = ; mBPC = m y BQC = 90 : Entonces la medida de BHC es.
Solución:
Como mBPC = mBQC = 90 , también mAPH = mAQH = 90 . APHQ es un cuadrilátero convexo y
en todo cuadrilátero convexo la suma de sus ángulos internos es 360o
, luego mBHC + + 90o
+ 90o
= 360o
y de ahí mBHC = 180o
.
52. Si las medianas en un triángulo rectángulo, trazadas a partir de los vértices de los ángulos agudos miden 5cm
y
p
20cm, entonces la medida en cm de la hipotenusa del triángulo rectángulo es.
Solución:
Sean M y N los puntos medios de BC y AB respectivamente. Sean AM =
p
20 y CN = 5, BC = a, AB = c,
luego BM =
a
2
y NB =
c
2
.
Sea la hipotenusa AC = b. Aplicando el teorema de Pitágoras en los 4ABM y BCN
AM2
= AB2
+ BM2
= c2
+
a2
4
= 20 (1)
CN2
= NB2
+ BC2
= a2
+
c2
4
= 25 (2)
Al sumar (1) y (2) resulta
5c2
4
+
5a2
4
= 45
a2
+ c2
=
4
5
(45) = 36 = b2
b = 6
67
53. En la …gura, los dos cuadrados tienen el mismo centro. La razón entre el lado del cuadrado menor y el lado del
cuadrado mayor es
2
5
. Entonces la razón entre el área sombreada y el área del cuadrado mayor es.
Solución:
Sean “b”la longitud del lado del cuadrado menor y “a”la longitud del lado del cuadrado mayor, luego
b
a
=
2
5
.
Por la simetría de la …gura se deduce que el área sombreada, es decir el trapecio ABFE, representa la cuarta
parte de la diferencia entre los dos cuadrados, luego
[ABFE] =
1
4
([ABCD] [EFGH])
[ABCD] = a2
y [EFGH] = b2
y de ahí [ABFE] =
1
4
a2
b2
. La razón buscada será
[ABFE]
[ABCD]
=
1
4 a2
b2
a2
=
1
4
a2
b2
a2
=
1
4
1
b2
a2
Como
b
a
=
2
5
, resulta
[ABFE]
[ABCD]
=
1
4
1
4
25
=
21
100
54. En la …gura, AB = AC = 4, BD = DC = 3 y mBAC = 60 , entonces la longitud del segmento AD es
Solución:
Al unir B con C, obtenemos un triángulo equilátero, ya que AB = AC y mBAC = 60 . Se tiene que
4ABD = 4ACE, ya que sus tres pares de lados son congruentes, de ahí resulta mBAD = mCAD y por
tanto AD es bisectriz del BAC.
68
Al prolongar AD, sea E el punto donde corta a BC. Luego como el 4ABC es equilátero, AE además de
bisectriz es mediatriz y por tanto AE ? BC y BE = EC = 2. Resulta entonces que el 4BED es rectángulo,
con hipotenusa BD = 3 y un cateto, BE = 2. Por el Teorema de Pitágoras, DE =
p
32 22 =
p
5:
Por otro lado AE es una altura en un triángulo equilátero de lado 4 y por tanto AE = 2
p
3. Finalmente
obtenemos que AD = AE DE = 2
p
3
p
5:
55. En la …gura el cuadrilátero ACDE es un trapecio tal que ED = 15cm , AC = 24 cm y la altura es 12cm.
Sabiendo que B es el punto medio del lado AC, el área del cuadrilátero OBCD es.
Solución:
Como EDkAC, resulta que 4ABO 4DEO, con razón de semejanza AB
DE =
12
15
=
4
5
.
Sean a y b las alturas de los triángulos ABO y DEO respectivamente, indicadas en la …gura. Dado que los
elementos homólogos en triángulos semejantes están en la misma razón de semejanza, se tiene
a
b
=
4
5
.
Como a + b = 12 (la altura del trapecio), al considerar la razón anterior resulta a =
16
3
, b =
20
3
. Al analizar la
…gura vemos que [OBCD] = [ACDE] [ABE] [DEO]. Tenemos que el área del trapecio ACDE resulta
[ACDE] =
24 + 15
2
12 = 234
El 4ABE, tiene base 12 y altura 12, luego su área es
[ABE] =
1
2
12 12 = 72
Para el 4DEO, resulta
[DEO] =
1
2
15
20
3
= 50
) [OBCD] = 234 72 50 = 112
56. En la …gura, ABCD es un cuadrado de lado 6cm y CE = DE = 5cm, entonces la longitud de es.
69
Solución:
Sean F y G los puntos medios de CD y BA respectivamente. Luego CF = FD = BG = GA = 3 y FG = 6:
Como CE = DE, el 4CED es isósceles y por tanto E, F, G son colineales y EF ? CD y EG ? AB. El 4CFE
es rectángulo en F, luego por el Teorema de Pitágoras, EF =
p
52 32 = 4. EG = EF + FG = 4 + 6 = 10.
El 4FGA también es rectángulo con EG = 10 y GA = 4, luego EA =
p
102 + 32 =
p
109:
57. En la …gura, a partir de la información dada, ¿cuál es el valor de x?
Solución:
Se tiene A = E , por dato, y ACB = ECD, por ser opuestos por el vértice, luego 4ABC 4EDC,
por el teorema de semejanza AA. Entonces:
CD
CE
=
BC
AC
x
10
=
66
132
x = 5
58. ABCD es un paralelogramo. P es un punto de la diagonal AC. Trazamos por P paralelas a los lados del
paralelogramo. Estas paralelas intersecan a los lados del paralelogramo en los puntos indicados en la …gura.
Sabiendo que el área de ABCD es 40cm2
, entonces el área del cuadrilátero RQMN es igual a.
Solución:
Dado que RMkADkBC y NQkABkDC, resulta que los cuadriláteros ANPR, PQBR, DMPN y MCQP son
paralelogramos y los segmentos NR, RQ, QM y MN son diagonales de esos paralelogramos. Es sabido que
una diagonal divide a un paralelogramo en dos triángulos congruentes, con áreas igual a la mitad del área del
paralelogramo. Por tanto [RQMN] = [ABCD] = 20:
70
59. En el triángulo rectángulo ABC¿cuál es la longitud del segmentoBC?
Solución:
Basta aplicar el teorema del cateto:
3x = 62
x = 12
60. Sea ABCD un cuadrado. Por el vértice A se traza un segmento que corta a la prolongación del ladoBC en E,
al lado DC en F y a la diagonal BD en G. Si AG = 3 y GF = 1 ¿cuál es la longitud de FE?
Solución:
Sea x la longitud de cada lado del cuadrado. Desde G tracemos una perpendicular a AD y sea H el pie de esta
perpendicular. Luego 4AGH 4AFD. Como AG = 3 y GF = 1, resulta AF = 4:
De la semejanza se tiene
AG
AF
=
AH
AD
3
4
=
AH
x
AH =
3
4
x
y HD = AD AH = x
3
4
x =
1
4
x:
Como G está sobre la diagonal, HG = HD = 1
4 x. De la misma semejanza se tiene
AG
AF
=
HG
DF
3
4
=
x
4
DF
DF =
1
3
x
71
Luego FC = DC DF = x 1
3 x = 2
3 x. Como ABCD es un cuadrado, ADkBC y por tanto ADkCE. De ahí
resulta que 4ADF 4ECF. De esta semejanza se tiene
FE
FA
=
FC
FD
FE
4
=
2
3 x
1
3 x
FE = 8
61. En la …gura de abajo si la medida de los arcos AD y BC son 140o
y 80 respectivamente, entonces el valor de
es.
Solución:
Tenemos que =
mdAC + mdCD
2
. Dado que
mdAB + mdBC + mdCD + mdAD = 360o
y
mdBC + mdAD = 80o
+ 140o
= 220o
Entonces mdAC + mdCD = 140o
, luego =
mdAC + mdCD
2
= 140o
2 = 70o
62. El triángulo ABC está inscrito en un semicírculo de diámetro AB. Si AC = 8 y CD = 6, el área de la región
sombreada tiene un valor de.
Solución:
El área sombreada es la diferencia entre el área del semicírculo y el área del triángulo. El 4ABC es rectángulo
en C, por estar inscrito en un semicírculo.
Luego por el Teorema de Pitágoras, AB =
p
82 + 62 = 10, entonces r = 5. Por tanto el área del semicírculo es
1
2
r2
=
1
2
52
=
25
2
El área del triángulo está dada por
1
2
AC BC =
1
2
8 6 = 24
El área buscada es
A =
25
2
24 15:27
72
63. El triángulo ABC está inscrito en un semicírculo de diámetro AB. Si AC = 8 y CD = 4:8, el área de la región
sombreada tiene un valor de
Solución:
Por el Teorema de Pitágoras, AD =
p
82 4:82 = 6:4. Como el 4ABC es rectángulo en C, se tiene por el
teorema del cateto . Y de nuevo por el Teorema de Pitágoras resulta BC = 6. Dado que estos valores coinciden
con los datos del ejercicio anterior, el área resulta la misma.
64. La circunferencia de la …gura tiene radio 2 y el arco XY Z tiene longitud . ¿Cuánto mide la cuerda XZ?
Solución:
Sea la medida del ángulo central XOZ. La longitud de un arco está dada por s = r , con el ángulo medido
en radianes. Tenemos s = y r = 2, luego = s
r = 2 , es decir 90o
.
Luego XOZ es un triángulo rectángulo isósceles con XZ como hipotenusa y por tanto XZ = 2
p
2
65. En la …gura el área del círculo mayor es 1 m2
. El círculo menor es tangente internamente al círculo mayor y
también es tangente a los lados del ángulo inscrito que mide 60 . Entonces el área del círculo menor es
73
Solución:
Desde el vértice del ángulo inscrito, trazamos un diámetro. Sean O y O0 los centros de los círculos, mayor y
menor respectivamente. Sea B el otro extremo del diámetro trazado, C el punto donde uno de los lados (el
arriba) del ángulo corta a la circunferencia. Y sea D el punto de tangencia de este lado del ángulo con el círculo
menor.
Sean R y r los radios de los círculos, mayor y menor respectivamente.Tenemos que AO0 biseca al ángulo inscrito,
luego mO0AD = mBAC = 30o
.
Como AB es un diámetro del circulo mayor, mACB = 90o
resultando que mABC = 60o
, y el 4ABC es
30 –60.
Dado que AB = 2R, se obtiene que BC = R, ya que BC es el cateto menor y AB la hipotenusa del 4ABC.
Como AC es tangente al círculo menor en D, AD?DO0, es decir mADO0 = 90o
y de ahí mAO0D = 60o
.
Luego también el 4AO0D es un triángulo 30 –60 y su cateto menor O0D = r, mide la mitad de su hipotenusa,
AO0. Se tiene O0B = r, por ser radio del circulo menor y de ahí AO0 = AB O0B = 2R r. Luego
AO0 = 2 O0D
2R r = 2r
r =
2
3
R
Como el área del círculo mayor es R2
= 1 el área del círculo menor es
r2
=
2R
3
2
=
4
9
R2
=
4
9
66. En la …gura C es el centro de la circunferencia de radio r y TP es un segmento tangente en T, de longitud 2r,
entonces PC mide
Solución:
74
Como TP es tangente a la circunferencia en T, mPTC = 90o
. Luego aplicando el teorema de Pitágoras
resulta
PC =
q
(2r)
2
+ r2 =
p
5r2 = r
p
5
67. Los extremos de la …gura son semicírculos, ¿Cuál es el área de la región sombreada?
Solución:
Como el área sombreada únicamente son los extremos y estos son semicírculos, al unirlos se forma un circulo
de diámetro 8, es decir de radio 4, luego
A = r2
= 42
= 16
68. En la …gura AC es un diámetro. Si mAB = 50 , entonces mBAC =?
Solución:
Dado que mdAB = 50 , se tiene mBCA = 1
2 mdAB = 25 , por ser ángulo inscrito que subtiende dicho arco;
mABC = 90o
, por estar inscrito en una semicircunferencia ( es diámetro). Luego la medida del ángulo
buscado es:
mBAC = 180 mBCA mABC
= 180o
25o
90o
= 65o
69. En la …gura, los círculos son tangentes y tienen radio igual a 10. Si se unen los centros de los círculos se forma
un cuadrado. ¿Cuál es el área de la región sombreada?
75
Solución:
El área de la región sombreada es la diferencia entre el área del cuadrado formado y las áreas de los cuatro
sectores circulares que se forman. Dado que la distancia entre los centros de dos círculos tangentes exterior-
mente, es la suma de las longitudes de los radios, resulta que el cuadrado formado tiene lado de longitud 20 y
por tanto el área del cuadrado es 400.
Cada sector formado tiene un ángulo central de 90o
, luego entre los cuatro forman un circulo de radio 10, cuyas
áreas suman entonces r2
= 102
= 100 : Por tanto el área buscada es:
A = 400 100
70. En la …gura, la medida del arco AB es 30 , y la medida del BPA es 35 . Las medidas del arco CD y el
ángulo DAC (en grados) son respectivamente.
Solución:
Dado que BPA es un ángulo exterior, formado por dos secantes, su medida es la semidiferencia de los las
medidas de los arcos que intercepta. Es decir
mBPA =
mdCD mdAB
2
De esta expresión despejamos mdAB, resultando
mdCD = 2 mBPA + mdAB
= 2 35o
+ 30o
= 100o
Por otro lado se tiene que el DAC es un ángulo inscrito que subtiende el arco DC, luego
mDAC =
1
2
mdCD =
1
2
100o
= 50o
71. La expresión (p + q)p = (r + s)r, se cumple en la situación representada por
76
Solución:
Al recordar las relaciones métricas en una circunferencia, vemos que los productos de esta forma surgen cuando
se tienen dos secantes que se cortan (también aparecen cuando hay semejanzas de triángulos) o una secante y
una tangente que se cortan. A partir de estas relaciones tenemos:
En la …gura a), la relación es r2
= s (s + p) :
En la …gura b), la relación es r(r + s) = p(p + q), la cual es la misma expresión dada. La respuesta es ésta.
Para estar más seguros vemos que resulta en las otras.
En la …gura c), la relación es r s = p q y en la …gura d), r2
= (p + q), que son diferentes a la dada.
Solo b) satisface y por tanto es la respuesta.
72. En la …gura se dan tres semicircunferencias mutuamente tangentes.CD y DA son diámetros de las circunfer-
encias menores. El punto B está en la semicircunferencia mayor. BD ? BC . Si BD = 2; entonces el área
sombreada es igual a.
Solución:
El área de la región sombreada es la diferencia entre el área del semicírculo exterior menos las áreas de los
semicírculos interiores. Sean r1, r2, R los radios del semicírculo menor, del semicírculo mediano y del semicírculo
exterior respectivamente. Luego CD = 2r1, DA = 2r2 y CA = 2R. Como CA = CD + DA, se tiene
2R = 2r1 + 2r2
R = r1 + r2 (1)
Al unir B con A y con C, se forma un triángulo rectángulo, con CA como hipotenusa y BD como altura
relativa a la hipotenusa. Por el teorema de la altura,
BD2
= CD DA
22
= 2r1 2r2
r1 r2 = 1 (2)
Como el área de un semicírculo está dada por 1
2 r2
, el área buscada es
A =
1
2
R2
r2
1 r2
2
77
Al considerar (1)
A =
1
2
h
(r1 + r2)
2
r2
1 r2
2
i
=
1
2
r2
1 + 2r1r2 + r2
2 r2
1 r2
2
=
1
2
2r1r2
= r1r2
Al considerar (2)
A = r1r2 = 1
73. Las medidas de los arcos AB y AC se indican en la …gura. La medida del BAC es.
Solución:
El BAC es un ángulo inscrito en una circunferencia, por tanto su medida es la mitad de la medida del arco
que subtiende, en este caso el arco BC. Tenemos que
mdBC = 360o
mdAB mdAC = 360o
110o
130o
= 120o
Luego mBAC = 1
2 mdBC = 60o
74. En la …gura, BC une los centros de los círculos tangentes. AB ? BC; BC = 8 y AC = 10, entonces la longitud
de la circunferencia pequeña es igual a
Solución:
Sean R y r los radios de las circunferencias grande y pequeña respectivamente. Como las circunferencias son
tangentes exteriormente, R + r = BC = 8. Dado que el 4ABC es rectángulo en B, tenemos R = AB =
p
102 82 = 6 y r = 2. Luego la longitud de la circunferencia pequeña resulta C = 2 r = 4 .
78
75. La …gura representa un hexágono regular, ¿cuál es el valor de x?
Solución:
Todo hexágono regular puede dividirse en seis triángulos equiláteros congruentes. En la …gura se indica que x
equivale al doble de la altura de cada triangulo: x = 2h. Como el lado de cada triangulo mide 6
p
3, las alturas
miden h =
p
3
2 6
p
3 = 9
) x = 2h = 18
76. La …gura representa un círculo inscrito en un cuadrado que a su vez está inscrito en otro cuadrado. B es punto
medio de AC ¿Cuál es el área de la región sombreada?
Solución:
Si llamamos A1 al área del cuadrado mayor,A2 al área del cuadrado menor y A3 al área del círculo, el área
de la región sombreada resulta A = A1 A2 + A3: El lado del cuadrado mayor mide 0:4, luego su área es
A1 = 0:16. Como B es punto medio AB = 0:2. Los triángulos que se forman en cada esquina del cuadrado
mayor, son rectángulos isósceles, y sus hipotenusas forman los lados del cuadrado menor, por tanto, el lado del
cuadrado menor resulta 0:2
p
2 y su área es A2 = 0:2
p
2
2
= 0:08.
Como el circulo está inscrito en el cuadrado menor, su diámetro es el lado de dicho cuadrado, y su radio es la
mitad o sea r = 0:1
p
2, su área A3 = r2
= 0:1
p
2
2
= 0:0628. Luego
A = 0:16 0:08 + 0:0628 = 0:1428
77. Los segmentos AC y BD se cortan en P y son tangentes a las circunferencias en los puntos A, C, B y D.
79
Solución:
Dado que PB y PC son segmentos tangentes a la circunferencia de la izquierda, desde un mismo punto, son
congruentes, luego PC = PB = 19. Como AC = AP + PC,
AP = AC PC = 31 19 = 12
78. Seis triángulos equiláteros de 1cm. de lado se unen para formar un hexágono como se muestra en la …gura. Se
circunscribe un círculo alrededor del hexágono ¿cuál es el área de la región sombreada?
Solución:
Tenemos que el área de la región sombreada es el área del circulo menos el área del hexágono. El radio del
circulo es la longitud del lado de los triángulos, es decir r = 1, luego su área es r2
= .
El área de cada triángulo equilátero es
p
3
4 x2
, donde x es el lado del triángulo, y como el lado mide 1, se reduce
a
p
3
4 . Como hay seis triángulos, el área del hexágono es 6
p
3
4 . Por tanto el área de la región sombreada es
A =
p
3
2
!
cm2
79. Un triángulo ABC está inscrito en una circunferencia como se muestra en la …gura. Se tiene mA = 50o
y
mC = 60o
. Se trazan tangentes por A; B y Cde manera que se forma el triángulo circunscrito A
0
; B
0
; C
0
.
Entonces la medida del ángulo A
0
es:
Solución:
80
Como BA0 y CA0 son tangentes a la circunferencia, los A0BC y A0CB son ángulos semiinscritos que
subtienden el arco BC y el ángulo A es un ángulo inscrito que subtiende el mismo arco. Por tanto estos
ángulos son congruentes, es decir
mA0BC = mA0CB = mA = 50o
Luego al considerar el 4A0BC, se tiene
mA0 = 180 mA0BC mA0CB
= 180o
50o
50o
= 80o
80. El triángulo ABC es equilátero y sus lados AC y BC son tangentes a la circunferencia con centro en O y radio
p
3. El área del cuadrilátero AOBC es
Solución:
Se tiene OC?AB, ya que los triángulos OAB y ABC son isósceles. También 4OAC = 4OBC, ya que sus tres
lados son congruentes. Como además el 4ABC es equilátero, mACO = 30o
, luego el 4OAC es un triángulo
30 –60 y de ahí resulta que OC = 2
p
3 y AC = 3.
Tenemos entonces
[AOBC] = 2 [OAC] = 2
1
2
p
3 3 = 3
p
3
81. Si un ángulo central de 30 en una circunferencia intercepta un arco de 6m de longitud, entonces el radio de la
circunferencia mide.
Solución:
Se tiene s = r , donde s es la longitud del arco, r el radio de la circunferencia y es el ángulo central
correspondiente, medido en radianes. Como = 30o
equivale a =6 radianes, tenemos
6 = r
6
r =
36
81
82. En la …gura se tiene una circunferencia de radio 1 y un hexágono regular de lado 1. Si O es el centro de la
circunferencia, entonces el área de la región sombreada es.
Solución:
En vista que el hexágono tiene lado 1 y la circunferencia tiene radio 1, el centro del hexágono es un punto de
la circunferencia. La región sombreada puede descomponerse en dos triángulos que tienen la misma base y la
misma altura que los triángulos que forman el hexágono. Luego el área buscada es
A = 2
p
3
4
12
=
p
3
2
0:866
83. Los arcosAB y BC son semicírculos cuyos centros están sobre un diámetro del círculo que se muestra en la
…gura.Si BC = 2AB, entonces la razón entre el área de la región sombreada y el área de la región no sombreada
es:
Solución:
Sean r1 el radio del semicírculo mayor,r2 el radio del semicírculo mediano y r3 el radio del semicírculo menor.
Se tiene
r3 = AB
2 BC = 2AB AC = AB + BC
2r2 = 2AB 2r1 = 3AB
r2 = AB r1 = 3
2 AB
Luego las áreas de estos semicírculos son:
Semicírculo mayor: 1
2 r2
1 = 1
2
3
2 AB
2
= 9
8 AB2
Semicírculo mediano: 1
2 r2
2 = 1
2 AB2
Semicírculo menor: 1
2 r2
3 = 1
2
AB
2
2
= 1
8 AB2
82
El área sombreada está dada por: área del semicírculo mayor menos el área del semicírculo mediano más el
área del semicírculo menor o sea
Área sombreada = 9
8 AB2 1
2 AB2
+ 1
8 AB2
= 3
4 AB2
La razón buscada resulta
Área sombreada
Área no sombreada
=
3
4
AB2
3
2
AB2
=
1
2
84. Una moneda circular de radio 1, está sobre una mesa. Si ponemos cuatro monedas más grandes de igual tamaño
alrededor de ella, ¿cuál es el radio de las monedas grandes que permite que cada una sea tangente a las dos
adyacentes y a la de radio 1?
Solución:
Sea R el radio de las monedas grandes. Como estas monedas son tangentes a las monedas adyacentes y a la
vez son tangentes a la moneda pequeña, al unir los centros de las monedas grandes se forma un cuadrado de
lado 2R.
Al trazar una diagonal, esta debe pasar por el centro de la moneda pequeña, la cual tiene diámetro 2, luego la
longitud de la diagonal resulta 2R + 2.
Por tanto, dado que en todo cuadrado de lado x, su diagonal mide
p
2x, se cumple en este caso que
2R + 2 =
p
2 (2R)
2
p
2 2 R = 2
R =
1
p
2 1
Al racionalizar el denominador obtenemos R =
p
2 + 1
85. En la siguiente …gura ABC y AEB son semicírculos, F es el punto medio del diámetro AC; B es punto medio
del arco AC y AF = 1. ¿Cuál es el área de la región sombreada?
83
Solución:
El área de la región sombreada resulta de la diferencia entre el semicírculo AEB y el segmento circular deter-
minado por la cuerda AB en el semicírculo ABC.
Como F es el punto medio del diámetro AC, B es punto medio del arco AC, resulta BF?AC, luego el 4ABF
es un triángulo rectángulo isósceles de cateto 1 y por tanto AB =
p
2. AB es diámetro del semicírculo AEB,
luego su radio es
p
2
2 y el área de este semicírculo resulta
A1 =
1
2
p
2
2
!2
=
4
El área del segmento circular, está dada por la diferencia entre el área del sector circular que lo contiene y el
área del triángulo determinado por la cuerda y los radios extremos.
En este caso el sector circular correspondiente tiene ángulo central de 90o
y radio 1, por tanto su área es la
cuarta parte del área de un círculo de radio 1 o sea 1
4 y el triángulo correspondiente tiene base 1 y altura 1,
luego su área es 1
2 . El área del segmento circular resulta A2 =
1
4
1
2
Finalmente el área buscada es
A = A1 A2 =
4 4
1
2
=
1
2
86. Si el radio de un círculo aumenta en unidades, ¿cuánto aumenta su perímetro?
Solución:
Sean L y L0 los perímetros del círculo original y el círculo con el radio aumentado, respectivamente. Luego
L = 2 r y L0 = 2 (r + ) = 2 r + 2 2
. El aumento es la diferencia
4 = L0 L = 2 r + 2 2
2 r = 2 2
87. Dos semicírculos de radio 3 están inscritos en un semicírculo de radio 6 como se muestra en la …gura. Un círculo
de radio r es tangente a los tres semicírculos. ¿Cuánto vale r ?
Solución:
84
Cuando se tienen círculos tangentes exteriormente, la distancia entre los centros es la suma de los radios, y
cuando son tangentes interiormente, la distancia entre los centros es la diferencia entre los radios. Además en
ambos casos los centros y el punto de tangencia están alineados.
Sean A, B, C y D los centros de los semicírculos y del círculo interior como se muestra en la …gura. Se tiene
AB = AD = 3 + r, CA = 6 r, BC = CD = 3. Como 4ABD es isósceles y C es punto medio de BD,
AC?BC, luego el 4ABC es rectángulo en C y por tanto sus lados cumplen con el teorema de Pitágoras. Luego
(3 + r)
2
= (6 r)
2
+ 32
9 + 6r + r2
= 36 12r + r2
+ 9
18r = 36
r = 2
88. En la …gura los círculos adyacentes son tangentes y tienen radio 1. ¿Cuánto vale el área de la región sombreada?
Solución:
Al considerar el círculo central y dos círculos externos contiguos, vemos que encierran la sexta parte del área
buscada. Vemos también que esta fracción corresponde al área de un triángulo equilátero de lado 2 menos tres
sectores circulares de radio 1 y de 60o
cada uno, que juntos forman un semicírculo de radio 1.
85
Luego
A = 6
"p
3
4
22 1
2
12
#
= (6
p
3 3 )u2
89. En la …gura, mBCA = 90o
; BA = 5y AC = 3: ¿Cuál es el área del círculo con centro en O?
Solución:
Como el 4ABCes rectángulo en C, aplicamos el Teorema de Pitágoras para hallar BC
BC =
p
AB2 AC2 =
p
52 32 = 4
Como BC es diámetro del círculo, se tiene r = 2 y su área resulta A = r2
= 4
90. El lado mayor del rectángulo de la …gura mide 20. La curva trazada en su interior está formada por cinco
semicircunferencias ¿cuál es la longitud de la curva?
Solución:
Se observa que la curva está formada por 5 semicircunferencias, cuyos diámetros suman 20, luego cada diámetro
mide 20 5 = 4 y los respectivos radios la mitad o sea 2 unidades. Luego
L = 5
1
2
2 r = 5 r = 5 2 = 10
91. La …gura muestra dos segmentos perpendiculares tangentes a ambas circunferencias, las cuales son tangentes
entre sí. Si el radio de la circunferencia pequeña mide 1, entonces el radio de la circunferencia más grande mide
Solución:
86
Solucionario guia de admision 2015
Solucionario guia de admision 2015
Solucionario guia de admision 2015
Solucionario guia de admision 2015
Solucionario guia de admision 2015
Solucionario guia de admision 2015
Solucionario guia de admision 2015
Solucionario guia de admision 2015
Solucionario guia de admision 2015
Solucionario guia de admision 2015
Solucionario guia de admision 2015
Solucionario guia de admision 2015
Solucionario guia de admision 2015
Solucionario guia de admision 2015
Solucionario guia de admision 2015
Solucionario guia de admision 2015
Solucionario guia de admision 2015
Solucionario guia de admision 2015
Solucionario guia de admision 2015
Solucionario guia de admision 2015
Solucionario guia de admision 2015
Solucionario guia de admision 2015
Solucionario guia de admision 2015
Solucionario guia de admision 2015
Solucionario guia de admision 2015
Solucionario guia de admision 2015
Solucionario guia de admision 2015
Solucionario guia de admision 2015
Solucionario guia de admision 2015
Solucionario guia de admision 2015
Solucionario guia de admision 2015
Solucionario guia de admision 2015
Solucionario guia de admision 2015
Solucionario guia de admision 2015
Solucionario guia de admision 2015
Solucionario guia de admision 2015
Solucionario guia de admision 2015
Solucionario guia de admision 2015
Solucionario guia de admision 2015
Solucionario guia de admision 2015
Solucionario guia de admision 2015
Solucionario guia de admision 2015
Solucionario guia de admision 2015
Solucionario guia de admision 2015
Solucionario guia de admision 2015
Solucionario guia de admision 2015
Solucionario guia de admision 2015
Solucionario guia de admision 2015
Solucionario guia de admision 2015
Solucionario guia de admision 2015
Solucionario guia de admision 2015
Solucionario guia de admision 2015
Solucionario guia de admision 2015
Solucionario guia de admision 2015
Solucionario guia de admision 2015
Solucionario guia de admision 2015
Solucionario guia de admision 2015
Solucionario guia de admision 2015
Solucionario guia de admision 2015
Solucionario guia de admision 2015
Solucionario guia de admision 2015
Solucionario guia de admision 2015
Solucionario guia de admision 2015
Solucionario guia de admision 2015
Solucionario guia de admision 2015
Solucionario guia de admision 2015
Solucionario guia de admision 2015
Solucionario guia de admision 2015
Solucionario guia de admision 2015
Solucionario guia de admision 2015
Solucionario guia de admision 2015
Solucionario guia de admision 2015
Solucionario guia de admision 2015
Solucionario guia de admision 2015
Solucionario guia de admision 2015

Más contenido relacionado

La actualidad más candente

Actividad función Monserrat
Actividad función MonserratActividad función Monserrat
Actividad función MonserratGerardo Yuszczyk
 
Solucionario de guía matemática 2018 - 2019 UNAN MANAGUA - EXAMEN DE ADMISIÓN
Solucionario de guía matemática 2018 - 2019 UNAN MANAGUA - EXAMEN DE ADMISIÓN  Solucionario de guía matemática 2018 - 2019 UNAN MANAGUA - EXAMEN DE ADMISIÓN
Solucionario de guía matemática 2018 - 2019 UNAN MANAGUA - EXAMEN DE ADMISIÓN Cliffor Jerry Herrera Castrillo
 
Presentacion de inecuaciones 2
Presentacion de inecuaciones 2Presentacion de inecuaciones 2
Presentacion de inecuaciones 2christopherjames25
 
Ecuaciones de power point
Ecuaciones de power pointEcuaciones de power point
Ecuaciones de power point1054917
 
Progresiones aritméticas y geométricas
Progresiones aritméticas y geométricasProgresiones aritméticas y geométricas
Progresiones aritméticas y geométricasAndrea Alejandra Rey
 
100 problemas maravillosos de matemáticas - Libro 14
100 problemas maravillosos de matemáticas - Libro 14100 problemas maravillosos de matemáticas - Libro 14
100 problemas maravillosos de matemáticas - Libro 14José Mari Melgarejo Lanero
 
100 problemas maravillosos de matemáticas - Libro 11
100 problemas maravillosos de matemáticas - Libro 11100 problemas maravillosos de matemáticas - Libro 11
100 problemas maravillosos de matemáticas - Libro 11José Mari Melgarejo Lanero
 
1. Diagramas de Venn (Conjuntos). Ejercicios Resueltos.pdf
1. Diagramas de Venn (Conjuntos). Ejercicios Resueltos.pdf1. Diagramas de Venn (Conjuntos). Ejercicios Resueltos.pdf
1. Diagramas de Venn (Conjuntos). Ejercicios Resueltos.pdfelvis1151
 
Solucionario Guía de Admisión 2020 Paso a Paso UNAN MANAGUA Matemática
Solucionario Guía de Admisión 2020  Paso a Paso UNAN MANAGUA Matemática Solucionario Guía de Admisión 2020  Paso a Paso UNAN MANAGUA Matemática
Solucionario Guía de Admisión 2020 Paso a Paso UNAN MANAGUA Matemática Cliffor Jerry Herrera Castrillo
 
INTRODUCCIÓN A LAS INECUACIONES
 INTRODUCCIÓN A  LAS INECUACIONES  INTRODUCCIÓN A  LAS INECUACIONES
INTRODUCCIÓN A LAS INECUACIONES Leonarda Frias
 
Ejercidos de conjuntos y proporcionalidad PDF
Ejercidos de conjuntos y proporcionalidad  PDF Ejercidos de conjuntos y proporcionalidad  PDF
Ejercidos de conjuntos y proporcionalidad PDF Yoner Chávez
 
100 problemas maravillosos de matemáticas - Libro 8
100 problemas maravillosos de matemáticas - Libro 8100 problemas maravillosos de matemáticas - Libro 8
100 problemas maravillosos de matemáticas - Libro 8José Mari Melgarejo Lanero
 
Ecuaciones de 1er grado. Solución de problemas.
Ecuaciones de 1er grado. Solución de problemas.Ecuaciones de 1er grado. Solución de problemas.
Ecuaciones de 1er grado. Solución de problemas.math class2408
 
Sistemas de numeracion
Sistemas de numeracionSistemas de numeracion
Sistemas de numeraciongerardotocto
 

La actualidad más candente (20)

Actividad función Monserrat
Actividad función MonserratActividad función Monserrat
Actividad función Monserrat
 
Operaciones con-monomios (1)
Operaciones con-monomios (1)Operaciones con-monomios (1)
Operaciones con-monomios (1)
 
Productos notables
Productos notablesProductos notables
Productos notables
 
Solucionario de guía matemática 2018 - 2019 UNAN MANAGUA - EXAMEN DE ADMISIÓN
Solucionario de guía matemática 2018 - 2019 UNAN MANAGUA - EXAMEN DE ADMISIÓN  Solucionario de guía matemática 2018 - 2019 UNAN MANAGUA - EXAMEN DE ADMISIÓN
Solucionario de guía matemática 2018 - 2019 UNAN MANAGUA - EXAMEN DE ADMISIÓN
 
Presentacion de inecuaciones 2
Presentacion de inecuaciones 2Presentacion de inecuaciones 2
Presentacion de inecuaciones 2
 
Ecuaciones de power point
Ecuaciones de power pointEcuaciones de power point
Ecuaciones de power point
 
Matematicas karen
Matematicas karenMatematicas karen
Matematicas karen
 
Solucionario onem 2017 n1
Solucionario onem 2017 n1Solucionario onem 2017 n1
Solucionario onem 2017 n1
 
Progresiones aritméticas y geométricas
Progresiones aritméticas y geométricasProgresiones aritméticas y geométricas
Progresiones aritméticas y geométricas
 
100 problemas maravillosos de matemáticas - Libro 14
100 problemas maravillosos de matemáticas - Libro 14100 problemas maravillosos de matemáticas - Libro 14
100 problemas maravillosos de matemáticas - Libro 14
 
Problemas de ONEM Fase 2
Problemas de ONEM Fase 2Problemas de ONEM Fase 2
Problemas de ONEM Fase 2
 
100 problemas maravillosos de matemáticas - Libro 11
100 problemas maravillosos de matemáticas - Libro 11100 problemas maravillosos de matemáticas - Libro 11
100 problemas maravillosos de matemáticas - Libro 11
 
Ruffini
RuffiniRuffini
Ruffini
 
1. Diagramas de Venn (Conjuntos). Ejercicios Resueltos.pdf
1. Diagramas de Venn (Conjuntos). Ejercicios Resueltos.pdf1. Diagramas de Venn (Conjuntos). Ejercicios Resueltos.pdf
1. Diagramas de Venn (Conjuntos). Ejercicios Resueltos.pdf
 
Solucionario Guía de Admisión 2020 Paso a Paso UNAN MANAGUA Matemática
Solucionario Guía de Admisión 2020  Paso a Paso UNAN MANAGUA Matemática Solucionario Guía de Admisión 2020  Paso a Paso UNAN MANAGUA Matemática
Solucionario Guía de Admisión 2020 Paso a Paso UNAN MANAGUA Matemática
 
INTRODUCCIÓN A LAS INECUACIONES
 INTRODUCCIÓN A  LAS INECUACIONES  INTRODUCCIÓN A  LAS INECUACIONES
INTRODUCCIÓN A LAS INECUACIONES
 
Ejercidos de conjuntos y proporcionalidad PDF
Ejercidos de conjuntos y proporcionalidad  PDF Ejercidos de conjuntos y proporcionalidad  PDF
Ejercidos de conjuntos y proporcionalidad PDF
 
100 problemas maravillosos de matemáticas - Libro 8
100 problemas maravillosos de matemáticas - Libro 8100 problemas maravillosos de matemáticas - Libro 8
100 problemas maravillosos de matemáticas - Libro 8
 
Ecuaciones de 1er grado. Solución de problemas.
Ecuaciones de 1er grado. Solución de problemas.Ecuaciones de 1er grado. Solución de problemas.
Ecuaciones de 1er grado. Solución de problemas.
 
Sistemas de numeracion
Sistemas de numeracionSistemas de numeracion
Sistemas de numeracion
 

Similar a Solucionario guia de admision 2015

Actividades de-vacaciones-de-matemáticas-para-4º-jesús-rodríguez-bravo
Actividades de-vacaciones-de-matemáticas-para-4º-jesús-rodríguez-bravoActividades de-vacaciones-de-matemáticas-para-4º-jesús-rodríguez-bravo
Actividades de-vacaciones-de-matemáticas-para-4º-jesús-rodríguez-bravotrosky15
 
Actividades de-vacaciones-de-matemáticas-para-4º-jesús-rodríguez-bravo
Actividades de-vacaciones-de-matemáticas-para-4º-jesús-rodríguez-bravoActividades de-vacaciones-de-matemáticas-para-4º-jesús-rodríguez-bravo
Actividades de-vacaciones-de-matemáticas-para-4º-jesús-rodríguez-bravotrosky15
 
100 problemas maravillosos de matemáticas - Libro 17
100 problemas maravillosos de matemáticas - Libro 17100 problemas maravillosos de matemáticas - Libro 17
100 problemas maravillosos de matemáticas - Libro 17José Mari Melgarejo Lanero
 
Teoria y problemas del metodo del cangrejo mc15 ccesa007
Teoria y problemas del metodo del cangrejo  mc15 ccesa007Teoria y problemas del metodo del cangrejo  mc15 ccesa007
Teoria y problemas del metodo del cangrejo mc15 ccesa007Demetrio Ccesa Rayme
 
100 problemas maravillosos de matemáticas - Libro 5
100 problemas maravillosos de matemáticas - Libro 5100 problemas maravillosos de matemáticas - Libro 5
100 problemas maravillosos de matemáticas - Libro 5José Mari Melgarejo Lanero
 
Taller de recuperacion de matematica segunda parte ciclo sexto septimo
Taller de recuperacion de matematica  segunda parte ciclo sexto septimoTaller de recuperacion de matematica  segunda parte ciclo sexto septimo
Taller de recuperacion de matematica segunda parte ciclo sexto septimoJorge Didier Obando Montoya
 
100 problemas maravillosos de matemáticas - Libro 6
100 problemas maravillosos de matemáticas - Libro 6100 problemas maravillosos de matemáticas - Libro 6
100 problemas maravillosos de matemáticas - Libro 6José Mari Melgarejo Lanero
 
100 problemas maravillosos de matemáticas - Libro 3
100 problemas maravillosos de matemáticas - Libro 3100 problemas maravillosos de matemáticas - Libro 3
100 problemas maravillosos de matemáticas - Libro 3José Mari Melgarejo Lanero
 
s12-3-sec-dia-4-solucion-matematica.pdf
s12-3-sec-dia-4-solucion-matematica.pdfs12-3-sec-dia-4-solucion-matematica.pdf
s12-3-sec-dia-4-solucion-matematica.pdfssuserb8278c
 

Similar a Solucionario guia de admision 2015 (20)

Solucionario ONEM 2018 F2N1.pdf
Solucionario ONEM 2018 F2N1.pdfSolucionario ONEM 2018 F2N1.pdf
Solucionario ONEM 2018 F2N1.pdf
 
Solucionario onem 2018 f2 n1
Solucionario onem 2018 f2 n1Solucionario onem 2018 f2 n1
Solucionario onem 2018 f2 n1
 
SOLUCIONARIO ONEM 2019 F1 N1
SOLUCIONARIO ONEM 2019 F1 N1SOLUCIONARIO ONEM 2019 F1 N1
SOLUCIONARIO ONEM 2019 F1 N1
 
Psicot. trucos
Psicot. trucosPsicot. trucos
Psicot. trucos
 
Examen bimestra il primero-solucion
Examen bimestra il   primero-solucionExamen bimestra il   primero-solucion
Examen bimestra il primero-solucion
 
Examen bimestra il primero-solucion
Examen bimestra il   primero-solucionExamen bimestra il   primero-solucion
Examen bimestra il primero-solucion
 
Actividades de-vacaciones-de-matemáticas-para-4º-jesús-rodríguez-bravo
Actividades de-vacaciones-de-matemáticas-para-4º-jesús-rodríguez-bravoActividades de-vacaciones-de-matemáticas-para-4º-jesús-rodríguez-bravo
Actividades de-vacaciones-de-matemáticas-para-4º-jesús-rodríguez-bravo
 
cuaderno-vacaciones 4.pdf
cuaderno-vacaciones 4.pdfcuaderno-vacaciones 4.pdf
cuaderno-vacaciones 4.pdf
 
Actividades de-vacaciones-de-matemáticas-para-4º-jesús-rodríguez-bravo
Actividades de-vacaciones-de-matemáticas-para-4º-jesús-rodríguez-bravoActividades de-vacaciones-de-matemáticas-para-4º-jesús-rodríguez-bravo
Actividades de-vacaciones-de-matemáticas-para-4º-jesús-rodríguez-bravo
 
100 problemas maravillosos de matemáticas - Libro 17
100 problemas maravillosos de matemáticas - Libro 17100 problemas maravillosos de matemáticas - Libro 17
100 problemas maravillosos de matemáticas - Libro 17
 
Teoria y problemas del metodo del cangrejo mc15 ccesa007
Teoria y problemas del metodo del cangrejo  mc15 ccesa007Teoria y problemas del metodo del cangrejo  mc15 ccesa007
Teoria y problemas del metodo del cangrejo mc15 ccesa007
 
100 problemas maravillosos de matemáticas - Libro 5
100 problemas maravillosos de matemáticas - Libro 5100 problemas maravillosos de matemáticas - Libro 5
100 problemas maravillosos de matemáticas - Libro 5
 
Taller de recuperacion de matematica segunda parte ciclo sexto septimo
Taller de recuperacion de matematica  segunda parte ciclo sexto septimoTaller de recuperacion de matematica  segunda parte ciclo sexto septimo
Taller de recuperacion de matematica segunda parte ciclo sexto septimo
 
Mate grado 7°
Mate grado 7°Mate grado 7°
Mate grado 7°
 
100 problemas maravillosos de matemáticas - Libro 6
100 problemas maravillosos de matemáticas - Libro 6100 problemas maravillosos de matemáticas - Libro 6
100 problemas maravillosos de matemáticas - Libro 6
 
I bimestre -- 2014
I bimestre  -- 2014I bimestre  -- 2014
I bimestre -- 2014
 
100 problemas maravillosos de matemáticas - Libro 3
100 problemas maravillosos de matemáticas - Libro 3100 problemas maravillosos de matemáticas - Libro 3
100 problemas maravillosos de matemáticas - Libro 3
 
P23 prueba 1 del modelo del bimestral solucion
P23 prueba 1 del  modelo del bimestral solucionP23 prueba 1 del  modelo del bimestral solucion
P23 prueba 1 del modelo del bimestral solucion
 
s12-3-sec-dia-4-solucion-matematica.pdf
s12-3-sec-dia-4-solucion-matematica.pdfs12-3-sec-dia-4-solucion-matematica.pdf
s12-3-sec-dia-4-solucion-matematica.pdf
 
Solucion 02
Solucion 02Solucion 02
Solucion 02
 

Más de Jose Adan Duarte Urbina (7)

Conversión
ConversiónConversión
Conversión
 
Regla de 3
Regla de 3 Regla de 3
Regla de 3
 
Trabajo master
Trabajo masterTrabajo master
Trabajo master
 
Funciones
FuncionesFunciones
Funciones
 
Factorizacion
FactorizacionFactorizacion
Factorizacion
 
Volltext algebra superior
Volltext algebra superiorVolltext algebra superior
Volltext algebra superior
 
Fundamentos de termodinámica van wylen - 6ed
Fundamentos de termodinámica   van wylen - 6edFundamentos de termodinámica   van wylen - 6ed
Fundamentos de termodinámica van wylen - 6ed
 

Último

3. Pedagogía de la Educación: Como objeto de la didáctica.ppsx
3. Pedagogía de la Educación: Como objeto de la didáctica.ppsx3. Pedagogía de la Educación: Como objeto de la didáctica.ppsx
3. Pedagogía de la Educación: Como objeto de la didáctica.ppsxJuanpm27
 
Presentacion minimalista aesthetic simple beige_20240415_224856_0000.pdf
Presentacion minimalista aesthetic simple beige_20240415_224856_0000.pdfPresentacion minimalista aesthetic simple beige_20240415_224856_0000.pdf
Presentacion minimalista aesthetic simple beige_20240415_224856_0000.pdfSarayLuciaSnchezFigu
 
IV SES LUN 15 TUTO CUIDO MI MENTE CUIDANDO MI CUERPO YESSENIA 933623393 NUEV...
IV SES LUN 15 TUTO CUIDO MI MENTE CUIDANDO MI CUERPO  YESSENIA 933623393 NUEV...IV SES LUN 15 TUTO CUIDO MI MENTE CUIDANDO MI CUERPO  YESSENIA 933623393 NUEV...
IV SES LUN 15 TUTO CUIDO MI MENTE CUIDANDO MI CUERPO YESSENIA 933623393 NUEV...YobanaZevallosSantil1
 
Mapa Mental de estrategias de articulación de las areas curriculares.pdf
Mapa Mental de estrategias de articulación de las areas curriculares.pdfMapa Mental de estrategias de articulación de las areas curriculares.pdf
Mapa Mental de estrategias de articulación de las areas curriculares.pdfvictorbeltuce
 
Contextualización y aproximación al objeto de estudio de investigación cualit...
Contextualización y aproximación al objeto de estudio de investigación cualit...Contextualización y aproximación al objeto de estudio de investigación cualit...
Contextualización y aproximación al objeto de estudio de investigación cualit...Angélica Soledad Vega Ramírez
 
Tarea 5_ Foro _Selección de herramientas digitales_Manuel.pdf
Tarea 5_ Foro _Selección de herramientas digitales_Manuel.pdfTarea 5_ Foro _Selección de herramientas digitales_Manuel.pdf
Tarea 5_ Foro _Selección de herramientas digitales_Manuel.pdfManuel Molina
 
Fichas de Matemática DE SEGUNDO DE SECUNDARIA.pdf
Fichas de Matemática DE SEGUNDO DE SECUNDARIA.pdfFichas de Matemática DE SEGUNDO DE SECUNDARIA.pdf
Fichas de Matemática DE SEGUNDO DE SECUNDARIA.pdfssuser50d1252
 
PROGRAMACION ANUAL DE MATEMATICA 2024.docx
PROGRAMACION ANUAL DE MATEMATICA 2024.docxPROGRAMACION ANUAL DE MATEMATICA 2024.docx
PROGRAMACION ANUAL DE MATEMATICA 2024.docxEribertoPerezRamirez
 
Estas son las escuelas y colegios que tendrán modalidad no presencial este lu...
Estas son las escuelas y colegios que tendrán modalidad no presencial este lu...Estas son las escuelas y colegios que tendrán modalidad no presencial este lu...
Estas son las escuelas y colegios que tendrán modalidad no presencial este lu...fcastellanos3
 
c3.hu3.p1.p2.El ser humano y el sentido de su existencia.pptx
c3.hu3.p1.p2.El ser humano y el sentido de su existencia.pptxc3.hu3.p1.p2.El ser humano y el sentido de su existencia.pptx
c3.hu3.p1.p2.El ser humano y el sentido de su existencia.pptxMartín Ramírez
 
PLAN DE TUTORIA- PARA NIVEL PRIMARIA CUARTO GRADO
PLAN DE TUTORIA- PARA NIVEL PRIMARIA CUARTO GRADOPLAN DE TUTORIA- PARA NIVEL PRIMARIA CUARTO GRADO
PLAN DE TUTORIA- PARA NIVEL PRIMARIA CUARTO GRADOMARIBEL DIAZ
 
05 Fenomenos fisicos y quimicos de la materia.pdf
05 Fenomenos fisicos y quimicos de la materia.pdf05 Fenomenos fisicos y quimicos de la materia.pdf
05 Fenomenos fisicos y quimicos de la materia.pdfRAMON EUSTAQUIO CARO BAYONA
 
GUIA DE TEXTOS EDUCATIVOS SANTILLANA PARA SECUNDARIA
GUIA DE TEXTOS EDUCATIVOS SANTILLANA PARA SECUNDARIAGUIA DE TEXTOS EDUCATIVOS SANTILLANA PARA SECUNDARIA
GUIA DE TEXTOS EDUCATIVOS SANTILLANA PARA SECUNDARIAELIASPELAEZSARMIENTO1
 
describimos como son afectados las regiones naturales del peru por la ola de ...
describimos como son afectados las regiones naturales del peru por la ola de ...describimos como son afectados las regiones naturales del peru por la ola de ...
describimos como son afectados las regiones naturales del peru por la ola de ...DavidBautistaFlores1
 
Fisiologia.Articular. 3 Kapandji.6a.Ed.pdf
Fisiologia.Articular. 3 Kapandji.6a.Ed.pdfFisiologia.Articular. 3 Kapandji.6a.Ed.pdf
Fisiologia.Articular. 3 Kapandji.6a.Ed.pdfcoloncopias5
 
Fichas de MatemáticA QUINTO DE SECUNDARIA).pdf
Fichas de MatemáticA QUINTO DE SECUNDARIA).pdfFichas de MatemáticA QUINTO DE SECUNDARIA).pdf
Fichas de MatemáticA QUINTO DE SECUNDARIA).pdfssuser50d1252
 
III SEGUNDO CICLO PLAN DE TUTORÍA 2024.docx
III SEGUNDO CICLO PLAN DE TUTORÍA 2024.docxIII SEGUNDO CICLO PLAN DE TUTORÍA 2024.docx
III SEGUNDO CICLO PLAN DE TUTORÍA 2024.docxMaritza438836
 

Último (20)

Aedes aegypti + Intro to Coquies EE.pptx
Aedes aegypti + Intro to Coquies EE.pptxAedes aegypti + Intro to Coquies EE.pptx
Aedes aegypti + Intro to Coquies EE.pptx
 
3. Pedagogía de la Educación: Como objeto de la didáctica.ppsx
3. Pedagogía de la Educación: Como objeto de la didáctica.ppsx3. Pedagogía de la Educación: Como objeto de la didáctica.ppsx
3. Pedagogía de la Educación: Como objeto de la didáctica.ppsx
 
Presentacion minimalista aesthetic simple beige_20240415_224856_0000.pdf
Presentacion minimalista aesthetic simple beige_20240415_224856_0000.pdfPresentacion minimalista aesthetic simple beige_20240415_224856_0000.pdf
Presentacion minimalista aesthetic simple beige_20240415_224856_0000.pdf
 
IV SES LUN 15 TUTO CUIDO MI MENTE CUIDANDO MI CUERPO YESSENIA 933623393 NUEV...
IV SES LUN 15 TUTO CUIDO MI MENTE CUIDANDO MI CUERPO  YESSENIA 933623393 NUEV...IV SES LUN 15 TUTO CUIDO MI MENTE CUIDANDO MI CUERPO  YESSENIA 933623393 NUEV...
IV SES LUN 15 TUTO CUIDO MI MENTE CUIDANDO MI CUERPO YESSENIA 933623393 NUEV...
 
TL/CNL – 2.ª FASE .
TL/CNL – 2.ª FASE                       .TL/CNL – 2.ª FASE                       .
TL/CNL – 2.ª FASE .
 
Mapa Mental de estrategias de articulación de las areas curriculares.pdf
Mapa Mental de estrategias de articulación de las areas curriculares.pdfMapa Mental de estrategias de articulación de las areas curriculares.pdf
Mapa Mental de estrategias de articulación de las areas curriculares.pdf
 
Contextualización y aproximación al objeto de estudio de investigación cualit...
Contextualización y aproximación al objeto de estudio de investigación cualit...Contextualización y aproximación al objeto de estudio de investigación cualit...
Contextualización y aproximación al objeto de estudio de investigación cualit...
 
Tarea 5_ Foro _Selección de herramientas digitales_Manuel.pdf
Tarea 5_ Foro _Selección de herramientas digitales_Manuel.pdfTarea 5_ Foro _Selección de herramientas digitales_Manuel.pdf
Tarea 5_ Foro _Selección de herramientas digitales_Manuel.pdf
 
Fichas de Matemática DE SEGUNDO DE SECUNDARIA.pdf
Fichas de Matemática DE SEGUNDO DE SECUNDARIA.pdfFichas de Matemática DE SEGUNDO DE SECUNDARIA.pdf
Fichas de Matemática DE SEGUNDO DE SECUNDARIA.pdf
 
Aedes aegypti + Intro to Coquies EE.pptx
Aedes aegypti + Intro to Coquies EE.pptxAedes aegypti + Intro to Coquies EE.pptx
Aedes aegypti + Intro to Coquies EE.pptx
 
PROGRAMACION ANUAL DE MATEMATICA 2024.docx
PROGRAMACION ANUAL DE MATEMATICA 2024.docxPROGRAMACION ANUAL DE MATEMATICA 2024.docx
PROGRAMACION ANUAL DE MATEMATICA 2024.docx
 
Estas son las escuelas y colegios que tendrán modalidad no presencial este lu...
Estas son las escuelas y colegios que tendrán modalidad no presencial este lu...Estas son las escuelas y colegios que tendrán modalidad no presencial este lu...
Estas son las escuelas y colegios que tendrán modalidad no presencial este lu...
 
c3.hu3.p1.p2.El ser humano y el sentido de su existencia.pptx
c3.hu3.p1.p2.El ser humano y el sentido de su existencia.pptxc3.hu3.p1.p2.El ser humano y el sentido de su existencia.pptx
c3.hu3.p1.p2.El ser humano y el sentido de su existencia.pptx
 
PLAN DE TUTORIA- PARA NIVEL PRIMARIA CUARTO GRADO
PLAN DE TUTORIA- PARA NIVEL PRIMARIA CUARTO GRADOPLAN DE TUTORIA- PARA NIVEL PRIMARIA CUARTO GRADO
PLAN DE TUTORIA- PARA NIVEL PRIMARIA CUARTO GRADO
 
05 Fenomenos fisicos y quimicos de la materia.pdf
05 Fenomenos fisicos y quimicos de la materia.pdf05 Fenomenos fisicos y quimicos de la materia.pdf
05 Fenomenos fisicos y quimicos de la materia.pdf
 
GUIA DE TEXTOS EDUCATIVOS SANTILLANA PARA SECUNDARIA
GUIA DE TEXTOS EDUCATIVOS SANTILLANA PARA SECUNDARIAGUIA DE TEXTOS EDUCATIVOS SANTILLANA PARA SECUNDARIA
GUIA DE TEXTOS EDUCATIVOS SANTILLANA PARA SECUNDARIA
 
describimos como son afectados las regiones naturales del peru por la ola de ...
describimos como son afectados las regiones naturales del peru por la ola de ...describimos como son afectados las regiones naturales del peru por la ola de ...
describimos como son afectados las regiones naturales del peru por la ola de ...
 
Fisiologia.Articular. 3 Kapandji.6a.Ed.pdf
Fisiologia.Articular. 3 Kapandji.6a.Ed.pdfFisiologia.Articular. 3 Kapandji.6a.Ed.pdf
Fisiologia.Articular. 3 Kapandji.6a.Ed.pdf
 
Fichas de MatemáticA QUINTO DE SECUNDARIA).pdf
Fichas de MatemáticA QUINTO DE SECUNDARIA).pdfFichas de MatemáticA QUINTO DE SECUNDARIA).pdf
Fichas de MatemáticA QUINTO DE SECUNDARIA).pdf
 
III SEGUNDO CICLO PLAN DE TUTORÍA 2024.docx
III SEGUNDO CICLO PLAN DE TUTORÍA 2024.docxIII SEGUNDO CICLO PLAN DE TUTORÍA 2024.docx
III SEGUNDO CICLO PLAN DE TUTORÍA 2024.docx
 

Solucionario guia de admision 2015

  • 1. MINISTERIO DE EDUCACION CONSEJO NACIONAL DE UNIVERSIDADES UNAN-MANAGUA UNI UNAN-LEON Solucionario de Guía de Estudio de Matemática Agosto, 2014
  • 2. UNIDAD DE ARITMÉTICA 1. La expresión 311 + 311 + 311 equivale a: Solución : Al sumar los tres términos se obtiene 3 311 = 312 2. Al número de tres dígitos 2a3 se le suma el número 326 y da el número de tres dígitos 5b9. Si sabemos que el número 5b9 es divisible entre 9, entonces a + b es: Solución : Al sumar ambos números se obtiene 2a3 + 326 = 5b9 como el número 5b9 es divisible entre 9; esto signi…ca que la suma de los valores absolutos de sus cifras es múltiplo de 9, entonces 5 + b + 9 = 18; de aqui b = 4; entonces a + 2 = b; lo cual signi…ca que a = 2 y por tanto a + b = 6: 3. A una determinada cantidad le sumo el 10% de sí misma y a la cantidad así obtenida le resto su 10%. ¿Qué porcentaje de la cantidad original me queda? Solución : Sea x = Cantidad Inicial , entonces x + 0:1x = 1:1x es la cantidad aumentada en un 10%, pero a ésta le restamo su 10% y obtenemos 1:1x 0:1 (1:1x) = 0:99x lo cual representa un 99% de la cantidad inicial. 4. Al simpli…car [(9 4) + ( 10 + 3)] ((6) ( 5)) [( 12 + 8) (6 9) (95 90)] el resultado es: Solución : Al efectuar las operaciones indicadas se tiene [(9 4) + ( 10 + 3)] ((6) ( 5)) [( 12 + 8) (6 9) (95 90)] = [5 + ( 7)] ( 30) [( 4) ( 3) (5)] = ( 2) ( 30) 60 = 60 60 = 1 2
  • 3. 5. ¿Cuántos divisores diferentes tiene el número 2000? Solución : La descomposición del 2000 en factores primos es 2000 = 24 53 ; sumando 1 a cada exponente y multiplicando dichas expresiones, la cantidad de divisores será (4 + 1) (3 + 1) = 20 6. Al simpli…car 4 (3) 2 6 3 p 4 + 2 [5 (7) 15 3] 4 12 9. El resultado es: Solución : Al efectuar las operaciones indicadas y respetando el orden de prioridad de los operadores aritméticos, se tiene 4 (3) 2 6 3 p 4 + 2 [5 (7) 15 3] 4 12 9 = 36 6 6 + 2 [35 15 3] 4 12 9 = 6 6 + 2 [35 5] 4 12 9 = 60 4 12 9 = 240 12 9 = 20 9 = 11 7. Simpli…que 1 2 5 3 3 4 3 4 3 5 6 17 1 Solución: 1 2 5 3 3 4 3 4 3 5 6 17 1 = 1 2 5 4 3 10 9 17 1 = 3 4 17 9 17 1 = 27 68 17 1 = 27 4 1 = 7 3 4 8. ¿Cuántos números válidos (números que no tienen al cero como primer dígito) de cinco cifras se pueden escribir usando solo los dígitos 0; 1; 2; 3 y 4? Solución : El número 0 no puede ser el primer dígito, entonces, los otros lugares pueden ser ocupados por cualquieras de los 5 dígitos restantes, es decir, 4 5 5 5 5 = 4 54 3
  • 4. 9. Pedro tiene 69 años y su edad excede a la de Juan en un 15%. ¿Qué edad tiene Juan? Solución : Una de la formas de resolver este problema es 69 ! 115% x ! 100% , de aqui x = 69 100% 115% = 60 10. En una ciudad, 2 3 de los hombres están casados con los 3 5 de las mujeres. Si nunca se casan con forasteros, ¿Cuál es la proporción de solteros en dicha ciudad? Solución : Sea x = Cantidad de Hombres y = Cantidad de Mujeres 2 3 x = 3 5 y ; de aqui, y = 10 9 x x + 10 9 x 2 2 3 x x + 10 9 x = 7 19 11. El resultado de 125 2 3 + 16 1 2 + 343 1 3 1 2 es: Solución: 125 2 3 + 16 1 2 + 343 1 3 1 2 = 53 2 3 + 24 1 2 + 73 1 3 1 2 = 52 + 22 + 7 1 2 = [36] 1 2 = 6 12. Obtenga el resultado de (0:027) 1 3 + 2560:75 3 1 + (4:5) 0 Solución: (0:027) 1 3 + 2560:75 3 1 + (4:5) 0 = 3 10 3 1 3 + 28 3 4 1 3 + 1 = 3 10 1 + 26 1 3 + 1 = 10 3 + 64 1 3 + 1 = 68 13. ¿Cuál es el valor de a en (3a) 5 = 248832? Solución: (3a) 5 = 248832 35 a5 = 210 35 a5 = 210 35 35 a = 22 a = 4 4
  • 5. 14. Un equipo de jugadores ganó 15 juegos y perdió 5. ¿Cuál es la razón geométrica de los juegos ganados a los jugados? Solución : Total de juegos = 20, Total de juegos ganados =15, dicha proporción es 15 20 = 3 4 15. Si x es un número par y y es un número impar. ¿Cuál.de las siguientes a…rmaciones siempre es falsa? Solución: La falsa es y+y 2 = 2y 2 = y es par, porque aqui se produce una contradicción, y no puede ser par e impar a la vez. 16. El mínimo común múltiplo de dos números es 105 y su máximo común divisor es 5. ¿Cuál de los siguientes números puede representar la suma de estos dos números? Solución : Como su m.c.d es 5, signi…ca que 5 es el único divisor común. Por tanto, se trata de dos números múltiplos de 5. Como su m.c.m. es 105, entonces 105 5 = 21. Descomponemos el 21 en el producto de dos divisores, esto es 3 y 7. Por tanto, uno de los números es 15 = 3 5, el otro es 35 = 5 7 , por tanto, su suma es 15 + 35 = 50 17. La maestra distribuyó la misma cantidad de dulces entre cada uno de 5 niños y se quedó tres para ella misma. No se acuerda cuántos dulces tenía, pero se acuerda que era un múltiplo de 6 entre 65 y 100. ¿Cuántos dulces tenía? Solución: Como se quedó con 3 dulces, el número inicial de dulces termina en 3 o en 8, pero como es un múltiplo de 6, es par, por lo que termina en 8. La única posibilidad es 78. 18. El resultado de 2 6 6 6 4 5 4 0 B B B @ 1 2 2 1 1 2 1 1 C C C A 3 7 7 7 5 es Solución : Al desarrollar la fracción se tiene 5 4 0 B B B @ 1 2 2 1 1 2 1 1 C C C A = 5 4 3 2 = 1 5
  • 6. 19. El resultado de 2 3 4 5 6 7 es: Solución : Al realizar operaciones básicas aritmética se tiene 2 3 4 5 7 6 = 2 3 14 15 = 4 15 20. Juan gasta el 20% de sus ingresos en el pago de impuestos y 20% del resto en el pago de la mensualidad de su casa. ¿Qué porcentaje de su ingreso gasta en el pago de su casa? Solución : El valor gastado en el pago de impuesto es 0:2x luego lo que le queda es x 0:2x = 0:8x por tanto, el pago de la mensualidad de la casa es 0:2 (0:8x) = 0:16x lo cual corresponde a un 16% 21. ¿Cuánto gano o pierdo si vendo por los 3 5 de los 7 2 del costo de un juguete que me ha costado C$40:00? Solución : Aplicando operaciones básicas aritméticas 3 5 7 2 40 = 84:0 luego se ha ganado 84 40 = 44:00 córdobas. 22. Cuatro personas juntaron sus ahorros para abrir un negocio aportando el 15%, 20%, 25% y 40%, respectiva- mente, del monto total. Si la menor de las aportaciones fue de C$9; 000, la mayor de las aportaciones fue de: Solución : La menor de la aportaciones equivale 0:15x = 9000 luego el monto total es x = 60; 000 La mayor de las aportaciones equivale 0:4 (60000) = 24; 000 6
  • 7. 23. De acuerdo al Reglamento de Admisión de una universidad, el puntaje total alcanzado por un estudiante está formado por el 70% de la nota obtenida en el Examen de Admisión y el 30% de su promedio de los dos últimos años de bachillerato. Si un estudiante alcanza un puntaje total de 81 y su promedio de los dos últimos años de bachillerato es 95, ¿qué puntaje obtuvo en el examen de admisión? Solución : Sea x la nota obtenida en el examen de admisión, entonces 0:7x + 0:3 (95) = 81 x = 75 24. Un grupo de amigas va de paseo y disponen de C$240:00 para la compra de sus pasajes. Si compran pasajes de C$30:00, les sobra dinero; pero si compran pasajes de C$40:00, les falta dinero. ¿Cuántas amigas van de paseo? Solución : Sea n la cantidad de amigas, entonces 30n < 240 40n > 240 y la solución de dicho sistema de ecuación se encuentra en el intervalo (6; 8), de aqui que la solución entera es n = 7: 25. En el parqueo de una cierta universidad, entre carros y motos hay 20 vehículos. Sabiendo que el número total de ruedas es 70. ¿Cuántos carros hay? Solución : Sean x la cantidad de carros y (20 x) la cantidad de motos respectivamente, entonces 4x + 2 (20 x) = 70 x = 15 por tanto, hay 15 carros y 5 motos. 26. Un estudiante de una cierta universidad proveniente del interior del país gasta la cuarta parte de su “mesada” en el alquiler de una habitación, la mitad en comida, la quinta parte en materiales educativos y el resto, C$ 100.00, en recreación. ¿Cuánto es la “mesada”de este estudiante? Solución : Sea x la cantidad de la mesada recibida, entonces x x 4 x 2 x 5 = 100 x = 2000 7
  • 8. 27. El hielo disminuye su volumen en un 9% cuando se derrite. Si se derriten 1000cc de hielo, ¿Cuál es el volumen del líquido que se forma? Solución : Hay que obtener el 9% de 1000, es decir 0:09 1000 = 90cc por tanto, el volumen que se forma es de 1000cc 90cc = 910cc 28. ¿Cuál de las siguientes expresiones es impar para cualquier entero n? Solución : La expresión 2n2 es un número par y 2003 es un número impar, por tanto, su suma siempre será impar 29. El resultado de 2 6 6 6 4 5 4 0 B B B @ 1 2 2 1 1 2 1 1 C C C A 3 7 7 7 5 es Solución : Al desarrollar la fracción se tiene 5 4 0 B B B @ 1 2 2 1 1 2 1 1 C C C A = 5 4 3 2 = 1 30. Calcular el producto L H sabiendo que L = a + b + c , H = d + c = f + g siendo a; b; c; d; f; g números naturales y que b f = 91 ; a d = 18 ; c d = 16 ; b g = 39 Solución : Como sabemos que b f = 91 ; a d = 18 ; c d = 16 ; b g = 39; podemos aplicar la teoria de máximo común divisor y obtenemos : b = gcd (39; 91) = 13 , d = gcd (16; 18) = 2; de aqui f = 7; c = 8; a = 9; g = 3 y entonces L = a + b + c , H = d + c = f + g; y sustituyendo L = 9 + 13 + 8 = 30; H = 2 + 8 = 7 + 3 = 10; por tanto el producto es 300: 31. Al desarrollar la expresión qpp 625a8 2 el resultado es: Solución: qpp 625a8 2 = h 54 a8 1 8 i2 = h 5 1 2 a i2 = 5a2 8
  • 9. 32. El resultado de q a 3 p a p a es: Solución: q a 3 p a p a = q a 3 pp a a2 = q 3 p a3 p a3 = q 3 pp a9 = 12 p a9 = 4 p a3 33. Una epidemia mató los 5 8 de las reses de un ganadero y luego él vendió los 2 3 de las que le quedaban. Si aún tiene 216 reses, ¿Cuántas tenía al principio, cuántas murieron y cuántas vendió? Solución : Formamos una ecuación lineal x 5 8 x 2 3 x 5 8 x = 216, cuya solución es x = 1728; este valor son las reses que tiene al inicio, las que mata la epidemia son 5 8 (1728) = 1080; las que le quedan son 1728 1080 = 648 y las vende son 2 3 (648) = 432 34. Una gallina pone dos huevos en tres días. ¿Cuántos días se necesitan para que cuatro gallinas pongan dos docenas de huevos? Solución : Este es un problema de proporcionalidad compuesta, Gallinas Huevos Dias 1 2 3 4 24 x De aqui 4 1 2 24 = 3 x ; x = 9 35. El 41 2 3 % es equivalente a: Solución: Usando una regla de tres simple: 100% ! 1 125 3 % ! x Tenemos que equivale a 5 12 9
  • 10. 36. Halla el número cuyo 3:6 porciento vale 3 + 4:2 0:1 1 0:3 2 1 3 0:3125 Solución : Llamamos N al número buscado y A a la expresión dada. Entonces: A = (3:6 N) 100 ; de aqui N = A 100 3:6 ; haciendo las operaciones respectivas, se obtiene que 3 + 4:2 0:1 1 0:3 2 1 3 0:3125 100 36 = 4000 37. Al realizar la operación 4:62 10 2 2:2 10 4 se obtiene el número Solución : Al realizar la división indicada 4:62 2:2 102 = 210 38. Un albañil y su ayudante pueden hacer una obra en 24 días. Después de 4 días de trabajo, el ayudante se retira y el albañil termina lo que falta en 30 días. El número de días que podría hacer la obra el ayudante trabajando solo es: Solución: Al plantear una regla de tres compuesta Hombres Dias Proyectado Dias Reales 2 24 20 1 x 30 De aqui x = 2 24 30 1 20 = 72 39. Al simpli…car la expresión 21 + 20 + 2 1 2 2 + 2 3 + 2 4 se obtiene: Solución: Al reescribir la expresión dada 2 + 1 + 1 2 1 4 + 1 8 + 1 16 y al efectuar operaciones básicas de suma y cociente, se tiene que el valor dado es 8 10
  • 11. 40. Se va a tender una línea eléctrica de 35:75km de longitud con postes separados entre sí por una distancia de 125m. Si el primer poste se coloca al inicio de la línea, y el último al …nal ¿cuántos postes serán necesarios en total? Solución: Al hacer la conversión de 35:75km a metros se tiene 35:75 1000 = 35750 lo cual a dividir entre 125; se tendría la cantidad de poste utilizado, es decir 35750 125 = 286 pero como el primer poste se coloca al inicio de la línea, se tiene que el total de poste es de 287: 41. La operación está de…nida por a b = 2ab 3b en la que a y b son números enteros. ¿Cuál es el resultado de [4 ( 1)] ( 3)? Solución: Realizando las operaciones por partes: [4 ( 1)] = 2 (4) ( 1) 3 ( 1) = 8 + 3 [( 5) ( 3)] = 2 ( 5) ( 3) 3 ( 3) = 30 + 9 = 39 42. ¿Cuál es la diferencia entre el 50% de 50 y el 20% de 20? Solución: Calculemos los porcentajes dados 0:5 (50) = 25 0:2 (20) = 4 por tanto, la diferencia dada es 21 43. En la sustracción a b = c, la suma del minuendo, el sustraendo y la diferencia es 32. ¿Cuál es el valor del minuendo? Solución: Sabemos que a + b + c = 32 pero a b = c; entonces a + b + a b = 32 a = 16 11
  • 12. 44. El resultado de la operación 2 2 5 4 5 + 3 1 3 4 3 4 1 4 1 2 + 5 1 5 24 7 20 11 2 es: Solución: 2 2 5 4 5 + 3 1 3 4 3 4 1 4 1 2 + 5 1 5 24 7 20 11 2 = 2 + 2 15 2 + 1 5 77 40 = 4 77 10 77 40 = 1 45. El valor numérico de la expresión 42 (3 2) 2 ( 6 + 1) 2 es: Solución: Al desarrollar la expresión dada 42 (3 2) 2 ( 6 + 1) 2 = 16 1 25 = 3 5 46. Si A comió 1 4 de un queque, B comió 1 3 de lo que quedó después que A comió; C comió 1 2 de lo que quedó después que A y B comieron ¿Qué parte del queque quedó? Solución : Sea x el total del queque, entonces al restar las partes que se comieron, se tiene x 1 4 x 1 3 x 1 4 x 1 2 x 1 4 x 1 3 x 1 4 x 1 4 x 47. Con los 2 7 del dinero que tenía, Mara compró gaseosas para festejar su cumpleaños. Con los 3 5 del dinero que le sobró compró hamburguesas. Al …nal Mara se quedó con C$100:00. ¿Cuánto gastó Mara en hamburguesas? Solución : Al aplicar los datos x 2 7 x 3 5 x 2 7 x = 100 x = 350 12
  • 13. lo gastado en hamburguesa es 3 5 x 2 7 x = 3 5 350 2 7 350 = 150 48. En una fábrica 60% de los artículos son producidos por una máquina A y el resto por otra máquina B. Si 3% de los artículos producidos por la máquina A y 8% de los producidos por la máquina B resultaron defectuosos ¿cuál es el porcentaje de artículos defectuosos producidos en toda la fábrica. Solución : Sea x el total de artículos producidos por la máquina A, entonces según los datos 0:6x + 0:4 (x 0:6x) = 100 49. La última vez que llené el tanque de gasolina, mi automóvil había recorrido 47; 286km. Ahora que acabo de llenarlo, la bomba marcó 22 litros y el cuentakilómetros marcaba 47; 506 km recorridos. Si el litro de gasolina cuesta C$20. ¿Cuánto me cuesta en promedio recorrer un kilómetro? Solución : Haciendo la diferencia 47506 47286 = 220 el promedio en kilometraje es 220 20 = 11 50. Un frasco contiene 12 onzas de una solución cuya composición es una parte de ácido por cada 2 partes de agua. Se agrega a otro frasco que contiene 8 onzas de una solución que contiene 1 parte de ácido por cada 3 partes de agua. ¿Cuál es la razón entre el ácido y el agua de la solución obtenida? Solución : La relación en el frasco de 12 onzas es 4 8 y en el frasco de 8 onzas es 2 6 ; entonces la relación total entre el ácido y el agua es 6 14 = 3 7 51. Por un préstamo de 20; 000 pesos se paga al cabo de un año 22; 400 pesos. ¿Cuál es la tasa de interés cobrada? Solución : La fórmula dada es F = P (1 + i) t entonces 22400 = 20000 (1 + i) i = 0:12 lo cual representa 12% 13
  • 14. 52. Si un número N se divide entre 4, se obtiene 9 de cociente y 1 de residuo. Si N se divide entre M, se obtiene 5 de cociente y 2 de residuo. ¿Cuál es el valor de M? Solución : De acuerdo a los datos del problema N = cd + R = 36 + 1 = 37 N = 5M + 2 al sustituir los datos 37 2 5 = M M = 7 53. Un contratista compró 4000 piedras y las vendió por 8,800 córdobas. ¿Cuánto pagó el por cada piedra si ganó, en relación a lo que pagó, un porcentaje igual a 5 veces el número de córdobas que a él le costó cada piedra? Solución : El costo real de cada piedra es 8800 4000 = x + 5x x 100 2:2 = x + x2 20 x = 2 54. El valor de la expresión 1 2 2 + ( 2) 2 ( 2) 3 es: Solución : Al reescribir la expresión dada 1 2 2 + ( 2) 2 ( 2) 3 = 22 + 4 8 = 1 55. Calcular a cuánto asciende el interés simple producido por un capital de 25 000 córdobas invertido durante 4 años a una tasa del 6 % anual. Solución : La fórmula a utilizar es I = C i t = 25000 0:06 4 = 6; 000 14
  • 15. 56. En el año 1982 la edad de la tierra era de 1:3 1017 segundos y la de la pirámide de Keops, 1:5 1011 segundos. La diferencia de edad entre la tierra y la pirámide en notación cientí…ca es: Solución : Sea d la diferencia de edad, entonces d = 1:3 1017 1:5 1011 d = 1011 1:3 106 1:5 d = 1011 (1299998:5) d = 1:2999985 1017 seg 57. La luz recorre aproximadamente 3 105 km por segundo. ¿Cuántos metros recorrerá en 365 días? El resultado en notación cientí…ca es: Solución: En un día hay 24 (60) (60) = 86; 400seg, en 365 días hay 365 (86; 400) = 31; 536; 000seg. Como la luz recorre 3 105 103 m, en esos segundos la luz recorrerá: 31; 536; 000 3 108 m = 94; 608 1011 m = 9:4608 1015 m 58. La velocidad de la luz es aproximadamente de 3 105 km=seg: La estrella más cercana a la tierra está a 4300 años luz de distancia. La distancia en km y escrita en notación cientí…ca es: Solución: En un día hay 24 (60) (60) = 86; 400seg, en 365 días hay 365 (86; 400) = 31; 536; 000seg. Como la luz recorre 3 105 103 m, en esos segundos la luz recorrerá 31; 536; 000 3 108 m = 94; 608 1011 m, entonces: 31; 536; 000 3 108 m = 94; 608 1011 m = 9:4608 1012 m=seg La estrella más cercana está a 4300 AL, entonces 4300 9:4608 1012 = 40681:44 1012 = 4:068144 104 1012 = 4:068144 1016 km 59. ¿Qué altura tendría una pila de 1; 000; 000 de hojas de cuaderno si se necesitan 10 hojas para tener 1mm? Solución: Utilizando una regla de tres simple: 15
  • 16. 1; 000; 000 ! x 10 ! 1mm Entonces, la altura x de la pila es 1; 000; 000 10 = 100; 000mm = 105 mm. 60. ¿Cuántos rieles de 15m se necesitan para enlazar a una fábrica con la estación que dista 765m? Solución: Se necesitan 765m 15m = 51 rieles. 61. ¿Cuántos al…leres de 3:5cm de largo pueden fabricarse con un alambre de latón de 152:07m, sabiendo que hay una pérdida de 2mm de alambre por al…ler? Solución: En total hay 152:07m = 152:07 100 = 15; 207cm de alambre y se pierde 2mm = 2 10 = 0:2cm de alambre por cada al…ler. Entonces, si x representa la cantidad de al…leres que pueden fabricarse: 15207 = (3:5 + 0:2) x 15207 3:7 = x 4110 = x Se pueden fabricar 4,110 al…leres. 62. Para ir a clase, Pedro tiene que andar por término medio 1; 520 pasos de 62 cm. ¿Cuántos km habrá recorrido durante un año escolar de 210 días si va al colegio y vuelve a su casa? Solución: De su casa a la escuela (y viceversa) recorre 1; 520 0:62 = 942:40m, al día recorre 2 942:40m = 1884:8m = 1884:8 1000 = 1:8848km: Durante el año habrá recorrido 210 1:8848 = 395:8km. 63. Se ha necesitado 54; 000 losetas para pavimentar los 2; 430 m2 que miden las aceras de una calle. ¿Cuál es en mm2 la super…cie de una loseta? Solución: La super…cie de cada loseta es de 2; 430 m2 54; 000 = 0:045m2 . Como 1m2 = 1; 000; 000mm2 . Entonces: 0:045 1; 000; 000mm2 = 45; 000mm2 16
  • 17. 64. Si el m2 de un terreno vale 2 dolar, ¿Cuántos dólares vale comprar un campo de 7 Ha? Solución: Como 7Ha = 7 10; 000m2 = 70; 000m2 , entonces comprar el campo cuesta 70; 000 $2 = $140; 000. 65. La isla mayor de la Tierra es Groenlandia y mide 2; 180; 000 km2 y una de las más pequeñas es Cabrera, con 2000 Ha. ¿Cuántas veces cabe Cabrera en Groenlandia? Solución: Como 2; 180; 000 km2 = 2; 180; 000 100 = 218; 000; 000Ha. Entonces la isla Cabrera cabe 218; 000; 000Ha 2000 Ha = 109; 000 veces en Groelandia. 66. Una tinaja que contiene 0; 4 m3 de aceite ha costado 800 euros ¿a cuántos euros resulta el litro? Solución: Como 0; 4m3 = 0; 4 1000 = 400l, el precio del aceite por litro es 800 400 = 2 euros. 67. Un caramelo tiene un volumen de 1; 3 cm3 . ¿Cuántos caramelos caben en una caja de 0; 4498 dm3 ? Solución: Como 0; 4498 dm3 = 0; 4498 1000cm3 = 449; 8cm3 . En la caja caben 449; 8 1; 3 = 346 caramelos. 68. Los trozos cúbicos de jabón de 5 cm de arista se envían en cajas cúbicas de 60 cm de arista. ¿Cuántos trozos puede contener la caja? Solución: El volumen de los trozos de jabón es (5cm) 3 = 125cm3 y el volumen de cada caja es (60cm) 3 = 216; 000cm3 . Entonces cada caja puede contener 216; 000 125 = 1728 trozos de jabón. 69. ¿Cuántas botellas de 750 cm3 se necesitan para envasar 300 litros de refresco. Solución: Como 750 cm3 = 750 1000 = 0:75 litros, entonces se necesitan 300 0:75 = 400 botellas para envasar 300 litros de refresco. 70. La capacidad de un depósito de gasolina es 1500 litros. ¿Cuál es su volumen en cm3 ? Solución: El volumen del depósito es de 1500 1000 = 150; 000cm3 . 17
  • 18. 71. Un camión transporta 50 cajas con botellas llenas de agua. Cada caja contiene 20 botellas de litro y medio. Una caja vacía pesa 1500 g, y una botella vacía, 50 g. ¿Cuál es el peso total de la carga? Solución: Cajas vacías: 50 1500g = 75; 000g Botellas vacías: 50 20 50g = 50; 000g Cajas llenas : 50 20 1:5l = 1500l = 1500 1000g = 1; 500; 000g El peso total de la carga es de 75; 000g + 50; 000g + 1; 500; 000g = 1; 625; 000g = 1; 625; 000 1000 = 1; 625kg: 72. Si para construir un muro necesito 2 toneladas de cemento, ¿cuántos sacos de 25 kilos de cemento tendré que comprar? Solución: Como 2 toneladas = 2 1000 = 2; 000kg, se tienen que comprar 2; 000 25 = 80 sacos. 73. Un barco transporta 2800 toneladas de mercancía. ¿Cuántos vagones harán falta para transportar esa mercancía si cada vagón carga 1400 kg? Solución: Como 2800 toneladas = 2800 1000 = 2; 800; 000kg, hacen falta 2; 800; 000 1; 400 = 2000 vagones. 74. La temperatura del cuerpo humano es 37 C. ¿A cuántos grados Fahrenheit equivalen? Solución: Para convertir grados celsius a Fahrenheit se utiliza la siguiente fórmula: o F = o C 9 5 + 32 En este caso: o F = 37 9 5 + 32 = 66:6 + 32 = 98:6o F 75. Para asar un pollo se necesita que el horno de la cocina alcance una temperatura de 374 F. ¿A qué temperatura debo …jar el graduador para asar el pollo, si la graduación está en grados centígrados ( C)? Solución: Para convertir grados Celsius a Fahrenheit se utiliza la siguiente fórmula: o C = (o F 32) 5 9 En este caso: o C = (374 32) 5 9 = 342 5 9 = 190o C 18
  • 19. UNIDAD DE ÁLGEBRA 1. Dado el polinomio lineal f(x) = x 1 2 ; la suma f(x) + f(x + 1 4 ) + f(x + 2 4 ) + f(x + 3 4 ) es igual a: Solución: Al evaluar el polinomio f (x) = x 1 2 ; se obtiene que f (x) = x 1 2 f x + 1 4 = x + 1 4 1 2 = x 1 4 f x + 2 4 = x + 2 4 1 2 = x f x + 3 4 = x + 3 4 1 2 = x + 1 4 de donde f (x) + f x + 1 4 + f x + 2 4 + f x + 3 4 = x 1 2 + x 1 4 + x + x + 1 4 = 4x 1 2 2. Si x + y = 1 y xy = 1 , ¿cuál será el valor de x3 + y3 ? Solución: Elevando al cubo la expresión (x + y) = 1, y aplicando las condiciones dadas en el ejercicio, se obtiene (x + y) 3 = x3 + 3x2 y + 3xy2 + y3 = 1 x3 + 3xy (x + y) + y3 = 1 x3 + 3 (1) (1) + y3 = 1 x3 + y3 = 2 3. Si a = 1; b = 3; c = 5, entonces a + b ja bj jaj + jbj + jcj es igual a: Solución: Haciendo las debidas sustituciones resulta 1 + 3 j 1 3j j 1j + j3j + j5j = 2 4 9 = 2 9 : 4. El valor numérico de la expresión a2 a + b2 a3 b3 a2 b (a2 + b2) (2a 3b2) para a = 1 y b = 2 es: Solución : Al sustituir los valores respectivos se obtiene (1) 2 1 + ( 2) 2 (1) 3 ( 2) 3 (1) 2 ( 2) (1) 2 + ( 2) 2 2 (1) 3 ( 2) 2 = 27 10 19
  • 20. 5. Las raíces de la ecuación ax2 + bx + c = 0 serán recíprocas si: Solución : Las raíces de la ecuación seran recíprocas si al multiplicarla el resultado es 1, de la fórmula general se puede ver que b + p b2 4ac 2a ! b p b2 4ac 2a ! = 1 b2 b2 4ac 4a2 = 1 de aquí, c = a 6. El resultado de (bn 5ym ) (5ym + bn ) es: Solución : El producto indicado es un producto notable y su resultado es (bn ) 2 (5ym ) 2 = b2n 25y2m 7. La descomposición en factores de la expresión 3x2 2x 8 es: Solución : Al factorizar dicha expresión se tiene 3x2 2x 8 = (3x + 4) (x 2) 8. La descomposición en factores de la expresión x3 64y3 es Solución : Al factorizar se tiene x3 64y3 = (x 4y) 4xy + x2 + 16y2 9. La simpli…cación de a2 4b2 ab + 2b2 3a2 5ab 2b2 3a2 + ab es Solución : Al factorizar los diferentes términos de las fracciones, se tiene a2 4b2 ab + 2b2 3a2 5ab 2b2 3a2 + ab = (a + 2b) (a 2b) b (a + 2b) (a 2b) (3a + b) a (3a + b) = (a + 2b) (a 2b) b (a + 2b) a (3a + b) (a 2b) (3a + b) = a b 20
  • 21. 10. Al simpli…car la expresión 1 a 1 p a 1 p a + 1 a se obtiene Solución : Al determinar el mínimo común de ambos denominadores p a a a p a a + p a a p a = p a a p a + a al racionalizar el denominador, obtenemos p a a p a + a p a a p a a = ( p a a) 2 a a2 = a1=2 1 a1=2 2 a (1 a) = (1 p a) 2 1 a 11. El resultado de la siguiente operación 1 x 1 + 12x2 4x 4x2 11x 3 3x2 + 8x 3 x2 9 es Solución : Al desarrollar las operaciones indicadas y factorizando, se tiene 1 x 1 + 4x (3x 1) (4x + 1) (x 3) (x + 3) (3x 1) (x 3) (x + 3) 1 x 1 + 4x (3x 1) (4x + 1) (x 3) (x 3) (x + 3) (x + 3) (3x 1) 1 x 1 + 4x 4x + 1 4x2 + 1 4x2 3x 1 4x2 + 1 (4x + 1) (x 1) 12. Al desarrollar x y y x 2 se obtiene Solución : Desarrollando el cuadrado x y y x 2 = x2 y2 xy 2 = x4 2x2 y2 + y4 x2y2 21
  • 22. 13. Al racionalizar el denominador de la fracción x 2 3 + p 2x + 5 se obtiene Solución : Al multiplicar por su conjugado x 2 3 + p 2x + 5 3 p 2x + 5 3 p 2x + 5 = p 2x + 5 3 2 14. El conjunto solución de la ecuación 3x x 5 = 1 + 15 x 5 es Solución : Al multiplicar por el mínimo común denominador (x 5) 3x x 5 = (x 5) 1 + 15 x 5 3x = x 5 + 15 2x = 10 x = 5 15. El valor de k que proporciona sólo una solución real de la ecuación x2 + kx + k = 2 3x es: Solución : Una ecuación de segundo orden tiene una solución si el discriminante b2 4ac = 0; entonces, al reescribir dicha ecuación en la forma x2 + (k + 3) x + (k + 2) = 0 y al analizar su discriminante, se tiene (k + 3) 2 4 (1) (k + 2) = 0 y al resolver dicha ecuación, se tiene que k = 1 16. Al resolver el sistema de ecuaciones 8 >< >: 2 3x + y + 4 3x y = 3 2 3x + y 4 3x y = 1 , se obtiene que el valor de la variable y es: Solución : Al sumar ambas ecuaciones, 4 3x + y = 4 3x + y = 1 y = 1 3x sustituyendo este valor en la primera ecuación, obtenemos 2 3x + 1 3x + 4 3x 1 + 3x = 3 4 6x 1 = 1 x = 6 6 y al sustituir en y = 1 3x; obtenemos y = 3 2 22
  • 23. 17. Al efectuar x2 4 (x 2) 2 + (x + 2) 2 x2 4 se obtiene : Solución : La expresión dada se puede reescribir por (x + 2) (x 2) (x 2) 2 + (x + 2) 2 (x + 2) (x 2) x + 2 x 2 + x + 2 x 2 2 (x + 2) x 2 18. Al resolver la ecuación x + 1 x 1 + 2x 1 x + 1 = 4 se obtiene que la diferencia entre la mayor y la menor de las raíces es : Solución : La expresión dada se puede reescribir por (x + 1) 2 + (2x 1) (x 1) x2 1 = 4 3x2 x + 2 = 4x2 4 x2 + x 6 = 0 al resolver dicha ecuación, se tiene que las soluciones reales son x1 = 3 ; x2 = 2; por tanto, la diferencia entre las raíces es 5 19. Al resolver el sistema de ecuaciones ( p 2x + p 3y 2 = 5 + 2 p 6xy 2x 3y = 1 , se obtiene que el valor de la variable y es: Solución : Podemos ver que 2x = 1 + 3y al sustituir en la ecuación dada, se tiene que p 1 + 3y + p 3y 2 = 5 + 2 p (1 + 3y) 3y p 1 + 3y + p 3y 2 = 5 + 2 p 3y (1 + 3y) desarrollando el cuadrado 1 + 3y + 2 p 3y (1 + 3y) + 3y = 5 + 2 p 3y (1 + 3y) 4 + 6y = 0 y = 2 3 23
  • 24. 20. El conjunto solución de la desigualdad x3 + x2 2x > 0 es : Solución : Al factorizar dicha expresión se tiene x (x + 2) (x 1) > 0 entonces los números críticos son x = 0; x = 2; x = 1 entonces Expresión x x + 2 x 1 Signo x < 2 2 < x < 0 + + 0 < x < 1 + + x > 1 + + + + por tanto, el conjunto solución está de…nido por los intervalos donde está el signo positivo, es decir ( 2; 0) [ (1; +1) 21. El valor de k de manera que la ecuación 2x2 + kx + 4 = 0 tenga una raíz igual a 3 es: Solución : Sabemos que la raices de la ecuación cuadrática tiene la forma b p b2 4ac 2a entonces, como una de las raices es 3; obtenemos k p k2 4 (2) (4) 2 (2) = 3 k = 22 3 22. El conjunto solución de la desigualdad jx + 2 3 j 2 es Solución : Aplicando propiedad de valor absoluto 2 x + 2 3 2 2 2 3 x 2 2 3 8 3 x 4 3 24
  • 25. 23. El conjunto solución de la desigualdad 1 7 x 2 3 es Solución : Aplicando propiedad de valor absoluto 1 7 x 2 3 2 7 x 6 2 7 x 6 7 multiplicando por 1 y cambiando el sentido de la desigualdad 5 x 1 1 x 5 en forma de intervalo se tiene [ 1; 5] 24. El conjunto solución de la desigualdad j5 2xj < 7, está dado por el intervalo Solución : Aplicando propiedad de valor absoluto 7 < 5 2x < 7 7 5 < 2x < 7 5 12 < 2x < 2 multiplicando por 1 2 y cambiando el sentido de la desigualdad 6 > x > 1 1 < x < 6 la cual se puede escribir en notación de intervalo por ( 1; 6) 25. El conjunto solución de la desigualdad (x + 10) (x 2) x2 7x 8 0 es Solución : Factorizando la expresión del denominador, obtenemos x2 7x 8 = (x + 1) (x 8) entonces los puntos criticos son x = 10 ; x = 1 ; x = 2 ; x = 8 25
  • 26. entonces Expresión x + 10 x 2 x + 1 x 8 Signo x 10 + 10 x < 1 + 1 < x 2 + + + 2 x < 8 + + + x > 8 + + + + + por tanto, el conjunto solución está de…nido por los intervalos donde está el signo negativo, es decir [ 10; 1) [ [2; 8) 26. El conjunto solución de la ecuación p 2x + 3 p x 2 = 2 es Solución : Elevando al cuadrado ambos miembros de la ecuación p 2x + 3 p x 2 2 = 4 2 p (2x + 3) (x 2) = 3 3x nuevamente elevando al cuadrado 2 p (2x + 3) (x 2) 2 = (3 3x) 2 8x2 4x 24 = 9x2 18x + 9 x2 14x + 33 = 0 al resolver dicha ecuación, se tiene que el conjunto solución es f3; 11g 27. Si j2x 1j > 3, el valor de x que no pertenece al conjunto solución es Solución : Por de…nición se obtiene 2x 1 > 3 o 2x 1 < 3 entonces 2x > 4 o 2x < 2 x > 2 o x < 1 por tanto, el conjunto solución es ( 1; 1) [ (2; 1) ; es decir x =2 [ 1; 2] ; lo cual es el valor x = 1 26
  • 27. 28. Si x + 1 x 2 = 3 entonces x3 + 1 x3 es igual a: Solución : Multiplicando por x + 1 x a la expresión x + 1 x 2 = 3 se obtiene que : x + 1 x x + 1 x 2 = 3 x + 1 x x + 1 x 3 = x3 + 3x2 1 x + 3x 1 x2 + 1 x3 = 3x + 3 x x3 + 1 x3 = 0 29. El conjunto solución de 3x + jxj = 8 es Solución : Aplicando la de…nición de valor absoluto, para x 0 3x + x = 8 x = 2 para x < 0 3x x = 8 x = 4 podemos notar que para x = 2 se tiene una solución extraña y por tanto, la única solución es x = 4 30. Al factorizar la expresión 12x3 + 36x2 27x uno de los factores es: Solución : Factorizando la expresión dada 12x3 + 36x2 27x = 3x (2x 3) 2 31. El resultado simpli…cado de 3y 2 4 p 8x3y7 1 3x 4 p 8x2y3, es: Solución : Al efectuar las operaciones indicadas, se tiene y 2x 4 p 82x5y10 = y 2x p 8x 4 p xy2 4 p y2 = y3 4 p 4xy2 27
  • 28. 32. Si x; y; z, son números positivos que satisfacen x + 1 y = 4 ; y + 1 z = 1 ; z + 1 x = 7 3 entonces el valor de xyz es: Solución : Reescribiendo las expresiones dadas xy + 1 = 4y xyz + z = 4yz xyz = 4yz z xz + 1 = 7 3 x xyz + y = 7 3 xy Igualando las expresiones y sabiendo que x = 4 1 y ; z = 7 3 1 x entonces 4yz z = z 4 1 y 4 1 y = 1 y (4y 1) (z 1) = 1 y (4y 1) 0 B B @ 7 3 1 4 1 y 1 1 C C A = 1 3y (13y 4) pero 1 3y (13y 4) = xyz 1 3y (13y 4) = 4 1 y y 0 B B @ 7 3 1 4 1 y 1 C C A y = 2 5 entonces x = 4 1 y = 3 2 z = 7 3 1 x = 5 3 de aqui xyz = 1 28
  • 29. 33. Si n > 1, entonces 3 q n 3 p n 3 p n es igual a: Solución : Aplicando la de…nición de radicales 3 r n 3 q n 3 p n = 3 q n 3 p nn1=3 = 3 q n 3 p n4=3 = 3 p nn4=9 = 3 p n13=9 = n13=27 34. La expresión n2p a:a33 :a53 ::::a(2n 1)3 es igual a: Solución : Al utilizar la igualdad 13 + 33 + ::: + (2n 1) 3 = n2 2n2 1 ; y observando que la cantidad subradical es un producto de potencia de la misma base y que dicha suma coincide con la dada en la sugerencia, podemos sustituir y obtener n2p an2(2n2 1) = a2n2 1 35. Si (x + y) 2 = 2 x2 + y2 el valor de E = 3x3 y3 x2y + 3x + y 5x + 6y 2x + y será: Solución: Al efectuar la suma tenemos que 3x3 y3 x2y + 3x + y 5x + 6y 2x + y = 30x4 + 21x3 y + 35x2 y2 9xy3 5y4 5 (2xy y2) y (2x + y) = 30x4 + 21x3 y + 35x2 y2 9xy3 5y4 20x2y2 5y4 Por otro lado, como (x + y) 2 = 2 x2 + y2 , resulta que x2 = 2xy y2 , así al sustituir obtenemos 3x3 y3 x2y + 3x + y 5x + 6y 2x + y = 30x4 + 21x3 y + 35x2 y2 9xy3 5y4 20x2y2 5y4 = 30 2xy y2 2 + 21 2xy y2 xy + 35 2xy y2 y2 9xy3 5y4 20 (2xy y2) y2 5y4 = 162x2 y2 80xy3 10y4 20 (2xy y2) y2 5y4 = 162 2xy y2 y2 80xy3 10y4 20 (2xy y2) y2 5y4 = 4y3 (43y 61x) 5y3 (5y 8x) = 172y 244x 25y 40x 29
  • 30. 36. Si el polinomio P (x) = x4 + ax3 bx2 + cx 1 es divisible por (x 1) (x + 1) (x 1) ; el valor de (a + b + c) 2 es: Solución El polinomio (x 1) (x + 1) (x 1) es igual a x3 x2 x+1. Al dividir P (x) entre x3 x2 x+1, el residuo es (a b + 2) x2 +(a + c) x+( a 2) y el cociente x+(a + 1). Pero P (x) es divisible entre (x 1) (x + 1) (x 1) así que el residuo debe ser cero, es decir, (a b + 2) x2 +(a + c) x+( a 2), y para que esto ocurra a b+2 = 0, a + c = 0, a 2 = 0. De esto resulta que a = 2; c = 2 y b = 0. Por lo tanto, (a + b + c) 2 = ( 2 + 0 + 2) 2 = 02 = 0 37. Sabiendo que x + 1 x = 3;al determinar el valor de E = x3 + x2 + 1 x3 + 1 x2 obtenemos: Solución Se tiene que x + 1 x 2 = x2 + 1 x2 + 2 por lo cual al sustituir x + 1 x = 3, resulta que x2 + 1 x2 = 32 2 = 7: Similarmente, como x + 1 x 3 = x3 + 1 x3 + 3 x + 1 x = x3 + 1 x3 + 9 se obtiene que x3 + 1 x3 = 33 9 = 18. Por tanto, E = 7 + 18 = 25 38. Si el cociente notable x30 ym xn y2 tiene 10 términos, entonces el valor de (m + n) es: Solución En un cociente notable, para hallar el número de términos que va a tener la solución de la división, por ejemplo de: xp yq xr ys se calcula como la división de los exponentes de la misma variable: n = p r = q s Así pues en este caso 10 = 30 n = m 2 de lo cual se deduce que n = 3 y m = 20; luego m + n = 23 30
  • 31. 39. Si 264 = aa y p 3 54 = (3b) b ; al determinar el valor de 3a + b se obtiene: Solución Tenemos que aa = 264 = 24 16 = (16) 16 , así que a = 16. Por otro lado, (3b) b = p 3 54 = (3) 27 = 33 9 = (3 9) 9 ; lo cual indica que b = 9. Luego, 3a + b = 48 + 9 = 57 40. Si (2a + b) c = 1 5 ; entonces el valor de b2 + 4ab + 4a2 c es: Solución Se tiene que b2 + 4ab + 4a2 c = h (2a + b) 2 ic = [(2a + b) c ] 2 Como el inverso multiplicativo de (2a + b) c es 1 5 , signi…ca que (2a + b) c = 5, resultando que b2 + 4ab + 4a2 c = [(2a + b) c ] 2 = 52 = 25 41. Sabiendo que a + b + c = 0; ab + ac + bc = 7 y abc = 6 entonces el valor de 1 a2 + 1 b2 + 1 c2 es: Solución Tenemos que 1 a + 1 b + 1 c 2 = 2 ab + 2 ac + 2 bc + 1 a2 + 1 b2 + 1 c2 bc + ac + ab abc 2 = 2 (c + b + a) abc + 1 a2 + 1 b2 + 1 c2 1 a2 + 1 b2 + 1 c2 = bc + ac + ab abc 2 2 (c + b + a) abc Ahora haciendo las debidas sustituciones, resulta 1 a2 + 1 b2 + 1 c2 = 7 6 2 2 (0) 6 = 49 36 42. Al simpli…car la expresión A = x2 (x y) (x z) y2 (y z) (y x) + z2 (z x) (z y) el resultado es: Solución Efectuando las operaciones indicadas se obtiene x2 (x y) (x z) y2 (y z) (y x) + z2 (z x) (z y) = x2 (y z) y2 (z x) + z2 (x y) (x y) (x z) (y z) = (x z) xy xz + yz + y2 (x y) (x z) (y z) = xy xz + yz + y2 (x y) (y z) 31
  • 32. 43. El conjunto solución de la ecuación x2 6x + 10 x2 + 8x + 17 = x 3 x + 4 2 , es: Solución Desarrollando el cuadrado en el miembro derecho de la igualdad, tenemos x2 6x + 10 x2 + 8x + 17 = x2 6x + 9 x2 + 8x + 16 de lo cual se obtiene x2 6x + 10 x2 + 8x + 16 = x2 6x + 9 x2 + 8x + 17 x4 + 2x3 22x2 16x + 160 = x4 + 2x3 22x2 30x + 153 16x + 160 = 30x + 153 16x + 30x = 153 160 14x = 7 x = 7 14 x = 1 2 44. Un barril contiene 120 litros de alcohol y 180 litros de agua;un segundo barril contiene 90 litros de alcohol y 30 litros de agua¿ Cuántos litros debe tomarse de cada uno de los barriles para formar una mezcla homogénea que contenga 70 litros de agua y 70 litros de alcohol. Solución El primer barril contiene una mezcla 300 litros, en la cual el (120) (100) 300 = 40 porciento es alcohol y el (180) (100) 300 = 60 porciento es agua. En el segundo barril hay una mezcla de 90 litros de alcohol y 30 litros de agua, es decir el (90) (100) 120 = 75 porciento es alcohol y el (30) (100) 120 = 25 porciento es agua. Esto signi…ca que cualquier cantidad que se tome del primer barril contiene un 60% de agua y un 40% de alcohol, así mismo al tomar cualquier cantidad del segundo barril contiene un 25% de agua y un 75% de alcohol. Sea x la cantidad de litros que se tomará del primer barril y y la cantidad que será tomada del segundo barril. Como la mezcla debe contener 70 litros de agua y 70 litros de alcohol, entonces planteamos el siguiente sistema de ecuaciónes ( 0:4x + 0:75y = 70 Ec (1) 0:6x + 0:25y = 70 Ec (2) Luego, si multiplicamos la primera ecuación por 0:25 y la segunda por 0:75, obtenemos ( 0:1x 0:1875y = 17:5 Ec (3) 0:45x + 0:1875y = 52:5 Ec (4) Ahora sumando miembro a miembro las ecuaciones 3 y 4 resulta 0:35x = 35: 32
  • 33. Despejando x se llega x = 100. Sustituyendo en la ecuación 1 o en la ecuación 2 se obtiene que el valor de y es 40. Por lo tanto, para formar una mezcla homogénea que contenga 70 litros de agua y 70 litros de alcohol hay que tomar 100 litros del primer barril y 40 del segundo. 45. La hierba crece en todo el prado de la hacienda "el Meymo" con igual rapidez y espesura. Se sabe que 70 vacas se la comerían en 24 días y 30 en 60 días ¿ Cuántas vacas se comerían toda la hierba en 96 días? Solución Sea p el prado y v la rapidez de crecimiento por día del pasto. Sabemos que # vacas Cantidad de pasto consumida # de días 70 p + 24v 24 30 p + 60v 60 Luego tendríamos que 30 70 p + 24v p + 60v = 24 60 30 70 60 24 = p + 60v p + 24v 15 14 = p + 60v p + 24v 15 (p + 24v) = 14 (p + 60v) 15p + 360v = 14p + 840v 15p 14p = 840v 360v p = 480v: Ahora # vacas Cantidad de pasto consumida # de días 70 p + 24v = 480v + 24v = 504v 24 x p + 96v = 480v + 96v = 576v 96 por lo cual x 70 504v 576v = 24 96 x 70 7 8 = 24 96 x = 24 96 8 7 70 x = 20 46. En un gallinero había cierto número de gallinas, se duplicó el número y se vendio 27 quedando menos de 54. después se triplicó el número de gallinas que habia al principio y se vendió 78, quedando más de 39,¿Cuántas gallinas habia al principio? Solución Sea x el número de gallinas que había al principio. La expresión: "se duplicó el número y se vendió 27 quedando menos de 54" se representa en el lenguaje algebraico de la siguiente manera 2x 27 < 54, y el conjunto solución 33
  • 34. de dicha desigualdad es ( 1; 40:5) : La otra expresión "Después se triplicó el número de gallinas que habia al principio y se vendió 78, quedando más de 39" la representamos mediante la desigualdad 3x 78 > 39, cuyo conjunto solución es (39; 1). Luego, como la intersección de los conjuntos soluciones de las dos desigualdades es 40, resulta que inicialmente habían 40 gallinas. 47. Un grupo de abejas cuyo número era igual a la raíz cuadrada de la mitad de todo su enjambre se posó sobre un jazmin, habiendo dejado muy atrás a 8 9 de su enjambre, sólo una abeja del mismo enjambre revoloteaba en torno a una ‡or de sacuanjoche, atraida por el zumbido de una de sus amigas que cayó imprudentemente en la trampa de dulce fragancia.¿cuántas abejas formaban el enjambre? Solución Sea x el número de abejas del enjambre. La información del problema nos proporciona la siguiente ecuación: r x 2 = x 9 2 Elevando al cuadrado ambos miembros de la igualdad, tenemos x 2 = x2 81 4x 9 + 4: Luego, x2 81 17 18 x + 4 = 0: Utilizando la fórmula general para resolver esta ecuación, se obtienen las soluciones 72 y 9 2 ; pero como 9 2 no es entero, nos quedamos 72, es decir que en el enjambre habían 72 abejas. 48. Si x4 y4 = z3 y x2 + y2 = 8, entonces z3 8 es igual a: Solución : Sustituyendo x4 y4 = z3 y x2 + y2 = 8 en z3 8 obtenemos z3 8 = x4 y4 x2 + y2 ; factorizando la diferencia de cuadrados z3 8 = x2 + y2 x2 y2 x2 + y2 y simpli…cando z3 8 = x2 y2 factorizando una vez más obtenemos z3 8 = (x + y) (x y) 49. Al simpli…car x 2=3 y 4=3 z 4 x 1=3y2=3z 7=3 3 resulta Solución : Al aplicar propiedades de exponentes x 2=3 y 4=3 z 4 x 1=3y2=3z 7=3 3 = x2 y4 z12 xy 2z7 = xy6 z5 34
  • 35. 50. Si 2x3 + x2 + px + 2p2 es divisible entre x + 1, siendo p un número real, entonces el valor de p es: Solución : Como el polinomio dado es divisible por x + 1; entonces P ( 1) = 0 P (x) = 2 1 p + 2p2 2p2 p 3 = 0 (p + 1) p 3 2 = 0 luego el polinomio es divisible por (p + 1) 51. El conjunto solución de la desigualdad 3 2x + 3 < 1 x 2 es Solución : Al reescribir la desigualdad dada 3 2x + 3 1 x 2 < 0 x 9 2x2 x 6 < 0 x 9 (2x + 3) (x 2) < 0 de aqui, podemos observar que los puntos criticos son x = 3 2 ; x = 2 ; x = 9; entonces Expresión x 9 2x + 3 x 2 Signo x < 3 2 3 2 < x < 2 + + 2 < x < 9 + + x > 9 + + + + por tanto, el conjunto solución está de…nido por los intervalos donde está el signo negativo, es decir 1; 3 2 [ (2; 9) 52. Dos enteros a > 1 y b > 1 satisfacen ab + ba = 57. Determinar la suma a + b Solución : Uno de los enteros, digamos a, debe ser par, mientras que el otro, b, debe ser impar. Como 43 = 64 > 57, tenemos que a = 2; entonces b = 5. Entonces dicha suma debe ser 7 35
  • 36. 53. Si x + y = 1 y xy = 1 , ¿cuál será el valor de x3 + y3 ? Solución : Elevando al cubo la expresión (x + y) = 1, y aplicando las condiciones dadas en el ejercicio, se obtiene (x + y) 3 = x3 + 3x2 y + 3xy2 + y3 = 1 x3 + 3xy (x + y) + y3 = 1 x3 + 3 (1) (1) + y3 = 1 x3 + y3 = 2 54. El polinomio p(x) = x3 x2 + x 1 se anula en 1, luego p(x) es divisible por: Solución : Decir que P (x) se anula en 1 signi…ca que P (1) = 13 12 + 1 1 = 0 entonces (x 1) es un divisor de este polinomio. 55. La suma de dos números es 666 y si se divide el mayor entre el menor el cociente es 5 y el residuo 78. Dichos números son: Solución : Sean x el número mayor, y el número menor, entonces x + y = 666 x = 5y + 78 cuya solución es fx = 568; y = 98g 56. Si suponemos que el cociente intelectual de Einstein era 170 y si éste se calcula al dividir la edad mental por la edad cronológica multiplicado por 100, la edad mental de Einstein cuando publicó en 1905 su teoría sobre el efecto fotoeléctrico era: Solución : El coe…ciente intelectual (CI), se de…ne por CI = EM EC 100 donde EM es la Edad Mental y EC es la Edad Cronologica, de aqui 170 = EM 26 100 cuya solución es 44:2 36
  • 37. 57. Mi hijo es ahora tres veces más joven que yo, pero hace cinco años era cuatro veces más joven. ¿Cuántos años tiene el hijo? Solución : Sea P la edad actual del padre y H la edad actual del hijo, entonces 3H = P 4 (H 5) = P 5 cuya solución es H = 15; P = 45 58. Un grupo de amigos fue a tomar unos refrescos y unas empanadas, y lo pusieron todo en una cuenta que ascendió a 36 córdobas. Todos iban a pagar por igual, pero tres de ellos se habían ido, por lo que a cada uno le tocó pagar 1 córdobas más. ¿Cuántas personas conformaban el grupo original? Solución : Llamemos x al número de amigo al principio e y al costo si hubieran estado todos, entonces xy = 36 (x 3) (y + 1) = 36 cuya solución es x = 12; y = 3 59. Un hombre entró en la cárcel para cumplir una condena. Para que su castigo fuera más duro no le dijeron cuanto tiempo tendría que estar allí dentro. Pero el carcelero era un tipo muy decente y el preso le había caído bien. Preso: ¡Vamos!. ¿puedes darme una pequeña pista sobre el tiempo que tendré que estar en este lugar? Carcelero: ¿Cuántos años tienes? Preso: Veinticinco. Carcelero: Yo tengo cincuenta y cuatro. Dime, ¿qué día naciste? Preso: Hoy es mi cumpleaños. Carcelero: Increíble. ¡También es el mío!. Bueno, por si te sirve de ayuda te diré (no es que deba, pero lo haré) que el día que yo sea exactamente el doble de viejo que tú, ese día saldrás. ¿Cuánto tiempo dura la condena del preso? Solución : Sea x tiempo de condena del preso, entonces 2 25 + x = 54 x = 4 37
  • 38. 60. El producto de tres enteros positivos consecutivos es 3360 y su suma es 45. ¿Cuál es el mayor de esos tres números? Solución : Este problema se puede resolver utilizando la segunda condición x + (x + 1) + (x + 2) = 45 x = 14 como los números son consecutivos, entonces el mayor es x + 2 = 16 61. Un autobús comienza su trayecto con un cierto número de pasajeros. En la primera parada descienden 1=3 de los pasajeros y suben 8. En la segunda parada descienden 1=2 de los pasajeros que quedan y suben 2 nuevos. En este momento, el autobús lleva la mitad del número de pasajeros de los que llevaba al principio del trayecto. ¿Cuántos pasajeros había al principio? Solución : Llamamos x al número de pasajeros que había al comienzo del viaje. En la primera parada desciende 1 3 de los pasajeros. Luego se quedan 2 3 x. Suben 8. Por tanto, después de la primera parada en el autobús hay 2 3 x + 8 pasajeros. En la segunda pasada descienden 1 2 de los pasajeros, luego se queda 2 3 x + 8 2 . Suben otros 2. Por tanto, después de la segunda parada en el autobús hay 2 3 x + 8 2 +2: En ese momento el número de pasajeros es la mitad de los que había al principio, es decir, x 2 : Igualamos y obtenemos 2 3 x + 8 2 + 2 = x 2 cuya solución es x = 36 62. Hallar tres números sabiendo que el segundo es mayor que el primero en la misma cantidad que el tercero es mayor que el segundo, que el producto de los dos menores es 85 y que el producto de los dos mayores es 115. Solución : Sean x; y; z los tres números buscados, además x < y < z; entonces 8 >>< >>: y x = z y xy = 85 yz = 115 cuya solución es z = 23 2 ; y = 10; x = 17 2 38
  • 39. 63. Daniel y Arturo, dos viejos amigos, vuelven a encontrarse en la calle al cabo de algunos años. Después de saludarse, Daniel : ¿Cuántos hijos tienes? Arturo : Tres hijos. Daniel : ¿Qué edades tienen? Arturo : Tú mismo lo vas a averiguar. El producto de sus edades es 36. Daniel, después de pensar durante algún tiempo, le dice a Arturo que necesita más datos. Arturo : En efecto, la suma de sus edades es igual al número de la casa que tenemos enfrente. Daniel mira el número de la casa que le indica Arturo y quedándose pensativo durante un par de minutos. - ¡No es posible! - responde, con lo que me has dicho no puedo conocer las edades de tus hijos. Me falta un dato más. Arturo : Perdona Daniel, olvidé decirte que mi hija la mayor toca el piano. Daniel: En ese caso, ya sé sus edades. ¿Qué edades tienen los hijos de Arturo? Solución : El problema se reduce a encontrar tres números naturales cuyos producto sea 36, sean x; y; z dichos números, entonces xyz = 36 entonces se forman las siguientes posibilidades, cada una con sus respectivas sumas 1 1 36 = 38 2 2 9 = 13 1 4 9 = 14 1 6 6 = 13 1 2 18 = 21 Observemos que el dato del número de la casa es la clave, ya que el número 13 se repite dos veces ( el cual constituye el número de la casa ), por tanto, de las posibilidades 2 2 9 = 13 y 1 6 6 = 13; la correcta es la primera, ya que para el segundo caso, no existe una única hija mayor. 64. Un ciclista calcula que si avanza a 10 km=hora llegará a su destino a la 1p:m., y si avanza a 15 km=hora llegará a su destino a las 11a:m. ¿a qué velocidad, en km=hora, tiene que avanzar para llegar a las 12m.? Solución : Sabemos que d = vt aplicando las condiciones del problema d = 10t d = 15 (t 2) igualando 10t = 15t 30 t = 6 39
  • 40. ahora 10t = v (t 1) 60 = 5v v = 12 65. Un camino puede recorrerse en “t”horas con una cierta velocidad en km=hr. El mismo camino se puede hacer en una hora menos aumentando en un kilómetro por hora la velocidad. Hallar la longitud del camino en km. Solución : Sabemos que d = vt además la velocidad se aumenta en 1; disminuyendo el tiempo en 1; es decir d = (v + 1) (t 1) d = vt v + t 1 entonces vt = vt v + t 1 v = t 1 por tanto d = (t 1) t d = t2 t 66. De un depósito de 100 litros de capacidad, lleno de alcohol puro, se saca una cierta cantidad de alcohol y se le reemplaza por agua. Se saca después la misma cantidad de mezcla y se reemplaza por agua, quedando ésta última mezcla con un 49% de alcohol. Determinar la cantidad de líquido que se ha sacado cada vez. Solución : La cantidad que se saca y se reemplaza es la misma, entonces la proporción de lo que se extrae es x 100 ; entonces en la primera extracción, se tiene Seleccion Alcohol Agua Inicio 100 0 1 100 x x 2 (100 x) x 100 (100 x) x x 100 por tanto (100 x) x 100 (100 x) = 49 y vemos que al resolver para x se obtiene el valor de x = 30: Note que este problema tiene una raíz rara, extraña o falsa y ocurre para x = 170 40
  • 41. 67. La suma de tres números es 21. El cociente de dos de ellos es 2:5 y la suma de estos dividida entre el tercero da como cociente 2. ¿Cuál es el menor de los tres números? Solución : Sean x; y; z los tres números, planteando el sistema de ecuaciones 8 >>>< >>>: x + y + z = 21 x y = 2:5 x + y z = 2 8 >>< >>: x + y + z = 21 x 2:5y = 0 x + y 2z = 0 al resolver dicho sistema, se tiene que x = 10:0; y = 4:0; z = 7:0 entonces el número menor corresponde a y = 4 68. Un padre actualmente tiene el triple de la edad de su hijo; si hace 6 años la edad del padre era el quíntuple de la edad de su hijo. Señale la suma de cifras de edad del padre. Solución : Sean x; y las edades respectivas del padre e hijo respectivamente, entonces ( x = 3y x 6 = 5 (y 6) al resolver dicho sistema de ecuación, se tiene x = 36; y = 12; entonces la suma de las cifras de la edad del padre es 9 69. Dos tuberías abiertas simultáneamente llenan un depósito en 1 hora 12 minutos. Si una de ellas tarda 1 hora más que la otra, en llenar el mismo depósito ¿en qué tiempo lo llenará la tubería de mayor caudal? Solución : Sean x : tiempo de la tubería de menor caudal y : tiempo de la tubería de mayor caudal planteando el sistema de ecuaciones lineales en formato de minutos 8 < : 1 x + 1 y = 1 72 x = y + 60 resolviendo para y; se tiene que y = 120 min, lo cual equivale a y = 2 horas 41
  • 42. 70. Un albañil y su ayudante pueden hacer una obra en 24 días. Después de 4 días de trabajo, el ayudante se retira y el albañil termina lo que falta del trabajo en 30 días. ¿En cuántos días podría hacer el trabajo el ayudante trabajando solo? Solución : Al plantear una regla de tres compuesta Hombres Dias Proyectado Dias Reales 2 24 20 1 x 30 se obtiene x = 2 24 30 1 20 = 72 71. En Navidad, en cierta empresa todos los empleados se ofrecen regalos. En esta ocasión las mujeres se han dado mutuamente un regalo, pero los hombres lo han repartido: la mitad han dado un regalo a sus compañeros y la otra mitad lo han ofrecido a cada una de sus compañeras. Sabemos que el doble del número de mujeres excede en 6 al número de hombres. Si en total se han dado 318 regalos, ¿cuántos empleados tiene la empresa? Solución : Sean x número de hombres, y número de mujeres, sabemos que x = 2y 6, y (y 1) cantidad de regalos de las mujeres, porque cada mujer da un regalo a otra mujer, también x 2 (x 1) + x 2 y porque la mitad de los hombres da un regalo a otro hombre y la otra mitad a las mujeres, de aqui, obtenemos que x = 2y 6 x 2 (x 1) + x 2 y + y (y 1) = 318 cuya solución es y = 11; x = 16 72. Determinar un entero positivo con los datos siguientes: si se añade un 5 a la derecha el número resultante es divisible exactamente por un número que sobrepasa en 3 el buscado, siendo el cociente igual al divisor menos 16. Solución : Llamemos N al número buscado. El número divisible será (10N + 5), el divisor será (N + 3) y el cociente será (N + 3) 16 = (N 13). Entonces (10N + 5) = (N + 3)(N 13) N2 20N 44 = 0 cuya solución es N = 2; N = 22; por tanto, la solución positiva es el resultado. 42
  • 43. 73. Hallar un número de dos cifras sabiendo que el número de unidades excede en dos el número de decenas y que el producto del número deseado por la suma de sus dígitos es 144. Solución : El número deseado N, cumplirá N = 10d + u; u = d + 2; entonces (10d + u)(d + u) = 144 (10d + d + 2) (d + d + 2) = 144 22d2 + 26d 140 = 0 cuya solución es d = 2; de donde u = 4 y N = 24 74. Si n es un entero positivo, la igualdad m4 km2 n + n2 n = m2 n 2n se cumple si k toma el valor: Solución : Si k = 2, entonces La expresión m4 km2 n + n2 n = m4 2m2 n + n2 n = h m2 n 2 in también m2 n 2n = h m2 n 2 in 75. Un factor de 5t 12 + 2t2 es t + 4 y el otro es: Solución : Al factorizar el polinomio dado 5t 12 + 2t2 = (t + 4) (2t 3) el otro factor es 2t 3 76. Si el producto de los monomios x2n yn y xm y es igual a x 2 y3 , entonces los valores de m y n son respectivamente: Solución : Como x2n yn (xm y) = x 2 y3 entonces 2n + m = 2 n + 1 = 3 de donde se obtiene de forma inmediata que n = 2; m = 6 43
  • 44. 77. Supongamos que x1 y x2 son las raíces de la ecuación ax2 + bx + c = 0; (a 6= 0) la expresión 1 x2 1 + 1 x2 2 expresada en función de las raíces, es igual a: Solución : La solución de una ecuación cuadrática tiene la forma b p b2 4ac 2a entonces se pretende calcular 1 x2 1 + 1 x2 2 = x2 2 + x2 1 x2 1x2 2 = b p b2 4ac 2a !2 + b + p b2 4ac 2a !2 " b + p b2 4ac 2a ! b p b2 4ac 2a !#2 = b2 2ac c2 78. La raíz quinta de la raíz cuarta de la raíz cuadrada de la raíz cuadrada de (a2 + b2 ) es igual a: Solución : Simbolizando las raices respectivas y multiplicando cada uno de sus indices, obtenemos que 5 s 4 rqp (a2 + b2) = 80 p (a2 + b2) = a2 + b2 1=80 79. El sistema ( kx + y = 1 x + ky = 2 tiene solución única si: Solución : Un sistema de ecuación tiene solución unica si y solo si el determinante del sistema es distinto de cero, para este ejercicio tenemos k 1 1 k = k2 1 6= 0 de aqui que, k 6= 1; k 6= 1 44
  • 45. 80. La suma de las cuatro raíces de las ecuaciones ax2 + bx + c = 0 y ax2 bx + c = 0; con a 6= 0 y b2 4ac > 0 es igual a: Solución : Las raices de las ecuaciones ax2 + bx + c = 0 y ax2 bx + c = 0 son respectivamente b 2a y b 2a las que al sumarse se obtiene b 2a + b 2a = 0 45
  • 46. UNIDAD DE GEOMETRÍA EUCLIDIANA 1. En la …gura, el ]COB = 120o y el ]COD mide la mitad del ángulo BOA. Entonces, la medida del ]BOA es: Solución: Sea mBOA = x, luego mCOD = x 2 . Se tiene x 2 + 120o + x = 180o 3x 2 = 60o x = 40o 2. Si dos planos diferentes se intersecan, su intersección es: Solución: Por uno de los axiomas de la Geometría Euclidiana si dos planos diferentes se intersecan, su intersección es una única recta. 3. . En la …gura, !m1 ? !m4, !m2 ? !m3¿cuál de las siguientes expresiones es siempre verdadera? Solución: Con la información dada las parejas de rectas perpendiculares están “libres”, luego pueden ser giradas y 46
  • 47. seguirían satisfaciendo los datos dados. Por tanto no puede a…rmarse ni A, ni B, ni C, ni D. 1m ↔ 2m ↔ 3m ↔ 4m ↔ 4. R; S y T son tres puntos colineales como se muestran en la …gura. Si ST = 4x + 4 y RS es la mitad de ST, entonces la longitud de RT es: Solución: Dado que los puntos son colineales, se tiene RT = RS + ST = 1 2 ST + ST = 3 2 ST = 3 2 (4x + 4) = 6x + 6 5. A partir de la información indicada en la …gura, el valor de Y es: Solución: Sean A, B y C los puntos indicados en la …gura, y sean mBAC = , mACB = . Se tiene = 50o , por ser opuesto por el vértice con el ángulo que mide 50o y = 180o 130o = 50o . El ángulo que mide yo es un ángulo exterior con respecto al 4ABC, luego su medida equivale a la suma de los ángulos internos no adyacentes, es decir y = + = 50o + 50o = 100o 47
  • 48. 6. En la …gura, si AB k CD, el valor de X es: Solución: Dado que las rectas son paralelas, xo = mFCE, por ser ángulos correspondientes. A su vez este ángulo por ser externo al 4ECD, es la suma de las medidas de los ángulos CED y EDC.Se tiene mCED = 90o y mEDC = 180o 140o = 40o , luego x = 90o + 40o = 130o : 7. A partir de la información brindada en la …gura, el valor de Z resulta: Solución: Las marcas en el ángulo A, indican que AD es bisectriz de dicho ángulo, luego x = 40o y mA = 80o . Al considerar el 4ABC, se tiene z = 180o 80o 70o = 30o . 8. En la …gura, AD ? AC; EB k DC,entonces el valor de Y es: Solución: Dado que EBkDC, se tiene x = 180o 130o = 50o por ser ángulos internos al mismo lado, entre paralelas y como AD?AC el 4ADC es triangulo rectángulo y por tanto “y”es el complemento de “x”, luego y = 90o 50o = 40o . 48
  • 49. 9. En la …gura el valor de X es Solución: Se tiene mABC = 180o 140o = 40o , x = 115o 40o = 75o ya que el ACD es externo al 4ABC. 10. En la …gura el valor de X es: Solución: Se tiene mEDB = 180o 150o = 30o , ABC = DBE por ser opuestos por el vértice y por el teorema de semejanza AA, 4ABC 4DBE, luego mBAC = xo = mEDB = 30o . 11. A B C D; E y F son puntos medios de AB y CD respectivamente; Si AC = 10 y BD = 12, entonces EF =? Solución: Sean AE = EB = x, CF = FD = y (E y F son puntos medios de AB y CD respectivamente).Se tiene: BC = AC AB = 10 2x (1) y también BC = BD CD = 12 2y (2) 49
  • 50. Igualando (1) y (2): 10 2x = 12 2y: Al simpli…car se obtiene: y x = 1 (3) Por otro lado se tiene EF = EB + BC + CF = x + (10 2x) + y = 10 x + y = 10 + (y x) Al introducir (3) resulta EF = 10 + 1 = 11 12. En la …gura o + o = 255o , entonces ¿mA =? Solución: En el 4ABC, tenemos que mA = 180o mABC mACB = 180o (mABC mACB) (1) Se tiene que mABC = 180o y mACB = 180o , luego mABC + mACB = 360o ( + ); Como + = 255, resulta mABC + mACB = 360o 255o = 105o ; Sustituyendo en (1) obtenemos mA = 180o 105o = 75o 13. ¿Para qué valor de x, los segmentos ABy CD son paralelos? Solución: Como el ángulo a la izquierda de C es congruente con el ángulo a la derecha, también mide 25o . Luego mACD = 180o 2 (25o ) = 130o . Como el 4APC, es recto en P, mPAC = 90o 25o = 65o . 50
  • 51. Para que AB y CD sean paralelos, el ángulo CAB debe ser el suplemento del ángulo ACD ya que serían ángulos internos al mismo lado entre paralelas o sea mCAB = 180o 130o = 50o . Se tiene entonces x + 50 + 65 = 180 x = 180 50 65 x = 65 14. Si AB k CD, ¿cuál es el valor de X? Solución: Trazamos EF, paralela a las rectasAB y CD, luego mAEF = 180o 120o = 60o y mFEC = 180o xo . Además se tiene mAEF + mFEC = mAEC = 90o , luego 60o + (180o xo ) = 90o . Al despejar x, resulta xo = 150o . 15. Si la medida de un] es tres veces la medida de su suplemento, ¿cuál es la medida de dicho ]? Solución: Sean la medida del ángulo buscado y la medida de su suplemento, luego + = 180o ) = 180o . El ejercicio indica que = 3 , luego = 3(180o ) = 540o 3 4 = 540o = 135o 16. . Dos veces la medida de un ] es 30 menos que cinco veces la medida de su complemento, ¿cuál es la medida de dicho ángulo? Solución: Sean la medida del ángulo buscado y la medida de su complemento, luego + = 90o = 90o 51
  • 52. Al interpretar la información del ejercicio se tiene 2 = 5 30o 2 = 5 (90o ) 30o 2 = 450o 5 30o 7 = 420o = 60o 17. En la …gura las rectas !m1 y !m2 son paralelas. Entonces el valor de x es: Solución: Sean A, B, C y D los puntos indicados en la …gura. Al trazar por B una paralela a !m1 y !m2 , se forman ángulos alternos –internos entre paralelas, y por tanto congruentes con los ángulos indicados inicialmente, luego x + 60 = 110 x = 110 60 x = 50 Otra Forma: Al prolongar CB, sea D el punto donde corta a la recta !m1. Se tiene mADB = xo , por ser alterno –interno con el ángulo que se forma en C. El ángulo ABC es externo al 4ABD, luego 60 + x = 110 ) x = 50. 18. En la …gura las rectas !m1 y !m2 son paralelas. Entonces el valor de x es: 52
  • 53. Solución: Como las rectas son paralelas se tiene: (3x + 10) + (x 6) = 84 4x + 4 = 84 4x = 80 x = 20 19. Si mP = 90o ; 1 = 2; 3 = 4, entonces mR es Solución: Sean y las medidas de los ángulos indicados en la …gura. Se tiene + = 90o . Como SQR = 2 y 1 = 2, se tiene 2 mSQR = 180o (1) Similarmente se obtiene que 2 mQSR = 180o (2) Al sumar (1) y (2) resulta 2 mSQR + 2 mQSR = (180o ) + (180o ) = 360o ( + ) Como + = 90o , 2(mSQR + mQSR) = 360o 90o 2(mSQR + mQSR) = 270o mSQR + mQSR = 135o Luego mR = 180o (mSQR + mQSR) = 180o 135o = 45o 20. En una recta se toman los puntos A; B y C, de manera que B es punto medio de . Se toma otro punto O, tal que B O C. Encuentre el valor numérico de: AO OC OB Solución: 53
  • 54. Se tiene AB = BC = x, por ser B punto medio de AC. Sea OB = y, luego OC = x y, AO = x + y. Al sustituir estos valores en la expresión dada se tiene: AO OC = (x y) (x + y) = 2y, luego AO OC OB = 2y y = 2 Nota: en ejercicios de este tipo no se admite asignar valores arbitrarios, ya que se estaría resolviendo para valores especí…cos. El planteamiento es general. Cuando se a…rma que B O C, está indicando que O es un punto cualquiera que se encuentra entre B y C, y el valor numérico encontrado es valido para cualquier punto O que esté entre B y C. 21. Un poste cercano a un árbol mide 2m y su sombra en un momento dado mide 1:8m, entonces si la sombra del árbol en ese momento mide 11m, la altura del árbol es: Solución: Dado que los rayos del sol prácticamente caen paralelos y que el poste y el tronco del árbol son perpendiculares al piso, el árbol y su sombra y la línea que une sus extremos forman un triángulo semejante al formado por el poste su sombra y la línea que une sus extremos, tenemos h 11 = 2 1:8 h = 11:2 1:8 h = 12:22 22. Una varilla clavada en el piso y cercana a un árbol mide 3m y su sombra mide 1:5m, entonces si el árbol mide 36m, su sombra mide. Solución: El problema es similar al anterior, en este caso se tiene x 36 = 1:5 3 x = 36 1:5 3 x = 18 23. El perímetro de un triángulo rectángulo isósceles con hipotenusa igual a 10 redondeado a dos decimales es Solución: En un triángulo rectángulo isósceles, la hipotenusa mide p 2x, siendo x la longitud de sus catetos, luego p 2x = 10 x = 10 p 2 x = 5 p 2 54
  • 55. Su perímetro será P = 10 + 2 5 p 2 = 10 + 10 p 2 = 24:14 24. En el triángulo rectángulo de la …gura, los valores de x y y, respectivamente son Solución: Por el teorema de la altura se tiene 4x = 82 = 64 x = 16 y la hipotenusa mide 4 + x = 20. Por el teorema de los catetos se tiene y2 = 4 20 = 80 y = p 80 y = 4 p 5 y 8:94 25. Un método para encontrar la altura de un edi…cio es colocar un espejo en el suelo y después situarse de manera que la parte más alta del edi…cio pueda verse en el espejo ¿qué altura tiene un edi…cio si una persona cuyos ojos están a 1:5m del piso observa la parte superior del edi…cio cuando el espejo está a 120 m del edi…cio y la persona está a6m del espejo? Solución: Dado que las leyes de la óptica indican que en un espejo plano, el ángulo de incidencia es igual al ángulo de re‡exión, se forman dos triángulos rectángulos semejantes, luego h 120 = 1:5 6 h = 30 m 26. La altura respecto a la hipotenusa de un triángulo rectángulo mide 10m y los segmentos que determina sobre la hipotenusa son entre sí como 7 es a 14. Entonces la longitud del cateto menor es 55
  • 56. Solución: Sean m y n los segmentos determinados por la altura sobre la hipotenusa, con m < n, luego m n = 7 14 n = 2m Por el teorema de la altura m n = 102 m 2m = 100 m2 = 50 m = 5 p 2 n = 10 p 2 La hipotenusa mide c = m + n = 15 p 2 Por el teorema de los catetos a2 = m (m + n) a2 = 5 p 2 15 p 2 a2 = 150 a = p 150 = 5 p 6 a 12:25 27. El perímetro de un rectángulo es 85m y su diagonal mide M. Por lo tanto los lados del rectángulo miden: Solución: Sean a y b los lados del rectángulo. Se tiene P = 2 (a + b) = 85 a + b = 42:5 (1) Además a2 + b2 = 32:52 = 1056:25 (2) Despejando b de (1), e introduciendo en (2) a2 + (42:5 a) 2 = 1056:25 2a2 85a + 750 = 0 a = 12:5 _ a = 30 56
  • 57. Al sustituir en (1) se obtiene b = 30 _ b = 12:5 28. El perímetro de un triángulo mide50 y sus lados son proporcionales a 4; 6 y 8. Entonces su lado mayor mide. Solución: Sean a, b y c las longitudes de los lados, con a < b < c, luego P = a + b + c = 50 y a 4 = b 6 = c 8 Por las propiedades de las proporciones a 4 = b 6 = c 8 = a + b + c 4 + 6 + 8 = 50 18 c = 8 50 18 = 200 9 29. En un triángulo rectángulo, un lado mide 2 p 106, otro 5 p 15. Si el lado desconocido es el menor, ¿cuánto mide? Solución: Como 2 p 106 > 5 p 15, la hipotenusa de este triángulo es 2 p 106, luego el cateto menor es a = r 2 p 106 2 5 p 15 2 = p 424 375 = p 49 = 7 30. El área del triángulo de la …gura, redondeada al entero más cercano, mide: Solución: Aplicamos la fórmula de Herón: A = p s (s a) (s b) (s c), donde s es el semiperímetro. Se tiene s = 6 + 7 + 9 2 = 11, luego A = p 11 (11 6) (11 7) (11 9) = p 11 5 4 2 = p 440 20:97 31. ¿Cuál es el área del triángulo de la …gura? Solución: 57
  • 58. Dado que es un triángulo rectángulo su área es la mitad del producto de sus catetos. El cateto desconocido mide b = p 102 62 = p 100 36 = p 64 = 8 Por tanto A = 1 2 (6) (8) = 24 32. Si un rectángulo de 3mde ancho y 10mde largo tiene la misma área que un triángulo rectángulo isósceles, entonces la longitud de cada cateto del triángulo es Solución: El área de un triángulo rectángulo isósceles está dada por A = 1 2 x2 , donde x es la longitud de sus catetos, luego tenemos que el área del rectángulo es 30, por tanto 1 2 x2 = 30 x = p 60 x = 2 p 15 33. El área de un trapecio isósceles de bases 22m y 10m y cuyos lados congruentes miden 10 es Solución: Por ser un trapecio isósceles, al proyectar la base menor sobre la base mayor, la base mayor queda dividida en tres segmentos de 6, 10 y 6 metros. Aplicando el teorema de Pitágoras, se tiene h = p 102 62 = p 64 = 8 Aplicando la fórmula para el área de un trapecio: A = (B+b) h 2 resulta A = (22 + 10) 8 2 = 128 m2 34. La siguiente …gura consta de siete cuadrados congruentes. El área total de esta …gura es 63cm2 . Entonces el perímetro de la …gura es: 58
  • 59. Solución: Observamos que el perímetro está formado por 16 veces el lado de cada cuadrado. Como hay siete cuadrados congruentes, cada uno tiene un área de x2 = 63 7 = 9 x = 3 Por tanto el perímetro de la …gura es P = 16 3 = 48cm. 35. Si ACEG es un cuadrado y el área del cuadrilátero BDFH mide 162 ¿cuánto mide AC? (las marcas iguales representan partes congruentes). Solución: La …gura indica que B, D, F y H son puntos medios de los lados del cuadrado ACEG, luego su área es el doble del área del cuadrado BDFH, es decir [ACEG] = 2 162 = 324 luego AC = p 324 = 18 36. Se tiene un trapecio ABCD donde es la base menor. BC = 10cm y CD = 20cm. Las medidas de los ángulos A; B y C son 30 ; 150 y 120 respectivamente, entonces AD =? Solución: Sean B0 y C0 las proyecciones de B y C sobre la base mayor y sean AB0 = x, C0D = y. Por ser BC paralela a AD, mD = 180 mC = 180o 120o = 60o El 4CC0D es un triángulo 30 –60, luego h = CC0 = 10 p 3 y y = C0D = 10. También el 4AB0B resulta ser un triángulo 30 –60, con su cateto menor BB0 = h = 10 p 3, luego AB0 = 10 p 3 p 3 = 30. Tenemos entonces que la base mayor mide AD = x + 10 + y = 30 + 10 + 10 = 50 59
  • 60. 37. Si las medianas en un triángulo rectángulo, trazadas a partir de los vértices de los ángulos agudos miden 5cm y p 40cm, entonces la medida de la hipotenusa del triángulo rectángulo es. Solución: Sean M y N los puntos medios de BC y AB respectivamente. Sean AM = 5 y CN = p 40, BC = a, AB = c, luego BM = a 2 y NB = c 2 . Sea la hipotenusa AC = b: Aplicando el teorema de Pitágoras en los 4ABM y 4BCN AM2 = AB2 + BM2 = c2 + a2 4 = 25 (1) CN2 = NB2 + BC2 = a2 + c2 4 = 40 (2) Al sumar (1) y (2) resulta 5c2 4 + 5a2 4 = 65 a2 + c2 = 4 5 (65) = 52 = b2 b = p 52 = 2 p 13 38. En la …gura, los cuadrados ABCD y EFGH son congruentes. AB = 10cm y G es el centro del cuadrado ABCD. Entonces el área total cubierta por el polígono AHEFBCDA es. Solución: Dado que los cuadrados son congruentes sus áreas son iguales y como el lado AB mide 10, cada uno tiene un área de 100cm2 , pero ellos comparten el 4ABG de manera que para el área total del polígono a la suma de las áreas de los cuadrados debemos restarle el área de este triángulo para que sea considerada solo una vez. Dado que G es el centro del cuadrado ABCD, el área del triángulo es la cuarta parte del área del cuadrado o sea 25cm2 . Luego el área buscada es A = [ABCD] + [EFGH] [ABG] = 100 + 100 25 = 175cm2 60
  • 61. 39. ABCD es un cuadrado, el 4ABE es isósceles, CF = FB. Entonces, la medida del ángulo EFB es igual a. Solución: Como el 4ABE es isósceles, AE = BE y por ser ABCD un cuadrado, E es el punto medio de DC y por tanto EC = CF, ya que por ser CF = FB, F es punto medio de BC. Luego el 4ECF resulta ser triangulo rectángulo isósceles y como consecuencia mCFE = 45o . El ángulo buscado es el suplemento del CFE, luego mEFB = 180o mCFE = 180o 45o = 135o 40. En la …gura, ABCF es un paralelogramo. B; C y D son colineales. Si AB = 18; AD = 30 y FE = 12. ¿Cuánto mide AE? Solución: Se tiene que CF = AB = 18, ya que ABCF es un paralelogramo. CE = CF FE = 18 12 = 6. Por otro lado BD y AF son paralelas, luego FAE = CDE, ya que son alternos internos entre paralelas y FEA = CED, ya que son opuestos por el vértice. Como consecuencia se tiene 4FEA 4CED. Sea AE = x, luego ED = AD AE = 30 x. Por la semejanza anterior, CE FE = ED EA 6 12 = 30 x x 6x = 360 12x 18x = 360 x = 20 41. En un trapecio isósceles, la diferencia de las bases es de 10m. La altura mide 12m. y el perímetro 76m. Entonces su área es: 61
  • 62. Solución: Como B b = 10, al proyectar la base menor sobre la base mayor se forman tres segmentos de longitudes 5, b y 5 como se muestra en la …gura. Luego como la altura es 12, en los extremos del trapecio se forman triángulos rectángulos de catetos 5 y 12. Aplicando el teorema de Pitágoras hallamos que la hipotenusa mide 13 lo cual corresponde a la longitud de los lados no paralelos del trapecio. Considerando que el perímetro mide 76 m. se tiene: 2b + 2(13) + 2(5) = 76 b = 20 Al considerar que B b = 10, resulta B = 30. Aplicando la fórmula para el área de un trapecio, el área buscada resulta A = (B + b) h 2 = (20 + 30) 12 2 = 300 m2 42. En la …gura ABCD es un cuadrado de lado 1cm y CE = 2cm, entonces el área del triángulo ADF en cm2 es igual a Solución: Dado que ABCD es un cuadrado AD y CE son paralelas, resultando que 4ADF 4ECF por el teorema de semejanza AA, ya que DAF = CEF por ser alternos internos entre paralelas y DFA = CFE por ser opuestos por el vértice. De la semejanza resulta que AD CE = DF CF 1 2 = DF CF CF = 2DF (1) Como CD = CF + FD = 1, resulta DF = 1 3 . Por tanto el área buscada resulta [ADF] = 1 2 AD DF = 1 2 1 1 3 = 1 6 62
  • 63. 43. Sea ABC un triángulo isósceles con AB = BC = 10 y AC = 16. Sea BD la mediana trazada sobre el lado AC y sea G el baricentro. Entonces el área del triángulo ADG es Solución: Por ser BD mediana, D es punto medio de AC, o sea AD = DC = 8. Ya que 4ABC es isósceles, BD además de mediana también es altura, luego mADB = 90o . Aplicando el teorema de Pitágoras hallamos que BD = p 102 82 = 6 Como G es el baricentro, BG = 2 GD y como BD = BG + GD = 6, resulta GD = 2. Por tanto el 4ADG resulta ser un triángulo rectángulo con catetos de longitudes 8 y 2, por tanto su área es [ADG] = 1 2 AD DG = 1 2 8 2 = 8 44. Sea ABC un triángulo isósceles con AB = AC = 17cm y P un punto cualquiera del lado BC, diferente de los puntos extremos. Por P se trazan una paralela a AC que corta a AB en Q y una paralela a AB que corta a AC en R. El perímetro del cuadrilátero AQPR es. Solución: Dado que QPkAC y RPkAB, AQPR es un paralelogramo y de ahí AQ = RP y AR = QP. Del paralelismo de los segmentos señalados anteriormente también resulta que los 4QBP y 4RPC son semejantes con el 4ABC y por tanto también son isósceles y de ahí QB = QP y RP = RC. Por tanto el perímetro del cuadrilátero AQPR, resulta P = AQ + QP + AR + RP = (AQ + QB) + (AR + RC) = AB + AC = 17 + 17 = 34 63
  • 64. 45. De acuerdo a la información que se proporciona en la …gura, el segmento de mayor longitud es. Solución: Dado que la suma de los ángulos internos de un triángulo suman 180o , en el 4ABD, resulta que el ángulo ABD mide 180o 70o 60o = 50o y en el 4BDC, mBDC = 180o 55o 60o = 65o . Una de las propiedades de los triángulos indica que el lado mayor se opone al ángulo mayor y viceversa. Al comparar las medidas de los ángulos del 4ABD, resulta que el mayor mide 70o y su lado opuesto es BD, luego BD es mayor que AB y AD. Pero al considerar el 4BDC, su ángulo mayor es 65o y el lado que se le opone es BC y por tanto BC > BD. Luego el lado mayor de la …gura resulta BC. 46. En la …gura ABCD es un cuadrado de lado 1; 4CMN es equilátero. El área de 4CMN es igual a. Solución: El área de un triángulo equilátero está dada por p 3 4 x2 , donde x es la longitud de su lado, luego debemos encontrar primero cuanto mide cada lado del triángulo equilátero CMN. Como ABCD es un cuadrado y CM = CN = x, se tiene que 4CDM = 4CBN y de ahí MD = NB y como AD = AB, resulta AM = AN y por tanto el 4MAN es rectángulo isósceles, luego MN = x x = p 2AN AN = x p 2 Como AB = 1, resulta NB = 1 AN = 1 xp 2 = p 2 xp 2 . El 4CBN es un triángulo rectángulo luego al aplicar el teorema de Pitágoras resulta CN2 = CB2 + NB2 = 1 + p 2 x p 2 !2 = x2 64
  • 65. Al desarrollar, simpli…car y resolver la ecuación resultante se obtiene x = p 6 p 2. Por tanto el área buscada es [CMN] = p 3 4 p 6 p 2 2 0:4641 47. La siguiente …gura muestra dos cuadrados de lado 1cm, donde AEFG se ha obtenido de ABCD al girar este cuadrado 45 sobre el vértice A. Entonces el área sombreada es. Solución: Al girar 45o , la recta diagonal AC se convierte en la recta AB la cual equivale a la recta diagonal AF, por tanto A, B, F son colineales. Además se tiene mBFH = 45o y por tanto FBH es un triángulo rectángulo isósceles con FB = BH. Luego [AGHB] = [AGF] [FBH]. Por ser AEFG un cuadrado de lado 1, su diagonal mide p 2 y como AB = 1, BF = BH = p 2 1. Como [AGF] tiene como área la mitad de la área del cuadrado, que tiene lado de longitud 1, resulta [AGHB] = 1 2 1 2 p 2 1 2 = 1 2 1 2 2 2 p 2 + 1 = p 2 1 48. Los ángulos agudos de un triángulo rectángulo, que también es isósceles, miden Solución: Por ser triángulo rectángulo isósceles tiene un ángulo de 90o y los otros dos ángulos congruentes, y dado que la suma de los ángulos internos de un triángulo suman 180o , cada uno de ellos mide 45o . 49. En la …gura ABCD es un cuadrilátero con AD kBC . La diagonal AC es perpendicular al lado CD .mBAC = 30 ; AC = 4 p 3 y AB = BC. Entonces el área de ABCD es igual a. 65
  • 66. Solución: Como AB = BC, el 4ABC es isósceles con mABC = 120o y mBCA = 30o y su base AC = 4 p 3. Al trazar una perpendicular desde B a AC, sea E el pie de la perpendicular. Por ser isósceles, BE también es mediana es decir E es punto medio de AC, luego se forman dos triángulos 30 –60 con las hipotenusas AB = BC y catetos mayor AE = EC = AC 2 = 2 p 3: Luego como el cateto mayor en un triángulo 30 –60, es veces el cateto menor, en este caso se tiene BE = 2. Como ADkBC el BAD es el suplemento del ABC, resulta mBAD = 180o 120o = 60o y como mBAC = 30o , se tiene mCAD = 30o y de ahí también el 4ADC es un triángulo 30 –60 con cateto mayor AC = 4 p 3. Como en todo triangulo 30 –60, el cateto mayor es p 3 el cateto menor, se tiene CD = 4. Finalmente tenemos [ABCD] = [ABC] + [ACD] = 1 2 AC BE + 1 2 AC CD = 1 2 4 p 3 2 + 1 2 4 p 3 4 = 12 p 3 50. Se tiene un trapecio ABCD donde BC es la base menor. BC = 10cm y CD = 20cm. Las medidas de los ángulos A; B y C son 30 ; 150 y 120 respectivamente, entonces el área del trapecio mide. Solución: Sean B0 y C0 las proyecciones de B y C sobre la base mayor y sean AB0 = x, C0D = y. Por ser BC paralela a AD, mD = 180 mC = 180o 120o = 60o . El 4CC0D es un triángulo 30 –60, luego h = CC0 = 10 p 3 y y = C0D = 10. También el 4AB0B resulta ser un triángulo 30 –60, con su cateto menor BB0 = h = 10 p 3, luego AB0 = 10 p 3 p 3 = 30. Tenemos entonces que la base mayor mide AD = x + 10 + y = 30 + 10 + 10 = 50 66
  • 67. Luego el área del trapecio resulta [ABCD] = (50 + 10) 10 p 3 2 = 300 p 3 51. En la …gura, mBAC = ; mBPC = m y BQC = 90 : Entonces la medida de BHC es. Solución: Como mBPC = mBQC = 90 , también mAPH = mAQH = 90 . APHQ es un cuadrilátero convexo y en todo cuadrilátero convexo la suma de sus ángulos internos es 360o , luego mBHC + + 90o + 90o = 360o y de ahí mBHC = 180o . 52. Si las medianas en un triángulo rectángulo, trazadas a partir de los vértices de los ángulos agudos miden 5cm y p 20cm, entonces la medida en cm de la hipotenusa del triángulo rectángulo es. Solución: Sean M y N los puntos medios de BC y AB respectivamente. Sean AM = p 20 y CN = 5, BC = a, AB = c, luego BM = a 2 y NB = c 2 . Sea la hipotenusa AC = b. Aplicando el teorema de Pitágoras en los 4ABM y BCN AM2 = AB2 + BM2 = c2 + a2 4 = 20 (1) CN2 = NB2 + BC2 = a2 + c2 4 = 25 (2) Al sumar (1) y (2) resulta 5c2 4 + 5a2 4 = 45 a2 + c2 = 4 5 (45) = 36 = b2 b = 6 67
  • 68. 53. En la …gura, los dos cuadrados tienen el mismo centro. La razón entre el lado del cuadrado menor y el lado del cuadrado mayor es 2 5 . Entonces la razón entre el área sombreada y el área del cuadrado mayor es. Solución: Sean “b”la longitud del lado del cuadrado menor y “a”la longitud del lado del cuadrado mayor, luego b a = 2 5 . Por la simetría de la …gura se deduce que el área sombreada, es decir el trapecio ABFE, representa la cuarta parte de la diferencia entre los dos cuadrados, luego [ABFE] = 1 4 ([ABCD] [EFGH]) [ABCD] = a2 y [EFGH] = b2 y de ahí [ABFE] = 1 4 a2 b2 . La razón buscada será [ABFE] [ABCD] = 1 4 a2 b2 a2 = 1 4 a2 b2 a2 = 1 4 1 b2 a2 Como b a = 2 5 , resulta [ABFE] [ABCD] = 1 4 1 4 25 = 21 100 54. En la …gura, AB = AC = 4, BD = DC = 3 y mBAC = 60 , entonces la longitud del segmento AD es Solución: Al unir B con C, obtenemos un triángulo equilátero, ya que AB = AC y mBAC = 60 . Se tiene que 4ABD = 4ACE, ya que sus tres pares de lados son congruentes, de ahí resulta mBAD = mCAD y por tanto AD es bisectriz del BAC. 68
  • 69. Al prolongar AD, sea E el punto donde corta a BC. Luego como el 4ABC es equilátero, AE además de bisectriz es mediatriz y por tanto AE ? BC y BE = EC = 2. Resulta entonces que el 4BED es rectángulo, con hipotenusa BD = 3 y un cateto, BE = 2. Por el Teorema de Pitágoras, DE = p 32 22 = p 5: Por otro lado AE es una altura en un triángulo equilátero de lado 4 y por tanto AE = 2 p 3. Finalmente obtenemos que AD = AE DE = 2 p 3 p 5: 55. En la …gura el cuadrilátero ACDE es un trapecio tal que ED = 15cm , AC = 24 cm y la altura es 12cm. Sabiendo que B es el punto medio del lado AC, el área del cuadrilátero OBCD es. Solución: Como EDkAC, resulta que 4ABO 4DEO, con razón de semejanza AB DE = 12 15 = 4 5 . Sean a y b las alturas de los triángulos ABO y DEO respectivamente, indicadas en la …gura. Dado que los elementos homólogos en triángulos semejantes están en la misma razón de semejanza, se tiene a b = 4 5 . Como a + b = 12 (la altura del trapecio), al considerar la razón anterior resulta a = 16 3 , b = 20 3 . Al analizar la …gura vemos que [OBCD] = [ACDE] [ABE] [DEO]. Tenemos que el área del trapecio ACDE resulta [ACDE] = 24 + 15 2 12 = 234 El 4ABE, tiene base 12 y altura 12, luego su área es [ABE] = 1 2 12 12 = 72 Para el 4DEO, resulta [DEO] = 1 2 15 20 3 = 50 ) [OBCD] = 234 72 50 = 112 56. En la …gura, ABCD es un cuadrado de lado 6cm y CE = DE = 5cm, entonces la longitud de es. 69
  • 70. Solución: Sean F y G los puntos medios de CD y BA respectivamente. Luego CF = FD = BG = GA = 3 y FG = 6: Como CE = DE, el 4CED es isósceles y por tanto E, F, G son colineales y EF ? CD y EG ? AB. El 4CFE es rectángulo en F, luego por el Teorema de Pitágoras, EF = p 52 32 = 4. EG = EF + FG = 4 + 6 = 10. El 4FGA también es rectángulo con EG = 10 y GA = 4, luego EA = p 102 + 32 = p 109: 57. En la …gura, a partir de la información dada, ¿cuál es el valor de x? Solución: Se tiene A = E , por dato, y ACB = ECD, por ser opuestos por el vértice, luego 4ABC 4EDC, por el teorema de semejanza AA. Entonces: CD CE = BC AC x 10 = 66 132 x = 5 58. ABCD es un paralelogramo. P es un punto de la diagonal AC. Trazamos por P paralelas a los lados del paralelogramo. Estas paralelas intersecan a los lados del paralelogramo en los puntos indicados en la …gura. Sabiendo que el área de ABCD es 40cm2 , entonces el área del cuadrilátero RQMN es igual a. Solución: Dado que RMkADkBC y NQkABkDC, resulta que los cuadriláteros ANPR, PQBR, DMPN y MCQP son paralelogramos y los segmentos NR, RQ, QM y MN son diagonales de esos paralelogramos. Es sabido que una diagonal divide a un paralelogramo en dos triángulos congruentes, con áreas igual a la mitad del área del paralelogramo. Por tanto [RQMN] = [ABCD] = 20: 70
  • 71. 59. En el triángulo rectángulo ABC¿cuál es la longitud del segmentoBC? Solución: Basta aplicar el teorema del cateto: 3x = 62 x = 12 60. Sea ABCD un cuadrado. Por el vértice A se traza un segmento que corta a la prolongación del ladoBC en E, al lado DC en F y a la diagonal BD en G. Si AG = 3 y GF = 1 ¿cuál es la longitud de FE? Solución: Sea x la longitud de cada lado del cuadrado. Desde G tracemos una perpendicular a AD y sea H el pie de esta perpendicular. Luego 4AGH 4AFD. Como AG = 3 y GF = 1, resulta AF = 4: De la semejanza se tiene AG AF = AH AD 3 4 = AH x AH = 3 4 x y HD = AD AH = x 3 4 x = 1 4 x: Como G está sobre la diagonal, HG = HD = 1 4 x. De la misma semejanza se tiene AG AF = HG DF 3 4 = x 4 DF DF = 1 3 x 71
  • 72. Luego FC = DC DF = x 1 3 x = 2 3 x. Como ABCD es un cuadrado, ADkBC y por tanto ADkCE. De ahí resulta que 4ADF 4ECF. De esta semejanza se tiene FE FA = FC FD FE 4 = 2 3 x 1 3 x FE = 8 61. En la …gura de abajo si la medida de los arcos AD y BC son 140o y 80 respectivamente, entonces el valor de es. Solución: Tenemos que = mdAC + mdCD 2 . Dado que mdAB + mdBC + mdCD + mdAD = 360o y mdBC + mdAD = 80o + 140o = 220o Entonces mdAC + mdCD = 140o , luego = mdAC + mdCD 2 = 140o 2 = 70o 62. El triángulo ABC está inscrito en un semicírculo de diámetro AB. Si AC = 8 y CD = 6, el área de la región sombreada tiene un valor de. Solución: El área sombreada es la diferencia entre el área del semicírculo y el área del triángulo. El 4ABC es rectángulo en C, por estar inscrito en un semicírculo. Luego por el Teorema de Pitágoras, AB = p 82 + 62 = 10, entonces r = 5. Por tanto el área del semicírculo es 1 2 r2 = 1 2 52 = 25 2 El área del triángulo está dada por 1 2 AC BC = 1 2 8 6 = 24 El área buscada es A = 25 2 24 15:27 72
  • 73. 63. El triángulo ABC está inscrito en un semicírculo de diámetro AB. Si AC = 8 y CD = 4:8, el área de la región sombreada tiene un valor de Solución: Por el Teorema de Pitágoras, AD = p 82 4:82 = 6:4. Como el 4ABC es rectángulo en C, se tiene por el teorema del cateto . Y de nuevo por el Teorema de Pitágoras resulta BC = 6. Dado que estos valores coinciden con los datos del ejercicio anterior, el área resulta la misma. 64. La circunferencia de la …gura tiene radio 2 y el arco XY Z tiene longitud . ¿Cuánto mide la cuerda XZ? Solución: Sea la medida del ángulo central XOZ. La longitud de un arco está dada por s = r , con el ángulo medido en radianes. Tenemos s = y r = 2, luego = s r = 2 , es decir 90o . Luego XOZ es un triángulo rectángulo isósceles con XZ como hipotenusa y por tanto XZ = 2 p 2 65. En la …gura el área del círculo mayor es 1 m2 . El círculo menor es tangente internamente al círculo mayor y también es tangente a los lados del ángulo inscrito que mide 60 . Entonces el área del círculo menor es 73
  • 74. Solución: Desde el vértice del ángulo inscrito, trazamos un diámetro. Sean O y O0 los centros de los círculos, mayor y menor respectivamente. Sea B el otro extremo del diámetro trazado, C el punto donde uno de los lados (el arriba) del ángulo corta a la circunferencia. Y sea D el punto de tangencia de este lado del ángulo con el círculo menor. Sean R y r los radios de los círculos, mayor y menor respectivamente.Tenemos que AO0 biseca al ángulo inscrito, luego mO0AD = mBAC = 30o . Como AB es un diámetro del circulo mayor, mACB = 90o resultando que mABC = 60o , y el 4ABC es 30 –60. Dado que AB = 2R, se obtiene que BC = R, ya que BC es el cateto menor y AB la hipotenusa del 4ABC. Como AC es tangente al círculo menor en D, AD?DO0, es decir mADO0 = 90o y de ahí mAO0D = 60o . Luego también el 4AO0D es un triángulo 30 –60 y su cateto menor O0D = r, mide la mitad de su hipotenusa, AO0. Se tiene O0B = r, por ser radio del circulo menor y de ahí AO0 = AB O0B = 2R r. Luego AO0 = 2 O0D 2R r = 2r r = 2 3 R Como el área del círculo mayor es R2 = 1 el área del círculo menor es r2 = 2R 3 2 = 4 9 R2 = 4 9 66. En la …gura C es el centro de la circunferencia de radio r y TP es un segmento tangente en T, de longitud 2r, entonces PC mide Solución: 74
  • 75. Como TP es tangente a la circunferencia en T, mPTC = 90o . Luego aplicando el teorema de Pitágoras resulta PC = q (2r) 2 + r2 = p 5r2 = r p 5 67. Los extremos de la …gura son semicírculos, ¿Cuál es el área de la región sombreada? Solución: Como el área sombreada únicamente son los extremos y estos son semicírculos, al unirlos se forma un circulo de diámetro 8, es decir de radio 4, luego A = r2 = 42 = 16 68. En la …gura AC es un diámetro. Si mAB = 50 , entonces mBAC =? Solución: Dado que mdAB = 50 , se tiene mBCA = 1 2 mdAB = 25 , por ser ángulo inscrito que subtiende dicho arco; mABC = 90o , por estar inscrito en una semicircunferencia ( es diámetro). Luego la medida del ángulo buscado es: mBAC = 180 mBCA mABC = 180o 25o 90o = 65o 69. En la …gura, los círculos son tangentes y tienen radio igual a 10. Si se unen los centros de los círculos se forma un cuadrado. ¿Cuál es el área de la región sombreada? 75
  • 76. Solución: El área de la región sombreada es la diferencia entre el área del cuadrado formado y las áreas de los cuatro sectores circulares que se forman. Dado que la distancia entre los centros de dos círculos tangentes exterior- mente, es la suma de las longitudes de los radios, resulta que el cuadrado formado tiene lado de longitud 20 y por tanto el área del cuadrado es 400. Cada sector formado tiene un ángulo central de 90o , luego entre los cuatro forman un circulo de radio 10, cuyas áreas suman entonces r2 = 102 = 100 : Por tanto el área buscada es: A = 400 100 70. En la …gura, la medida del arco AB es 30 , y la medida del BPA es 35 . Las medidas del arco CD y el ángulo DAC (en grados) son respectivamente. Solución: Dado que BPA es un ángulo exterior, formado por dos secantes, su medida es la semidiferencia de los las medidas de los arcos que intercepta. Es decir mBPA = mdCD mdAB 2 De esta expresión despejamos mdAB, resultando mdCD = 2 mBPA + mdAB = 2 35o + 30o = 100o Por otro lado se tiene que el DAC es un ángulo inscrito que subtiende el arco DC, luego mDAC = 1 2 mdCD = 1 2 100o = 50o 71. La expresión (p + q)p = (r + s)r, se cumple en la situación representada por 76
  • 77. Solución: Al recordar las relaciones métricas en una circunferencia, vemos que los productos de esta forma surgen cuando se tienen dos secantes que se cortan (también aparecen cuando hay semejanzas de triángulos) o una secante y una tangente que se cortan. A partir de estas relaciones tenemos: En la …gura a), la relación es r2 = s (s + p) : En la …gura b), la relación es r(r + s) = p(p + q), la cual es la misma expresión dada. La respuesta es ésta. Para estar más seguros vemos que resulta en las otras. En la …gura c), la relación es r s = p q y en la …gura d), r2 = (p + q), que son diferentes a la dada. Solo b) satisface y por tanto es la respuesta. 72. En la …gura se dan tres semicircunferencias mutuamente tangentes.CD y DA son diámetros de las circunfer- encias menores. El punto B está en la semicircunferencia mayor. BD ? BC . Si BD = 2; entonces el área sombreada es igual a. Solución: El área de la región sombreada es la diferencia entre el área del semicírculo exterior menos las áreas de los semicírculos interiores. Sean r1, r2, R los radios del semicírculo menor, del semicírculo mediano y del semicírculo exterior respectivamente. Luego CD = 2r1, DA = 2r2 y CA = 2R. Como CA = CD + DA, se tiene 2R = 2r1 + 2r2 R = r1 + r2 (1) Al unir B con A y con C, se forma un triángulo rectángulo, con CA como hipotenusa y BD como altura relativa a la hipotenusa. Por el teorema de la altura, BD2 = CD DA 22 = 2r1 2r2 r1 r2 = 1 (2) Como el área de un semicírculo está dada por 1 2 r2 , el área buscada es A = 1 2 R2 r2 1 r2 2 77
  • 78. Al considerar (1) A = 1 2 h (r1 + r2) 2 r2 1 r2 2 i = 1 2 r2 1 + 2r1r2 + r2 2 r2 1 r2 2 = 1 2 2r1r2 = r1r2 Al considerar (2) A = r1r2 = 1 73. Las medidas de los arcos AB y AC se indican en la …gura. La medida del BAC es. Solución: El BAC es un ángulo inscrito en una circunferencia, por tanto su medida es la mitad de la medida del arco que subtiende, en este caso el arco BC. Tenemos que mdBC = 360o mdAB mdAC = 360o 110o 130o = 120o Luego mBAC = 1 2 mdBC = 60o 74. En la …gura, BC une los centros de los círculos tangentes. AB ? BC; BC = 8 y AC = 10, entonces la longitud de la circunferencia pequeña es igual a Solución: Sean R y r los radios de las circunferencias grande y pequeña respectivamente. Como las circunferencias son tangentes exteriormente, R + r = BC = 8. Dado que el 4ABC es rectángulo en B, tenemos R = AB = p 102 82 = 6 y r = 2. Luego la longitud de la circunferencia pequeña resulta C = 2 r = 4 . 78
  • 79. 75. La …gura representa un hexágono regular, ¿cuál es el valor de x? Solución: Todo hexágono regular puede dividirse en seis triángulos equiláteros congruentes. En la …gura se indica que x equivale al doble de la altura de cada triangulo: x = 2h. Como el lado de cada triangulo mide 6 p 3, las alturas miden h = p 3 2 6 p 3 = 9 ) x = 2h = 18 76. La …gura representa un círculo inscrito en un cuadrado que a su vez está inscrito en otro cuadrado. B es punto medio de AC ¿Cuál es el área de la región sombreada? Solución: Si llamamos A1 al área del cuadrado mayor,A2 al área del cuadrado menor y A3 al área del círculo, el área de la región sombreada resulta A = A1 A2 + A3: El lado del cuadrado mayor mide 0:4, luego su área es A1 = 0:16. Como B es punto medio AB = 0:2. Los triángulos que se forman en cada esquina del cuadrado mayor, son rectángulos isósceles, y sus hipotenusas forman los lados del cuadrado menor, por tanto, el lado del cuadrado menor resulta 0:2 p 2 y su área es A2 = 0:2 p 2 2 = 0:08. Como el circulo está inscrito en el cuadrado menor, su diámetro es el lado de dicho cuadrado, y su radio es la mitad o sea r = 0:1 p 2, su área A3 = r2 = 0:1 p 2 2 = 0:0628. Luego A = 0:16 0:08 + 0:0628 = 0:1428 77. Los segmentos AC y BD se cortan en P y son tangentes a las circunferencias en los puntos A, C, B y D. 79
  • 80. Solución: Dado que PB y PC son segmentos tangentes a la circunferencia de la izquierda, desde un mismo punto, son congruentes, luego PC = PB = 19. Como AC = AP + PC, AP = AC PC = 31 19 = 12 78. Seis triángulos equiláteros de 1cm. de lado se unen para formar un hexágono como se muestra en la …gura. Se circunscribe un círculo alrededor del hexágono ¿cuál es el área de la región sombreada? Solución: Tenemos que el área de la región sombreada es el área del circulo menos el área del hexágono. El radio del circulo es la longitud del lado de los triángulos, es decir r = 1, luego su área es r2 = . El área de cada triángulo equilátero es p 3 4 x2 , donde x es el lado del triángulo, y como el lado mide 1, se reduce a p 3 4 . Como hay seis triángulos, el área del hexágono es 6 p 3 4 . Por tanto el área de la región sombreada es A = p 3 2 ! cm2 79. Un triángulo ABC está inscrito en una circunferencia como se muestra en la …gura. Se tiene mA = 50o y mC = 60o . Se trazan tangentes por A; B y Cde manera que se forma el triángulo circunscrito A 0 ; B 0 ; C 0 . Entonces la medida del ángulo A 0 es: Solución: 80
  • 81. Como BA0 y CA0 son tangentes a la circunferencia, los A0BC y A0CB son ángulos semiinscritos que subtienden el arco BC y el ángulo A es un ángulo inscrito que subtiende el mismo arco. Por tanto estos ángulos son congruentes, es decir mA0BC = mA0CB = mA = 50o Luego al considerar el 4A0BC, se tiene mA0 = 180 mA0BC mA0CB = 180o 50o 50o = 80o 80. El triángulo ABC es equilátero y sus lados AC y BC son tangentes a la circunferencia con centro en O y radio p 3. El área del cuadrilátero AOBC es Solución: Se tiene OC?AB, ya que los triángulos OAB y ABC son isósceles. También 4OAC = 4OBC, ya que sus tres lados son congruentes. Como además el 4ABC es equilátero, mACO = 30o , luego el 4OAC es un triángulo 30 –60 y de ahí resulta que OC = 2 p 3 y AC = 3. Tenemos entonces [AOBC] = 2 [OAC] = 2 1 2 p 3 3 = 3 p 3 81. Si un ángulo central de 30 en una circunferencia intercepta un arco de 6m de longitud, entonces el radio de la circunferencia mide. Solución: Se tiene s = r , donde s es la longitud del arco, r el radio de la circunferencia y es el ángulo central correspondiente, medido en radianes. Como = 30o equivale a =6 radianes, tenemos 6 = r 6 r = 36 81
  • 82. 82. En la …gura se tiene una circunferencia de radio 1 y un hexágono regular de lado 1. Si O es el centro de la circunferencia, entonces el área de la región sombreada es. Solución: En vista que el hexágono tiene lado 1 y la circunferencia tiene radio 1, el centro del hexágono es un punto de la circunferencia. La región sombreada puede descomponerse en dos triángulos que tienen la misma base y la misma altura que los triángulos que forman el hexágono. Luego el área buscada es A = 2 p 3 4 12 = p 3 2 0:866 83. Los arcosAB y BC son semicírculos cuyos centros están sobre un diámetro del círculo que se muestra en la …gura.Si BC = 2AB, entonces la razón entre el área de la región sombreada y el área de la región no sombreada es: Solución: Sean r1 el radio del semicírculo mayor,r2 el radio del semicírculo mediano y r3 el radio del semicírculo menor. Se tiene r3 = AB 2 BC = 2AB AC = AB + BC 2r2 = 2AB 2r1 = 3AB r2 = AB r1 = 3 2 AB Luego las áreas de estos semicírculos son: Semicírculo mayor: 1 2 r2 1 = 1 2 3 2 AB 2 = 9 8 AB2 Semicírculo mediano: 1 2 r2 2 = 1 2 AB2 Semicírculo menor: 1 2 r2 3 = 1 2 AB 2 2 = 1 8 AB2 82
  • 83. El área sombreada está dada por: área del semicírculo mayor menos el área del semicírculo mediano más el área del semicírculo menor o sea Área sombreada = 9 8 AB2 1 2 AB2 + 1 8 AB2 = 3 4 AB2 La razón buscada resulta Área sombreada Área no sombreada = 3 4 AB2 3 2 AB2 = 1 2 84. Una moneda circular de radio 1, está sobre una mesa. Si ponemos cuatro monedas más grandes de igual tamaño alrededor de ella, ¿cuál es el radio de las monedas grandes que permite que cada una sea tangente a las dos adyacentes y a la de radio 1? Solución: Sea R el radio de las monedas grandes. Como estas monedas son tangentes a las monedas adyacentes y a la vez son tangentes a la moneda pequeña, al unir los centros de las monedas grandes se forma un cuadrado de lado 2R. Al trazar una diagonal, esta debe pasar por el centro de la moneda pequeña, la cual tiene diámetro 2, luego la longitud de la diagonal resulta 2R + 2. Por tanto, dado que en todo cuadrado de lado x, su diagonal mide p 2x, se cumple en este caso que 2R + 2 = p 2 (2R) 2 p 2 2 R = 2 R = 1 p 2 1 Al racionalizar el denominador obtenemos R = p 2 + 1 85. En la siguiente …gura ABC y AEB son semicírculos, F es el punto medio del diámetro AC; B es punto medio del arco AC y AF = 1. ¿Cuál es el área de la región sombreada? 83
  • 84. Solución: El área de la región sombreada resulta de la diferencia entre el semicírculo AEB y el segmento circular deter- minado por la cuerda AB en el semicírculo ABC. Como F es el punto medio del diámetro AC, B es punto medio del arco AC, resulta BF?AC, luego el 4ABF es un triángulo rectángulo isósceles de cateto 1 y por tanto AB = p 2. AB es diámetro del semicírculo AEB, luego su radio es p 2 2 y el área de este semicírculo resulta A1 = 1 2 p 2 2 !2 = 4 El área del segmento circular, está dada por la diferencia entre el área del sector circular que lo contiene y el área del triángulo determinado por la cuerda y los radios extremos. En este caso el sector circular correspondiente tiene ángulo central de 90o y radio 1, por tanto su área es la cuarta parte del área de un círculo de radio 1 o sea 1 4 y el triángulo correspondiente tiene base 1 y altura 1, luego su área es 1 2 . El área del segmento circular resulta A2 = 1 4 1 2 Finalmente el área buscada es A = A1 A2 = 4 4 1 2 = 1 2 86. Si el radio de un círculo aumenta en unidades, ¿cuánto aumenta su perímetro? Solución: Sean L y L0 los perímetros del círculo original y el círculo con el radio aumentado, respectivamente. Luego L = 2 r y L0 = 2 (r + ) = 2 r + 2 2 . El aumento es la diferencia 4 = L0 L = 2 r + 2 2 2 r = 2 2 87. Dos semicírculos de radio 3 están inscritos en un semicírculo de radio 6 como se muestra en la …gura. Un círculo de radio r es tangente a los tres semicírculos. ¿Cuánto vale r ? Solución: 84
  • 85. Cuando se tienen círculos tangentes exteriormente, la distancia entre los centros es la suma de los radios, y cuando son tangentes interiormente, la distancia entre los centros es la diferencia entre los radios. Además en ambos casos los centros y el punto de tangencia están alineados. Sean A, B, C y D los centros de los semicírculos y del círculo interior como se muestra en la …gura. Se tiene AB = AD = 3 + r, CA = 6 r, BC = CD = 3. Como 4ABD es isósceles y C es punto medio de BD, AC?BC, luego el 4ABC es rectángulo en C y por tanto sus lados cumplen con el teorema de Pitágoras. Luego (3 + r) 2 = (6 r) 2 + 32 9 + 6r + r2 = 36 12r + r2 + 9 18r = 36 r = 2 88. En la …gura los círculos adyacentes son tangentes y tienen radio 1. ¿Cuánto vale el área de la región sombreada? Solución: Al considerar el círculo central y dos círculos externos contiguos, vemos que encierran la sexta parte del área buscada. Vemos también que esta fracción corresponde al área de un triángulo equilátero de lado 2 menos tres sectores circulares de radio 1 y de 60o cada uno, que juntos forman un semicírculo de radio 1. 85
  • 86. Luego A = 6 "p 3 4 22 1 2 12 # = (6 p 3 3 )u2 89. En la …gura, mBCA = 90o ; BA = 5y AC = 3: ¿Cuál es el área del círculo con centro en O? Solución: Como el 4ABCes rectángulo en C, aplicamos el Teorema de Pitágoras para hallar BC BC = p AB2 AC2 = p 52 32 = 4 Como BC es diámetro del círculo, se tiene r = 2 y su área resulta A = r2 = 4 90. El lado mayor del rectángulo de la …gura mide 20. La curva trazada en su interior está formada por cinco semicircunferencias ¿cuál es la longitud de la curva? Solución: Se observa que la curva está formada por 5 semicircunferencias, cuyos diámetros suman 20, luego cada diámetro mide 20 5 = 4 y los respectivos radios la mitad o sea 2 unidades. Luego L = 5 1 2 2 r = 5 r = 5 2 = 10 91. La …gura muestra dos segmentos perpendiculares tangentes a ambas circunferencias, las cuales son tangentes entre sí. Si el radio de la circunferencia pequeña mide 1, entonces el radio de la circunferencia más grande mide Solución: 86